pat an anew

126
1 7.0 Патологічна анатомія 1 Multiple oval ulcers along the intestine were revealed on autopsy of the person, who died from diffuse of peritonitis in the distant part of the small intestine. Bottom parts of the ulcers are clear, smooth, formed with muscular or serous covering, edges of ulcers are flat, rounded. There are perforations up to 0,5 cm in diameter in two ulcers. What diseasis can be diagnosed? A Typhoid fever B Dysentery C Cholera D Tuberculosis E Typhus 2 Patient suffering from trombophlebitis of the deep veins suddenly died. Autopsy has shown freely lying red friable masses with dim crimped surface in the trunk and bifurcation of the pulmonary artery. What pathologic process was revealed by morbid anatomist? A Tromboembolism B Thrombosis C Tissue embolism D Embolism with foreign body E Fat embolism 3 Examination of a patient revealed a dense, movable skin tumour that is standing out distinctly from the surrounding tissues. Its section is found to be white and composed of fibrous tissue. Microscopic examination revealed interlacing collagen fibers and few cells. What tumour is it? A Fibroma B Myoma C Histiocytoma D Dermatofibroma E Desmoid 4 A 50-year-old man has felt vague abdominal discomfort within past 4 months. Physical examination revealed no lymphadenopathy, and no abdominal masses or organomegaly at palpation. Bowel sounds are heard. An abdominal CT scan shows a 20 cm retroperitoneal soft tissue mass obscuring the left psoas muscle. A stool specimen tested for occult blood is negative. Which of the following neoplasms is this man most likely to have? A Lipoma B Melanoma C Hamartoma D Adenocarcinoma E Lymphoma 5 A 40-year-old woman has had a feeling of abdominal discomfort for the past 8 months. On pelvic examination, there is the right adnexal mass. Abdominal CT scan demonstrates a 7 cm cystic mass involving the right ovary with small areas of calcification. The uterus is normal in size. The right fallopian tube and ovary have been removed surgically. Grossly, the mass on sectioning is filled with abundant hair and sebum. Microscopically, the mass has glandular spaces lined by columnar epithelium, squamous epithelium with hair follicles, cartilage, and dense connective tissue. What type of tumour is it?

Upload: gshchurovskiy

Post on 27-Nov-2014

368 views

Category:

Documents


4 download

TRANSCRIPT

Page 1: Pat an Anew

1

7.0 Патологічна анатомія1

Multiple oval ulcers along the intestine were revealed on autopsy of the person, who died from diffuse of peritonitis in the distant part of the small intestine. Bottom parts of the ulcers are clear, smooth, formed with muscular or serous covering, edges of ulcers are flat, rounded. There are perforations up to 0,5 cm in diameter in two ulcers. What diseasis can be diagnosed?

A Typhoid feverB DysenteryC CholeraD TuberculosisE Typhus

2

Patient suffering from trombophlebitis of the deep veins suddenly died. Autopsy has shown freely lying red friable masses with dim crimped surface in the trunk and bifurcation of the pulmonary artery. What pathologic process was revealed by morbid anatomist?

A Tromboembolism B Thrombosis C Tissue embolism D Embolism with foreign bodyE Fat embolism

3

Examination of a patient revealed a dense, movable skin tumour that is standing out distinctly from the surrounding tissues. Its section is found to be white and composed of fibrous tissue. Microscopic examination revealed interlacing collagen fibers and few cells. What tumour is it?

A Fibroma B Myoma C Histiocytoma D Dermatofibroma E Desmoid

4

A 50-year-old man has felt vague abdominal discomfort within past 4 months. Physical examination revealed no lymphadenopathy, and no abdominal masses or organomegaly at palpation. Bowel sounds are heard. An abdominal CT scan shows a 20 cm retroperitoneal soft tissue mass obscuring the left psoas muscle. A stool specimen tested for occult blood is negative. Which of the following neoplasms is this man most likely to have?

A LipomaB MelanomaC HamartomaD AdenocarcinomaE Lymphoma

5

A 40-year-old woman has had a feeling of abdominal discomfort for the past 8 months. On pelvic examination, there is the right adnexal mass. Abdominal CT scan demonstrates a 7 cm cystic mass involving the right ovary with small areas of calcification. The uterus is normal in size. The right fallopian tube and ovary have been removed surgically. Grossly, the mass on sectioning is filled with abundant hair and sebum. Microscopically, the mass has glandular spaces lined by columnar epithelium, squamous epithelium with hair follicles, cartilage, and dense connective tissue. What type of tumour is it?

Page 2: Pat an Anew

2

A TeratomaB Squamous cell carcinoma of ovaryC MelanomaD Sarcoma of ovaryE Metastase of cervical carcinoma

6

A man died 8 days after the beginning of the disease. He was diagnosed with dysentery. Atthe autopsy it was found out a thickened wall of the sigma and rectum, fibrinous membraneon the surface of mucous membrane. Histologically: there is a deep necrosis of mucous

membrane with infiltration of necrotic masses with fibrin. What kind of colitis does correspond to the changes?

A DiphtheriticB CatarrhalC UlcerativeD ChronicE Gangrenous

7

A woman suffering from dysfunctional metrorrhagia was made a diagnostic abortion. Histologically in the scrape there were a lot of small stamped glandulars covered with multirowed epithelium. The lumens of some glandulars were cystically extended. Choose the variant of general pathologic process in the endometrium.

A Glandular-cystic hyperplasia of endometriumB Atrophy of endometriumC Metaplasia of endometriumD Neoplasm of endometriumE Hypertrophic growth

8

A 46 year-old man complains of difficult nose breathing. Mikulich cells, storage of epithelioid cells, plasmocytes, lymphocytes, hyaline balls are discovered in the biopsy material of the nose thickening. What is the most likely diagnosis?

A ScleromaB Virus rhinitisC Allergic rhinitis D Rhinovirus infection E Meningococcal nasopharyngitis

9

Extensive thromboembolic infarction of the left cerebral hemispheres, large septic spleen, immunocomplex glomerulonephritis, ulcers on the edges of the aortic valves, covered with polypous thrombus with colonies of staphylococcus were revealed on autopsy of the young man who died in coma. What disease caused cerebral thromboemboly?

A Septic bacterial endocarditisB Septicemia C Acute rheumatic valvulitisD SepticopyemiaE Rheumatic thromboendocarditis

10

A patient ill with diabetes mellitus felt acute pain in his right foot. Objectively: foot thumb is

Page 3: Pat an Anew

3

black, foot tissues are edematous, there are foci of epidermis desquamation, stinking discharges. What clinicopathological form of necrosis is it?

A Moist gangrene B Bedsore C Sequestrum D Dry gangrene E Infarction

11

A denaturation of proteins can be found in some substances. Specify the substance that is used for the incomplete denaturation of hemoglobin:

A UreaB TolueneC Sulfuric acidD Nitric acidE Sodium hydroxide

12

During surgery in a 17-year-old patient it was revealed the tumour of 4,5х5,0х3,5 sm in size on the lower surface of the liver with subserose localization, of dark-red color. On the section tumour has cavities with marked amount of blood. What is preliminary diagnosis?

A Cavernous hemangiomaB Capillar hemangiomaC HemangiopericytomaD HemangioendotheliomaE Lymphangioma

13

A sick man with high temperature and a lot of tiny wounds on the body has been admitted to the hospital. Lice have been found in the folds of his clothing. What disease can be suspected in the patient?

A Epidemic typhusB TularemiaC ScabiesD MalariaE Plague

14

On autopsy it is revealed that kidneys are enlarged, surface is large-granular because of multiple cavities with smooth wall, which are filled with clear fluid. What kidney disease did the patient have?

A Polycystic kidneyB Necrotic nephrosisC Pyelonephritis D Glomerulonephritis E Infarction

15

On autopsy it is revealed enlarged dense right lung, fibrin layers on the pleura. Lung tissue is light green color on incision with muddy liqued exudates. What lung disease are these symptoms typical for?

Page 4: Pat an Anew

4

A Lung-feverB Bronchopneumonia,

C Interstitial pneumoniaD Pulmonary gangreneі E Fibrosing alveolitis

16

On autopsy it is revealed: soft arachnoid membrane of the upper parts of cerebral hemisphere is plethoric, it is of yellowish-green color, soaked with purulent and fibrose exudate, it lookes like cap. For what disease is it characteristical picture

A Meningococcal meningitisB Tuberculous meningitisC Influenza meningitis D Meningitis at anthrax E Meningitis at typhus

17

On autopsy of the 58-year-old man it is revealed: mitral valve is deformed, thickened, not totally closed. Microscopically: centers of collagen fibers are eosinophilic, have positive fibrin reaction. The most likely it is:

A Fibrinoid swellingB Fibrinoid inflammationC Mucoid swellingD HyalinosisE Amyloidosis

18

Chronic inflammation and transformation of the one-layer ciliated epithelium into multiple-layers flat epithelium was revealed in the thickened mucous membrane of the bronchus bioptate of the patient with smoke abuse. Which of the processes is the most likely?

A MetaplasiaB Hyperplasia of the epitheliumC Squamous cancerD LeucoplaciaE Epithelium hypertrophy

19

For a long time a 49-year-old woman was suffering from glomerulonephritis which caused death.On autopsy it was revealed that kidneys size was 7х3х2.5 sm, weight is 65,0 g, they are dence and small-grained. Microscopically: fibrinogenous inflammation of serous and mucous capsules, dystrophic changes of parenchymatous organs, brain edema. What complication can cause such changes of serous capsules and inner organs?

A UraemiaB AnemiaC Sepsis D DIC-syndromeE Thrombopenia

Page 5: Pat an Anew

5

20

A patient died under conditions of cardiovascular insufficiency. Autopsy results: postinfarction cardiosclerosis, myocardium hypertrophy and dilatation of its cavities, especially of its right ventricle. Liver is enlarged, its surface is smooth, incision revealed that it was plethoric, with dark-red specks against the background of brownish tissue. Histologically: plethora of central parts of lobules; peritheral parts around portal tracts contain hepatocytes in a state of adipose degeneration. How are these liver changes called?

A Nutmeg liver B Pseudonutmeg liver C Amyloidosis D Liver cirrhosis E Liver steatosis

21

A 59-year-old man has signs of the parenchymatous jaundice and portal hypertension. On histological examination of the puncture of the liver bioptate, it was revealed: beam-lobule structure is affected, part of hepatocytes has signs of fat dystrophy, port-portal connectivetissue septa with formation of pseudo-lobules,with periportal lympho-macrophage

infiltrations. What is the most probable diagnosis?A Liver cirrhosisB Alcohol hepatitisC Chronic hepatosisD Viral hepatitisE Toxic dystrophy

22

On microscopic examination of the enlarged neck gland of a 14-year-old girl it was revealed destruction of the tissue structure of the node, absence of the lymph follicles, sclerotic and necrosis parts, cell constitution of the node is polymorphous, lymphocites, eosinophiles, atypical cells of the large size with multiple-lobule nuclei (Beresovsky-Shternberg cells) and onenucleus cells of the large size are present. What is the most likely diagnosis?

A LymphogranulomatousB Acute lympholeucosisC Chronic lympholeucosisD Berkitt's lymphoma E Fungous mycosis

23

A female patient suffering from bronchial asthma had got a viral infection that provoked status asthmaticus with fatal outcome. Histological examination of lungs revealed spasm and edema of bronchioles, apparent infiltration of their walls with lymphocytes, eosinophils and other leukocytes; labrocyte degranulation. What mechanism of hypersensitivity underlies the described alterations?

A Reagin reaction B Inflammatory C Autoimmune D Immune complex E Immune cytolysis

Page 6: Pat an Anew

6

24

On autopsy of the 40-year-old woman suffering from rheumatic arthritis, the enlarged solid spleen was revealed. On section its tissue is of the mahogany color with enlarged follicles, which look like semi-transparent grayish-whitish grains. What pathological process is the most likely?

A Sago spleenB Glaze spleenC Waxy spleenD Hyaline spleenE Porphyric spleen

25

Local lymphonodules enlarged near the infected wound. Increased amount of macrophages, lymphocytes, lymphatic follicles in the cortical layer and large amount of plasma cells were revealed on histological examination. What process in the lymphatic nodules represent these histological changes?

A Antigen stimulationB Acquired insufficiency of the lymphoid tissueC Innate insufficiency of the lymphoid tissueD Tumour transformationE Hypersensibility reaction

26

On autopsy of the man with alcohol abuse for a long time it was revealed: dense, small-knobby, small size liver. Microscopically: small pseudo-lobules, divided with thin layers of connective tissue with lymphomacrophagial infiltrates; hepatocytes in the state of globular fatty dystrophy. What is the most likely diagnosis?

A Alcohol cirrhosisB Chronic active alcohol hepatitisC Chronic persistent alcohol hepatitisD Toxic liver dystrophyE Fatty hepatosis

27

An 8-year-old child was admitted to the infectious department with fever (up to $38^oC$) and punctuate bright-red skin rash. The child was diagnosed as having scarlet fever. Objectively: mucous membrane of pharynx is apparently hyperaemic and edematic, the tonsils are enlarged and have dull yellowish-grey foci with some black areas. What inflammation is the reason for the pharynx alterations?

A Purulent necrotic B Fibrinous C Haemorrhagic D Serous E Catarrhal

28

A 30-year-old patient with bacteriologically proved dysentery developed the signs of paraproctitis. What is the stage of local changes in this patient?

A Ulceration stageB Fibrinous colitisC Follicular colitisD Catarrhal colitisE Healing of the ulcers stage

Page 7: Pat an Anew

7

29

A patient who has been abusing tobacco smoking for a long time has got cough accompanied by excretion of viscous mucus; weakness after minor physical stress, pale skin. The patient has also lost 12,0 kg of body weight. Endoscopic examination of biosy material his illness was diagnosed as squamous cell carcinoma. Name a pathological process that preceded formation of the tumour:

A Metaplasia B Hypoplasia C Hyperplasia D Necrosis E Sclerosis

30

Diagnostic scraping was performed to the woman with dysfunctional uterine bleeding. Multiple convoluted glands, ganglially dilated cavities of some glands were revealed histologically in the scrape. Name the type of general pathological process.

A Glandulo-gangliac hyperplasia B AtrophyC MetaplasiaD DisplasiaE Hypertrophic excrescence

31

Tuberculine was injected intracutaneously to the child for tuberculin test. Marked hyperemia, tissue infiltration developed on the place of injection in 24 hours. What mechanism caused these modifications?

A Cells cytotoxityB Reagin type cytotoxityC Antibody cytotoxityD Granuloma formation E Immunocomplex cytotoxity

32

The intraoperational biopsy of mammal gland has revealed the signs of atypical tissue with disorder of parenchyma stroma proportion with domination of the last, gland structures ofthe different size and shape, lined with single-layer proliferative epithelium. What is the

most appropriate diagnosis?A FibroadenomaB PapillomaC Noninfiltrative cancerD Infiltrative cancerE Mastitis

33

Arterial hypertension, hyperglycemia, glucosuria were observed clinically for a long time in the patient with upper type of obesity. Death was due to the cerebral haemorrhage. Basophilic hypophysis adenoma, hyperplasia of adrenal gland cortex were revealed on pathomorphological examination. What is the likely diagnosis?

A Cushing diseaseB Diabetes mellitusC Acromegaly

Page 8: Pat an Anew

8

D Hypophysis nanismE Adiposogenitalis dystrophy

34

On autopsy it was revealed: large (1-2 cm) brownish-red, easy crumbling formations covering ulcerative defects on the external surface of the aortic valve. What is the most likely diagnosis?

A Polypus-ulcerative endocarditisB Recurrent warty endocarditisC Acute warty endocarditisD Fibroplastic endocarditisE Diffusive endocarditis

35

Purulent endometritis with fatal outcome was progressing in the woman after abortion performed not at the hospital. On autopsy multiple lung abscesses, subcapsule ulcers in the kidneys, spleen hyperplasia were revealed. What form of sepsis developed in the patient?

A SeptopyemiaB SepticemiaC ChroniosepsisD Lung sepsisE Urosepsis

36

Autopsy of a 73-year-old man who had been suffering from the coronary heart disease along with cardiac insufficiency for a long time revealed: nutmeg liver, brown induration of lungs, cyanotic induration of kidneys and spleen. What kind of circulation disorder was the cause of such effects?

A General chronic venous congestion B Arterial hyperaemia C General acute venous congestion D Acute anaemia E Chronic anaemia

37

A 22 year old patient from the West Ukraine complains of laboured nasal breathing. Morphological examination of biopsy material of nasal mucous membrane revealed lymphoid, epithelioid, plasma cells as well as Mikulicz's cells. What is the most probable diagnosis?

A Rhinoscleroma B Glanders C Tuberculosis D Leprosy E Syphilis

38

Autopsy of a man who had been working as a miner for many years and died from cardiopulmonary decompensation revealed that his lungs were airless, sclerosed, their apexex had emphysematous changes, the lung surface was greyish-black, the incised lung tissue was coal-black. What disease caused death?

A Anthracosis

Page 9: Pat an Anew

9

B Silicosis C Talcosis D Asbestosis E Aluminosis

39

Examination of coronary arteries revealed atherosclerotic calcific plaques that close vessel lumen by 1/3. The muscle has multiple whitish layers of connective tissue. What process was revealed in myocardium?

A Diffuse cardiosclerosis B Tiger heart C Postinfarction cardiosclerosis D Myocarditis E Myocardium infarction

40

A 63 year old male patient who had been suffering from chronic diffuse obstructive disease, pulmonary emphysema, for 15 years died from cardiac insufficiency. Autopsy revealed nutmeg liver cirrhosis, cyanotic induration of kidneys and spleen, ascites, edemata of lower limbs. These changes of internal organs are typical for the following disease:

A Chronic right-ventricular insufficiency B Acute right-ventricular insufficiency C Chronic left-ventricular insufficiency D Acute left-ventricular insufficiency E General cardiac insufficiency

41

Microscopical examination of an enlarged cervical lymph node revealed blurring of its structure, absence of lymphoid follicles; all the microscopic fields showed cells with roundish nuclei and thin limbus of basophil cytoplasm. It is known from the clinical data that other groups of lymph nodes are also enlarged as well as spleen and liver. What disease might be suspected?

A Lymphoid leukosis B Lymphogranulomatosis C Lymphosarcoma D Myeloid leukosis E Multiple myeloma

42

A worker of a cattle farm fell acutely ill and then died from the progressing intoxication. Autopsy revealed enlarged, hyposthenic spleen of dark-cherry colour when dissected; excessive pulp scraping. At the base and fornix of brain pia maters are edematous, soakedwith blood, dark-red ("scarlet hat"). Microscopic examination revealed serous

haemorrhagic inflammation of brain tissues and tunics along with destruction of small vessel walls. What is the most likely diagnosis?

A Anthrax B Tularemia C Brucellosis D Plaque E Cholera

Page 10: Pat an Anew

10

43

Histological examination of a skin tissue sampling revealed granulomas consisting of macrophagal nodules with lymphocytes and plasmatic cells. There are also some big macrophages with fatty vacuoles containing causative agents of a disease packed up in form of spheres (Virchow's cells). Granulation tissue is well vascularized. What disease is this granuloma typical for?

A Lepra B Tuberculosis C Syphilis D Rhinoscleroma E Glanders

44

A 40 year old man noticed a reddening and an edema of skin in the area of his neck that later developed into a small abscess. The incised focus is dense, yellowish-green. The puscontains white granules. Histological examination revealed drusen of a fungus, plasmatic

and xanthome cells, macrophages. What type of mycosis is the most probable? A Actinomycosis B Aspergillosis C Candidosis D Sporotrichosis E Coccidioidomycosis

45

A physician examined a patient and found inguinal hernia. Through what anatomic formation does it penetrate into the skin?

A $Hiatus$ $saphenus$ B $Anulus$ $femoralis$ C $Canalis$ $adductorius$ D $Lacuna$ $musculorum$ E $Anulus$ $inguinalis$ $superficialis$

46

Autopsy of a man who died from burn disease revealed brain edema, liver enlargement as well as enlargement of kidneys with wide light-grey cortical layer and plethoric medullary area. Microscopic examination revealed necrosis of tubules of main segments along with destruction of basal membranes, intersticium edema with leukocytic infiltration and haemorrhages. What is the most probable postmortem diagnosis?

A Necrotic nephrosis B Tubulointerstitial nephritis C Pyelonephritis D Gouty kidney E Myeloma kidney

47

A 30 year old man had been suffering from acute respiratory disease and died from cardiopulmonary decompensation. Autopsy revealed fibrinous-haemorrhagic inflammation in the mucous membrane of larynx and trachea, destructive panbronchitis, enlarged lungs that look black due to the multiple abcesses, haemorrhages, necrosis. What is the most probable postmortem diagnosis?

A Influenza B Parainfluenza C Respiratory syncytial infection

Page 11: Pat an Anew

11

D Measles E Adenoviral infection

48

A man with a wound of his limb that had been suppurating for a long time died from intioxication. Autopsy revealed extreme emaciation, dehydration, brown atrophy of liver, myocardium, spleen and cross-striated muscles as well as renal amyloidosis. What diagnosis corresponds with the described picture?

A Chroniosepsis B Septicopyemia C Septicemia D Chernogubov's syndrome E Brucellosis

49

6 months after delivery a woman had uterine bleeding. Gynecological examination revealedin the uterine cavity a dark-red tissue with multiple cavities that resembled of "sponge".

Microscopic examination of the tumour revealed some atypic light epithelial Langhans cells and giant cells of cyncytiotrophoblast in blood lacunas. What tumour is it?

A Chorioepithelioma B Squamous cell nonkeratinous carcinoma C Adenocarcinoma D Fibromyoma E Vesicular mole

50

A patient with android-type obesity had been suffering from arterial hypertension, hyperglycemia, glycosuria for a long time and died from the cerebral haemorrhage. Pathologic examination revealed pituitary basophil adenoma, adrenal cortex hyperplasia. What is the most likely diagnosis?

A Itsenko-Cushing's syndrome B Diabetes mellitus C Acromegalia D Pituitary nanism E Adiposogenital dystrophy

51

Mucous membrane of the right palatine tonsil has a painless ulcer with smooth lacquer fundus and regular cartilagenous edges. Microscopically: inflammatory infiltration that consists of lymphocytes, plasmocytes, a small number of neutrophils and epithelioid cells; endovasculitis and perivasculitis. What disease is it?

A Syphilis B Actinomycosis C Tuberculosis D Pharyngeal diphtheria E Ulcerous necrotic Vincent's angina

52

Autopsy of a man with a malignant stomach tumour who had died from cancer intoxication revealed in the posteroinferior lung fields some dense, grayish-red irregular foci protrudingabove the section surface. Microscopic examination revealed exudate containing a large

amount of neutrophils in the lumen and walls of small bronchi and alveoles. Such

Page 12: Pat an Anew

12

pulmonary alterations indicate the following disease: A Acute purulent bronchopneumonia B Acute bronchitis C Croupous pneumonia D Intermittent pneumonia E Acute serous bronchopneumonia

53

Microscopical examination of a removed appendix revealed an edema, diffuse neutrophilic infiltration of appendix wall along with necrosis and defect of mucous membrane with affection of its muscle plate. What appendicitis form was developed?

A Ulcerophlegmonous B Phlegmonous C Gangrenous D Superficial E Apostematous

54

A 39 y.o. woman went through an operation in course of which surgeons removed her uterine tube that was enlarged and a part of an ovary with a big cyst. Histological examination of a tube wall revealed decidual cells, chorion villi. What was the most probable diagnosis made after examination of the uterine tube?

A Tubal pregnancy B Placental polyp C Choriocarcinoma D Papyraceous fetus E Lithopedion

55

Autopsy of a 1,5-year-old child revealed haemorrhagic skin rash, moderate hyperaemia and edema of nasopharyngeal mucous membrane, small haemorrhages in the mucous membranes and internal organs; dramatic dystrophic alterations in liver and myocardium; acute necrotic nephrosis; massive haemorrhages in the adrenal glands. What disease are these alterations the most typical for?

A Meningococcal infection B Scarlet fever C Diphtheria D Measles E Epidemic typhus

56

48 hours after performing tuberculin test (Mantoux test) to a child a 10 mm papule appeared on the spot of tuberculin introduction. What hypersensitivity mechanism underlies these changes?

A Cellular cytotoxicity B Anaphylaxis C Antibody-dependent cytotoxicity D Immune complex cytotoxicity E Granulomatosis

57

Colonoscopy of a patient ill with dysentery revealed that mucous membrane of his large

Page 13: Pat an Anew

13

intestine is hyperemic, edematic, its surface was covered with grey-and-green coats. Name the morphological form of dysenteric collitis:

A Fibrinous B Catarrhal C Ulcerous D Purulent E Necrotic

58

A patient has been syffering from diarrhea for 5 day. On the fith day colonoscopy revealed that membrane of rectum was inflamed, there were greyish-green films closely adhering to the subjacent tissue. What is the most probable diagnosis?

A Dysentery B Typhoid fever C Nonspecific ulcerous colitis D Salmonellosis E Crohn's disease

59

Autopsy of a 48 y.o. man revealed a round formation 5 cm in diameter with clear-cut outlines in the region of the 1st segment of his right lung. This formation was encircled with a thin layer of connective tissue full of white brittle masses. Make a diagnosis of the secondary tuberculosis form:

A Tuberculoma B Caseous pneumonia C Acute cavernous tuberculosis D Acute focal tuberculosis E Fibrous cavernous tuberculosis

60

A man had worked in a coal mine for over 20 years. After his death autopsy revealed that his lungs were dense, grayish-black and had large areas of neogenic connective tissue containing a lot of microphages with black pigment in the cytoplasm. What is the most likely diagnosis?

A Anthracosis B Anthracosilicosis C Silicoanthracosis D Talcosis E Siderosis

61

Skin of a man who died from cardiac insufficiency has an eruption in form of spots and specks. There are also bedsores in the area of sacrum and spinous vertebral processes. Microscopical examination of CNS, skin, adrenal glands revealed in the vessels of microcirculatory bed and in small arteries destructive-proliferative endothrombovasculitis with Popov's granulomas; interstitial myocarditis. What diagnosis corresponds with the described picture?

A Spotted fever B Q fever C Enteric fever D Nodular periarteritis E HIV

Page 14: Pat an Anew

14

62

Autopsy of a 17 year old girl who died from pulmonary failure revealed a small area of caseous necrosis in the inferior lobe of the right lung, and occurences of caseous necrosis in the bronchopulmonary, bronchial and bifurcational lymph nodes. What is the most probable postmortem diagnosis?

A Primary tuberculosis B Hematogenous progression of primary tuberculosis C Hematogenous tuberculosis with predominant lung affection D Tuberculoma E Caseous pneumonia under secondary tuberculosis

63

Autopsy of a man who died from the sepsis in his femoral bone revealed phlegmonous inflammation that affected the marrow, haversian canals and periosteum. Under the periosteum there are multiple abscesses, adjoining soft tissues of thigh also have signs of phlegmonous inflammation. What pathological process was described?

A Acute hematogenous osteomyelitis B Osteoporosis C Chronic hematogenous osteomielitis D Osteopetrosis E -

64

An experimental animal was first sensibilized whereupon an antigen dose was introduced subcutaneously. This injection resulted in the development of a fibrinous inflammation with alteration of vessel walls, basal substance and fibrous structures of connective tissue in form of mucoid and fibrinoid swelling and necrosis. What immunological reaction took place?

A Immediate hypersensitivity B Delayed-type hypersensitivity C Reaction of transplantation immunity D Normergic reaction E Granulomatosis

65

Examination of a 55 year old woman revealed under the skin of submandibular area a movable slowly growing pasty formation with distinct borders 1,0x0,7 cm large. Histological examination revealed lipocytes that form segments of diffrent forms and sizes separated from each other by thin layers of connective tissue with vessels. What is the most probable diagnosis?

A Lipoma B Fibroma C Angioma D Liposarcoma E Fibrosarcoma

66

A 4 year old child complained of pain during deglutition, indisposition. Objectively: palatine arches and tonsils are moderately edematic and hyperemic, there are greyish-white films up to 1 mm thick closely adhering to the subjacent tissues. What pathological process are these changes typical for?

A Inflammation

Page 15: Pat an Anew

15

B Dystrophy C Necrosis D Metaplasia E Organization

67

A 9 m.o. child has delayed dentition, it is also out of order. Upper jaw configuration is horizontal ("high" palate); microscopically - irregular mineralization of tooth enamel, wrinkled enamel prisms, some of them are vacuolized. Predentin zone is extended; there are solitary denticles. What disease is it?

A Early rickets B Late rickets C Osteomalacia D Gout E Hypervitaminosis D

68

Microscopical renal examination of a 36 y.o. woman who died from renal insufficiency revealed in the glomerules proliferation of capsule nephrothelium as well as of podocytes and phagocytes accompanied by formation of "crescents", capillary loop necrosis, fibrinousthrombs in their lumens; sclerosis and hyalinosis of glomerules, atrophy of tubules and

fibrosis of renal stroma. What is the most probable diagnosis? A Subacute glomerulonephritis B Acute glomerulonephritis C Chronic glomerulonephritis D Focal segmentary sclerosis E Membranous nephropathy

69

A forensic medical expert examines the body of a 58 y.o. man who had been consuming large amounts of alcochol for a long time and died at home. Microscopicaly: the right lung is dense and enlarged, its incision revealed that the tissue is greyish and homogenous, pleura is covered with greyish layers. Microscopically - alveolar cavities contain fibrin, hemolyzed erythrocytes. Make a diagnosis:

A Croupous pneumonia B Focal pneumonia C Interstitial pneumonia D Primary pulmonary tuberculosis E Caseous pneumonia

70

Autopsy of a 50-year-old man revealed the following changes: his right lung was moderately compact in all parts, the dissected tissue was found to be airless, fine-grained, dryish. Visceral pleura had greyish-brown layers of fibrin. What is the most likely diagnosis?

A Croupous pneumonia B Tuberculosis C Bronchopneumonia D Interstitial pneumonia E Pneumofibrosis

71

Page 16: Pat an Anew

16

Autopsy of a 56 y.o. man revealed in the right temporal part of brain a big focus of softened grey matter that was semi-liquid and light grey. Arteries of cerebral tela contain multiple whitish-yellow thickenings of intima that abruptly narrow the lumen. What is your diagnosis?

A Ischemic stroke B Brain abscess C Hemorrhage D Hemorrhagic infarction E Brain edema

72

A 22 y.o. woman has enlarged lymph nodes. Histologically: a lymph node contains lymphocytes, histiocytes, reticular cells, small and big Hodgkin's cells, multinucleated Sternberg cells, isolated foci of caseous necrosis. What disease are these changes typical for?

A Lymphogranulomatosis B Lymphosarcoma C Chronic leukosis D Acute leukosis E Lung cancer metastasis

73

Analysis of a punction biopsy material of liver revealed hepatocyte dystrophy with necrosesas well as sclerosis with disorder of beam and lobulous structure, with formation of

pseudolobules and regenerative nodes. What is the most probable diagnosis: A Liver cirrhosis B Chronic hepatosis C Chronic hepatitis D Progressive massive liver necrosis E Acute hepatitis

74

Autopsy of a man, who had been suffering from the multiple bronchiectasis for 5 years and died from chronic renal insufficiency, revealed that kidneys were dense and enlarged, with thickened cortical layer of white colour with greasy lustre. What renal disease might be suspected?

A Secondary amyloidosis B Glomerulonephritis C Chronic pyelonephritis D Necrotic nephrosis E -

75

Autopsy of a 49-year-old woman who died from chronic renal insufficiency, revealed: kidneys were dense, reduced, multicoloured, with haemorrhagic areas. Microscopic examination revealed some hematoxylin bodies in the nuclei of the renal tubule epithelium; "wire-loop" thickening of the glomerular capillary basement membrane; here and there in the capillaries some hyaline thrombi and foci of fibrinoid necrosis were present. What is themost likely diagnosis?

A Systemic lupus erythematosus B Rheumatism C Arteriosclerotic pneumosclerosis D Amyloidosis

Page 17: Pat an Anew

17

E Atherosclerotic nephrosclerosis

76

Unpainfull formation without marked borders appeared in the soft tissues of the thigh in the young man. On the tissue bioptate the formation lookes like a meat of a fish, consisting of the immature fibroblast-like cells with multiple mitosis, which grow through the muscles. What is the most likely diagnosis?

A FibrosarcomaB MyosarcomaC FibromaD CancerE Myoma

77

A 45 y.o. patient consulted a doctor about plaque-shaped formation on his neck. Histological examination of biopsy skin material revealed tumourous cells of round and ovalform with thin ring of basophilic cytoplasma that resemble of cells of basal epidermal layer. What tumour is it?

A Basalioma B Epidermal cancer C Hydradenoma D Trichoepithelioma E Syringoadenoma

78

A 63 y.o. man fell ill with acute tracheitis and bronchitis accompanied by bronchial pneumonia. On the 10th day the patient died from cardiopulmonary insufficiency. Autopsy revealed fibrinous hemorrhagic laryngotracheobronchitis; lungs were enlarged, their incision revealed the "coal-miner's" effect caused by interlacing of sections of bronchial pneumonia, hemorrhages into the pulmonary parenchyma, acute abscesses and atelectases. Internal organs have discirculatory and dystrophic changes. What is the most probable diagnosis?

A Influenza, severe form B Moderately severe influenza C Parainfluenza D Respiratory syncytial infection E Adenoviral infection

79

Autopsy of a man who died from influenza revealed that his heart was slightly enlarged, pastous, myocardium was dull and had specks. Microscopical examination of myocardium revealed signs of parenchymatous adipose and hydropic dystrophy; stroma was edematic with poor macrophagal and lymphocytic infiltration, vessels were plethoric; perivascular analysis revealed petechial hemorrhages. What type of myocarditis was developed in this case?

A Serous diffuse B Interstitial proliferative C Serous focal D Purulent E Granulomatous

80

Page 18: Pat an Anew

18

A boy is 7 y.o. Objectively: against the background of hyperemic skin there is knobby bright-pink rash on his forehead, neck, at the bottom of abdomen, in the popliteal spaces; nasolabial triangle is pale. Examination of oropharyngeal surface revealed localized bright-red hyperemia; tonsils are swollen, soft, lacunas contain pus, tongue is crimson. Cervical lymph nodes are enlarged, dense and painful. What is the most probable diagnosis?

A Scarlet fever B Rubella C Whooping cough D Diphtheria E Infectious mononucleosis

81

Gynecological examination of the uterine cervix in a 30-year-old woman revealed some bright-red lustrous spots that easily bleed when touched. Biopsy showed that a part of the uterine cervix was covered with cylindrical epithelium with papillary outgrowths; in the depth of tissue the growth of glands was present. What pathology of the uterine cervix was revealed?

A Pseudoerosion B True erosion C Endocervicitis D Glandular hyperplasia E Leukoplakia

82

A stillborn child was found to have thickened skin resembling of the tortoise shell, underdeveloped auricles. Histological examination of skin revealed hyperkeratosis, atrophy of the granular epidermis layer; inflammatory changes were not present. What is the most likely diagnosis?

A Ichthyosis B Leukoplakia C Xerodermia D Erythroplakia E Dermatomyositis

83

A pathology-histology laboratory received a vermiform appendix up to 2,0 cm thick. Its serous membrane was pale, thick and covered with yellowish-green films. The wall was flaccid, of grayish-red colour. The appendix lumen was dilated and filled with yellowish-green substance. Histological examination revealed that the appendix wall was infiltrated with neutrophils. Specify the appendix disease:

A Acute phlegmonous appendicitis B Acute gangrenous appendicitis C Acute superficial appendicitis D Acute simple appendicitis E Chronic appendicitis

84

A 46 year old patient who had been suffering from tuberculosis for 6 years died from massive pulmonary haemorrhage. Autopsy revealed different-sixed foci of sclerosis and caseous necrosis in lungs, in the upper part of the right lung there was a cavity 5 cm in diameter with dense grey walls, the cavity contained liquid blood and blood clots. What typeof tuberculosis is it?

Page 19: Pat an Anew

19

A Fibrocavernous B Acute cavernous C Infiltrative D Fibrous focal E Acute focal

85

A patient died from cardiopulmonary decompensation. Histological examination revealed diffused pulmonary lesion together with interstitial edema, infiltration of tissue by limphocytes, macrophages, plasmocytes; pulmonary fibrosis, panacinar emphysema. What disease corresponds with the described picture?

A Fibrosing alveolitis B Chronic bronchitis C Bronchopneumonia D Pulmonary atelectasis E Bronchial asthma

86

A 50 year old patient has been taking treatment thrice for the last 6 months because of fractures caused by domestic accidents. Microscopical examination of bony tissue revealedfoci of lacunar resolution, giant-cell granulomas in the tumour-like formations, cysts. Bony tissue was substituted by fibrous connective tissue. Examination revealed also adenoma of parathyroid gland and hypercalcemia. What is the most probable diagnosis?

A Parathyroid osteodystrophy B Myelomatosis C Osteomyelitis D Osteopetrosis E Paget's disease

87

2 hours after a skeletal extension was performed to a 27 year old patient with multiple traumas (closed injury of chest, closed fracture of right thigh) his condition abruptly became worse and the patient died from acute cardiopulmonary decompensation. Histological examination of pulmonary and cerebral vessels stained with Sudan III revealed orange drops occluding the vessel lumen. What complication of polytrauma was developed?

A Fat embolism B Gaseous embolism C Microbal embolism D Thromboembolism E Air embolism

88

A 50 year old patient underwent resection of tumour of large intestine wall. Microscopically it presents itself as fascicles of divergent collagen fibers of different thickness and form andsome monomorphous fusiform cells that are irregularly distributed among the fibers.

Cellular atypia is not evident. What tumour is it? A Hard fibroma B Fibromyoma C Soft fibroma D Desmoma E Fibrosarcoma

Page 20: Pat an Anew

20

89

Autopsy of a 5 year old child revealed in the area of vermis of cerebellum a soft greyish-pink node 2 cm in diameter with areas of haemorrhage. Histologically this tumour consisted of atypical monomorphous small roundish cells with big polymorphous nuclei. What tumour is it?

A Medulloblastoma B Meningioma C Glioblastoma D Astrocytoma E Oligodendroglioma

90

In course of severe respiratory viral infection there appeared clinical signs of progressing cardiac insufficiency that caused death of a patient in the 2nd week of disease. Autopsy revealed that the heart was sluggish, with significant cavity dilatation. Histological examination of myocardium revealed plephora of microvessels and diffuse infiltration of stroma by lymphocytes and histiocytes. What disease corresponds with the described picture?

A Myocarditis B Stenocardia C Acute coronary insufficiency D Myocardium infarction E Cardiomyopathy

91

A 38 year old patient with full-blown jaundice, small cutaneous hemorrhages, general weakness and loss of appetite underwent puncture biopsy of liver. Histological examination revealed disseminated dystrophy, hepatocyte necrosis, Councilman's bodies. Lobule periphery has signs of significant infiltration by lymphocytes, there are also individual multinuclear hepatocytes. What is the most probable diagnosis?

A Acute viral hepatitis B Acute alcoholic hepatitis C Miliary hepatic cirrhosis D Toxic degeneration of liver E Chronic hepatitis

92

A 20 year old patient died from intoxication 8 days after artificial illegal abortion performed in her 14-15th week of pregnancy. Autopsy of the corpse revealed yellowish colour of eye sclera and of skin, necrotic suppurative endometritis, multiple pulmonary abscesses, spleen hyperplasia with a big number of neutrophils in its sinuses. What complication after abortion was developed?

A Septicopyemia B Septicemia C Hemorrhagic shock D Chroniosepsis E Viral hepatitis type A

93

A section of the left lung was found to have an area of dense red tissue. The area was cone-shaped, stood out distinctly from the healthy tissue, with its base directed to the pleura. The dissected tissue was granular, dark-red. What is the most likely diagnosis?

Page 21: Pat an Anew

21

A Haemorrhagic infarction B Lung abscess C Lung gangrene D Primary tuberculous affection E Croupous pneumonia

94

A patient has a cluster of matted together dense lymph nodes on his neck. Histological examination of a removed lymph node revealed proliferation of reticular cells, presense of Reed-Sternberg cells. What disease is meant?

A Lymphogranulomatosis B Lymphoblastic leukosis C Myeloblastic leukosis D Myelocytic leukosis E Lymphocytic leukosis

95

A patient had been suffering from profuse diarrhea and vomiting for 2 days. He died from acute dehydration. Autopsy revealed that the intestinal wall was edematic and hyperemic, with multiple haemorrhages in the mucous membrane. Intestine lumen contains whitish fluidresembling of rice water. What disease caused death?

A Cholera B Dysentery C Salmonellosis D Typhoid fever E Enterocolitis

96

Examination of a 66 year old patient revealed a lytic tumour in the locus of pathological rib fracture. Histologically this tumour consists of atypical plasmoblasts. Further examination revealed osteoporosis in the bones of vertebral column and pelvis. These changes are typical for:

A Myelomatosis B Tuberculous osteomyelitis C Ewing's osteosarcoma D Neuroblastoma E Metastatic lung cancer

97

A patient died from acute cardiac insufficiency, among clinical presentations there was gastrointestinal haemorrhage. Examination of mucous membrane of sromach revealed some defects reaching myenteron; their edges and bottom were mostly even and loose, some of them contained dark-red blood. What pathological process was revealed?

A Acute ulcers B Chronic ulcers C Erosions D Thrombosis E Inflammation

98

A 33 year old man died from uraemia. Autopsy revealed enlarged kidneys weighing 500,0

Page 22: Pat an Anew

22

each and consisting of multiple cavities 0,5-2 cm in diameter. The cavities were full of light-yellow transparent liquid. Renal pelvis and ureters had no pecularities. What renal disease caused uraemia?

A Bilateral polycystic renal disease B Chronic pyelonephritis C Renal tumour D Renal tuberculosis E Rapidly progressing glomerulonephritis

99

A patient ill with tuberculosis died from progressing cardiopulmonary decompensation. Autopsy in the area of the right lung apex revealed a cavity 5 cm in diameter communicating with lumen of a segmental bronchus. On the inside cavity walls are covered with caseous masses with epithelioid and Langhans cells beneath them. What morphological form of tuberculosis is it?

A Acute cavernous tuberculosis B Tuberculoma C Caseous pneumonia D Infiltrative tuberculosis E Acute focal tuberculosis

100

A 30 year old woman has applied a lipstick with a fluorescent substance for a long time. Then she got a limited erythema and slight peeling on her lip border, later there appeared transversal striae and cracks. Special methods of microscopic examination of the affected area helped to reveal sensibilized lymphocytes and macrophages in the connective tissue; cytolysis. What type of immunological hypersensitivity was developed?

A IV type (cellular cytotoxicity) B I type (reaginic) C II type (antibody cytotoxicity) D III type (immune complex cytotoxicity) E Granulomatosis

101

Examination of a young woman revealed a node-like, soft and elastic homogenous tumour of pinkish-white colour along the acoustic nerve. The tumour contains cell bundles with ovalnuclei. Cellular fibrous bundles form rhythmic structures made up by parallel rows of

regularly oriented cells arranged in form of a palisade with cell-free homogenous zone (Verocay bodies) between them. What tumour is it?

A Neurinoma B Malignant neurinoma C Ganglioneurinoma D Neuroblastoma E Ganglioneuroblastoma

102

A 23 year old man has perforation of hard palate. In the area of this perforation there was a compact well-defined formation. Microscopic examination of the resected formation revealed a large focus of caseous necrosis surrounded by granulation tissue with endovasculitis, cellular infiltration composed of lymphocytes, epithelioid cells (mainly plasmocytes). What is the most probable diagnosis?

A Syphilis B Tuberculosis

Page 23: Pat an Anew

23

C Scleroma D Sarcoma E Leprosy

103

A 50 year old man who was referred to the hospital for treatment of cervical lymphadenitis underwent test for induvidual sensitivity to penicillin. 30 seconds after he went hot all over, AP dropped down to 0 mm Hg that led to cardiac arrest. Resuscitation was unsuccessful. Autopsy results: acute venous plethora of internal organs; histological examination of skin (from the site of injection) revealed degranulation of mast cells (tissue basophils). Degranulation was also revealed in myocardium and lungs. What type of hypersensitivity reaction is it?

A Anaphylactic B Delayed-type hypersensitivity C Complement-mediated cytotoxic D Immunocomplex-mediated E -

104

A 2 year old child had acute respiratory viral infection and died from cardiopulmonary decompensation. Autopsy revealed that his right lung was hyperemic; in the 2nd, 6th and 10th segments and on the incision there were airless yellowish foci of irregular form, from several mm up to 1 cm large. Microscopical examination revealed exudate consisting mainlyof neutrophils in the given areas of pulmonary tissue in the alveoles, bronchioles and

bronchial tubes. What is the most probable diagnosis? A Focal pneumonia B Interstitial pneumonia C Croupous pneumonia D Acute bronchitis E Pulmonary abscess

105

The upper lobe of the right lung is enlarged, grey and airless, the inscision surface is dripping with turbid liquid, the pleura has many fibrinogenous films; microscopical examination of alveoles revealed exudate containing neutrophils, desquamated alveolocytes and fibrin fibers. The bronchus wall is intact. What is the most probable diagnosis?

A Croupous pneumonia B Interstitial pneumonia C Pulmonary abscess D Focal pneumonia E Influenzal pneumonia

106

A 28 year old patient had high arterial pressure, hematuria and facial edemata. In spite of treatment renal insufficiency was progressing. 6 months later the patient died from uremia. Microscopic examination of his kidneys and their glomerules revealed proliferation of capsule nephrothelium and of podocytes with "demilune" formation, sclerosis and hyalinosis of glomerules. What disease corresponds with the described picture?

A Subacute glomerulonephritis B Acute pyelonephritis C Nephrotic syndrome D Chronic glomerulonephritis

Page 24: Pat an Anew

24

E Acute glomerulonephritis

107

Autopsy of a man ill with severe hypothyroidism revealed that connective tissue, organ stroma, adipose and cartilaginous tissues were swollen, semitransparent, mucus-like. Microscopic examination of tissues revealed stellate cells having processes with mucus between them. What type of dystrophy is it?

A Stromal-vascular carbohydrate B Stromal-vascular adipose C Stromal-vascular proteinaceous D Parenchymatous proteinaceous E Parenchymatous adipose

108

Examination of the anterior abdominal wall of a pregnant woman revealed a tumour-like formation that arose on the spot of a tumour that was removed two years ago. The neoplasm was well-defined, dense, 2х1 cm large. Histological examination revealed that thetumour was composed of differentiated connective tissue with prevailing collagen fibres.

What tumour might be suspected? A Desmoid B Lipoma C Fibrosarcoma D Hibernoma E Leiomyoma

Pathology of cell. Parenchymal dystrophy.1. During an autopsy a parenchymal fatty dystrophy of the myocardium was diagnosed. What is the common or descriptive name of the heart due to this dystrophy?A. *'Tabby cat' heart ('Tiger's' heart)B. Bovine heartC. 'Hairy' heartD. Solder plaque (bony heart)E. Cor pulmonale2. A patient with leukemia died from severe chronic anemia. An autopsy revealed an enlarged heart, with flabby myocardium It had a dim pale-grey color, yellow spots and bars. Which pathological process was found in the heart at post-mortem?A. * Parenchymal fatty dystrophyB. Vacuolar dystrophy.C. Hydropic dystrophy.D. Mesenchymal fatty dystrophy.E. Mixed dystrophy.3. A 53 year old patient died with symptoms of liver insufficiency. A post-mortem examination revealed the enlarged, flabby, yellow-brown liver. Gross examination of the liver's section showed drops of fat. Microscopically: hepatocytes on the peripheries of the hepatic lobules contained masses of small drops within the cytoplasm. Which process most likely took place in the liver?A. *Fatty dystrophy of the liverB. Glucosylceramide lipidosis (Gaucher's disease)C. Sphingolopidosis (Niemann-Pick disease)D. Gangliosidosis (Тау-Sachs disease)E. Systemic lipoidoses4. A patient died from chronic cardiovascular insufficiency. At the post-mortem a 'tabby cat' heart was found. From the side of the endocardium, a yellow-white striped pattern was noticeable. The myocardium was a dim with gray-yellow color. Which process is most likely diagnosed?A. * Fatty parenchymal dystrophy.

Page 25: Pat an Anew

25

B. Carbohydrate dystrophyC. Hydropic dystrophy.D. Fatty mesenchymal dystrophy.E. Amyloidosis.5. A seven-year-old child presented with diphtheria of the pharynx. He subsequently died from acute cardiac insufficiency. Post-mortem examination of the heart revealed that the cavities of the heart were extended horizontally. Muscle of the heart were dim and flabby. Gross section showed motley appearance, with yellow areas. Microscopically in the cytoplasm of some rnyocardial cells small vacu-oles were determined. The frozen sections showed vacuoles within cells stained with sudan-III in orange color. Which type of dystrophy was found in myocardial cells?A. *Fatty dystrophyB. Carbohydrate dystrophyC. Vacuolar dystrophyD. Hyaline dystrophyE. Hydropic dystrophy6. A man died due to cardiac insufficiency. At autopsy revealed the heart increased volume and flabby. A myocardium was a clay-yellow color and dim. From the side of the endocardium a yellow-white striped pattern was visible ('tabby cat'). Under the microscope the groups of myocardial cells lost their normal structure, their cytoplasm contained shallow drops which were black when stained with sudan-IV. Which one of the following is the correct diagnosis?A. *Fatty dystrophy of myocardiumB. CardiosclerosisC. Rheumatic myocarditisD. Obesity of the heartE. Myomalacia7. A 66-year-old male died from cardiac insufficiency. During the dissection an increase volume heart was found. Observation of the heart revealed a flabby consistency with stretched chambers. The myocardium section had a dim, clay-yellowish color. From the side of the endocardium a yellow-white striped pattern was present, which was especially noted in the papillary muscles. Which pathological process is the most credible?A. *Fatty dystrophy of the myocardiumB. Obesity of the heartC. Dilatation cardiomyopathyD. MyomalaciaE. Cardiosclerosis8. A patient died from pulmonary-cardiac insufficiency. During the dissection a significantly enlarged anemic liver, with yellow doughy consistencies was found. A liver specimen stained with hematoxylin and eosin exposed various sizes of vacu-oles in the cytoplasm of the hepatocytes. Which one of the following dystrophies occurred?A. *Parenchymal fattyB. Parenchyma! carbohydrateC. HyalineD. Mesenchymal fattyE. Hydropic9. A 38-year-old patient, suffering from chronic alcoholism and cirrhosis of the liver, developed profuse bleeding due to varicose veins of the esophagus which resulted in death. During the autopsy a liver was noted to be diminished in size with micronodular tuberosity. The organ was dense and rather yellow in color. A histological evaluation of the cryostat specimens of the liver, stained with hematoxylin and eosin, revealed hepatocytes with large, optically empty vacuoles. These vacuoles were black when stained with osmium acid. These optically empty vacuoles hepatocytes indicate:A. *Fatty dystrophyB. Inclusions of hyaline.C. Alcoholic hyaline (Mallory bodies)D. Vacuolar dystrophy.E. Carbohydrates dystrophy.10. A 16-year-old girl presents with the symptoms of sharp pain during swallowing, lymph node enlargement of the neck, and the body .temperature of 38°C. The mucous membrane of the tonsils revealed grayish membranes with yellow tapes with were not easily separated from the defect. The patient's state progressively worsened which death occurring on the 8th day of the disease due to cardiac insufficiency. Which of following histological changes in the myocardial cells will be the most likely finding?A. * Fatty dystrophyB. Hydropic dystrophyC. Hyaline dystrophyD. Ballooning dystrophyE. Mucous dystrophy

Page 26: Pat an Anew

26

11. A 44-year-old woman died from chronic alcoholic intoxication. During the autopsy a significantly enlarged liver of doughy consistency and rather yellowish color was found. Microscopically, after staining with hematoxylin and eosin, cytoplasm of the hepatocytes contained optically empty vacuoles. Which type of dystrophy has taken place?A. *Parenchymal fatty dystrophyB. Carbohydrate parenchymal dystrophyC. Hyaline dystrophyD. Mesenchymal fatty dystrophyE. Hydropic dystrophy12. A patient has died from toxic sepsis. During dissection a 'tabby cat' ('tiger's heart') was found. Microscopically, lipids were detected in the cytoplasm of the myocardial cells. What is the primary morphological mechanism for development for this dystrophy?A. *DecompositionB. InfiltrationC. TransformationD. Pathological synthesis.E. Neoplastic alterations.13. Ultrastructural investigation of a liver biopsy, revealed that between the mitochondria there were numerous flat cisterns and bubbles with secretory granules circumscribed with membrane. Name a cell structure with the hyperplasic constituents?A. * Golgi apparatusB. Pinocytosis bubblesC. Endoplasmic reticulumD. LysosomsE. Microtubes14. A 42-year-old female became ill with diphtheria and died from acute cardiac insufficiency. During dissection it was noted that the heart cavities were extended and that the muscle of the heart was dim, motley and on a cut surface had yellow areas. Which process was exposed in the myocardial cells?A. *Fatty dystrophyB. Carbohydrate dystrophyC. Ballooning dystrophyD. Hyaline dystrophyE. Hydropic dystrophy15. During the examination of a newborn, some apparent skin differences are noted. The skin is dry, with an uneven surface and with the presence of grey plates which can be removed layer by layer. These changes are related to which type of dystrophy?A. *Homy dystrophyB. Hydropic dystrophyC. Hyaline dystrophyD. Fibrinoid swellingE. Mucoid swelling16. A male patient had a prosthetic appliance on the lower jaw. The ventral surface of tongue revealed a dense, gray plaque with a clear boundary. Histology revealed the thickened of the stratified squamous epithelium due granular and to basal layers thickening, hyperkeratosis, acanthosis, lymphocyte's infiltration of connective tissue. Make a diagnosis.A. *LeukoplakiaB. ErythroplakiaC. PapillomaD. Cancer in sity (infra-epithelial neoplasia)E. Condyloma17. During the preventive examination of a worker employed in the coal resins production the areas of thickening and keratinization of the mucous membrane in the oral cavity were found. This occured mainly on the cheeks areas, showing a whitish color with a rough surface. They were not painful. Which pathology is this related to?A. * LeukoplakiaB. PapillomatosisC. GlossitisD. StomatitisE. Calcification18. A 45-year old male is found to have a severe intoxication. A diagnosis of sepsis is made. Several days later he dies. At autopsy, his myocardium grossly had a 'tiger heart' pattern. . Microscopically, lipids were detected in the cytoplasm of cardiac hystiocytes. What morphogenetic mechanism prevails in the development of this dystrophy?A. * Decomposition..B. Infiltration.

Page 27: Pat an Anew

27

C. TransformationD. Abnonnal synthesis.E. Colliquation.19. A 3-month- old infant dies and autopsy is requested. Electron microscopic examination of liver tissue revealed a great amount of flat cisterns and vesicles with secretory granules, surrounded by membrane, scattered among numerous mitochondria. Which of the following cell ultra structures has been shown to be hyper-plastic?A. *Golgi complex.B. Pinocytic vesicles.C. Endoplasmic reticulum.D. Lysosomes.E. Filaments.20. A 36-year-old female develops liver failure followed with lethal outcome. Autopsy has shown an enlarged liver of yellow-brown color and soft consistence. Drops of fat are noticed on the liver cut surface and on the scalpel. Microscopically: hepatocytes at peripheral zone of a liver lobules contain small drops that fill cytoplasm and push the nucleus to the periphery. What process in the liver do the following changes testify to?A. *Fatty degeneration of liver.B. Cerebrosidelipidosis (Gaucher's disease).C. Sphingomyelinlipidosis (Niemann — Pick disease).D. Gangliosidelipidosis (Тау —Sachs disease).E. Generalized gangliosidosis (Norman—Landing disease).21. Autopsy of a menopausal woman with a ling history of a chronic ischemic heart disease revealed soft and enlarged heart. Its chambers were extended; the myocardium sectional view was lack-luster with grey- yellowish coloring. An endocardium presented with yellow-white banding, most evident in papillary muscles. What is the most likely pathological process in woman's heart?A. *Fatty degeneration of myocardium.B. Fatty heart.C. Dilated cardiomyopathy.D. Myomalation.E. Cardiosclerosis.22. A 77-year-old male with a dental prosthesis on his upper jaw is seen by his dentist because of a solid gray patch on his tongue. A lesion has irregular contour, uneven surface, and clear borders. Microscopic investigation of its biopsy revealed the thickening of stratified squamous epithelium, its hyperkeratosis, and acanthosis accompanied with lymphocytes and macrophages infiltration of subjacent connective tissue. What is the most likely diagnosis?A. *Leukoplakia.B. Erythroplakia.C. Papilloma.D. Cancer in situ.E. Condyloma.Connective tissue's (mesenchymal) dystrophy1. A 56 year old female has been ill with chronic fibrocavernous tuberculosis of the lungs for the past 20 years. She entered the nephrology department with an uremia syndrome. A test for the presence of amyloid in kidneys was positive. Which form of amyloid is indicated in this case?A. * SecondaryB. PrimaryC. LocalizedD. Familial congenitalE. Senile2. The dissection of a 49 year old male reveals a deformed mitral valve, which is thickened and does not completely close. Microscopically the foci of the collagen fibers are eosinophilic and give a positive reaction on a fibrin test? The most credible explanation is:A. *Fibrinoid swellingB. Fibrinoid inflammationC. Mucoid swellingD. HyalinosisE. Amyloidosis3. A 56 year old patient with a six year history of peritonitis has died. During dissection the capsule of the liver and the spleen was markedly thickened in places and was noted as being dense and semi-lucent. The most credible explanation for this is:A. *HyalinosisB. NecrosisC. Mucoid swellingD. Fibrinoid swellingE. Amyloidosis

Page 28: Pat an Anew

28

4. The dissection of a 48 year old patient who suffered with rheumatoid arthritis reveals an enlarged, dense spleen. A spleen's section demonstrates its brown-reddish color with enlarged follicles which have the appearance of semi-lucent, grayish-white corns. What is the name of these lesions in the spleen?A. * Sago-like spleen.B. Glazed spleen.C. Sebaceous spleen.D. Hyalinosis of spleen.E. Porphyry spleen.5. During the post-mortem performed on a 72 year old man there are noted some diminished areas of the spleen with a pinkish color. Microscopic examination revealed that the follicles are diminished in volume and the walls of the arterioles and trabeculas are thickened as well as containing homogeneous eosinophilic, PAS-positive masses. Staining with picrofuksin dye reveals the masses to be a red color. These changes indicate the presence of:A. *HyalinosisB. AmyloidosisC. Mucoid swellingD. Fibrinoid swellingE. Sclerosis6. A 52 year old male died from a heart attack. At the time of dissection a symmetric type of severe obesity discovered. The rapture of the right ventriculum wall resulted in hemopericardium. Under epicardium an excessive fat tissue formation discovered. A microscopy of the sample showed the excecive growth of fatty tissue accompanied with atrophy of myocardial fibers. Which pathological process is most likely responsible for the patient's death?A. * Simple obesity of the heart.B. Fatty dystrophy of myocardium.C. Ischemic heart disease.D. HypertensionE. Acute myocardium infarct7. During dissection of a 65 year old patient, who suffered from a fibrous-cavernous tuberculosis, an enlarged, dense spleen was found. Spleen section grossly had brown-pinkish color, smooth, waxy-like surface. Which pathological process listed below is the most credible?A. * Sebaceous spleen.B. Glazed spleen.C. Porphyry spleen.D. Sago spleen.E. Cyanotic induration8. A post mortem performed on a 50 year old male who died of a heart attack indicated a symmetric type of obesity of the III degree with rupture of the walls of the right ventricle and hemopericardium. Under the epicardium surplus deposits of fat were found. Microscopically, fatty tissue from the epicardium was dispersed in the myocardium with an atrophy of the muscle fibers. Which process listed below is the most reliable?A. *Obesity the heart.8. Fatty dystrophy of myocardium.C. Acute infarct of myocardium.D. Ischemic heart disease.E. Hypertension.9. An autopsy of a 45-year-old female revealed the kidneys were dense, yellow in color and appear to have a greasy brilliance. Which pathological process is most likely?A. *AmyloidosisB. HyalinosisC. Fatty dystrophyD. Mucoid swellingE. Hemochromatosis10. Macroscopic examination of a stomach delivered from surgery, revealed a round lesion 1.5cm in diameter which extended by the muscle layer at the antral zone of a small curvature. A semilucent dense area on the bottom of the defect was also determined. It resembled hyaline cartilage. Which process developed in the bottom of the stomach lesion?A. *Localized hyalinosisB. AmyloidosisC. Mucoid swellingD. Fibrinoid changesE. Generalized hyalinosis11. A skin biopsy of a patient with allergic vasculitis was submitted for examination. It is discovered that the vessel walls were thickened and homogeneous. Picro-fuxin stained a tissues a yellow color. They were Shiff-positive. Which pathological process developed in the walls of the vessels?

Page 29: Pat an Anew

29

A. *Fibrinoid swellingB. AmyloidosisC. Mucoid swellingD. HyalinosisE. Lipidosis12. The post-mortem of a patient revealed feature of chronic kidney insufficiency. Grossly, kidneys were enlarged, dense, wax-like, with foci of irregular depressed scars on their surface. Microscopically, the mesangeal areas were expanded and the glomerular capillaries obstructed by Congo red stain-positive amorphous aceilular material. In some sections the deposits took on nodular appearance. Which of the following diagnoses is most reliable?A. *Amyloidosis of the kidneys (Amyloid nephropathy)B. Acute glomerulonephritisC. Chronic glomerulonephritisD. Subacute glomerulonephritisE. Lipoid nephrosis13. At autopsy a 76-year-old male, with a history of peritonitis 10 years ago, is found to have thickened and dense both liver and spleen capsules. They were translucent on a sectional view. What is the most likely pathology of the described organs capsules?A. *Hyalinosis.B. Necrosis.C. Mucoid swelling.D. Fibrinoid swelling.E. Amyloidosis.14. A 55-year-old female, with a long history of rheumatoid arthritis, develops renal failure and dies. An autopsy revealed an enlarged solid spleen. On the sectional view, its tissue had brown-reddish coloring with enlarged follicles that look like translucent grayish-white grains. What is the most likely pathological process?A. *Sago spleen.B. Frosted spleen.C. Lardaceous spleen.D. Spleen hyalinosis.E. Porphyry spleen.15. A 66-year-old female, with a long history of post-traumatic osteomyelitis, is admitted to the hospital for treatment of nephrotic syndrome. On the night pf admission she suddenly dies. Autopsy revealed dense, white kidneys with scars in the cortical layer; they had a sebaceous glow on the cut surface. What is the most likely kidneys pathology?A. *Secondary amyloidosis.B. Primary amyloidosis.C. Idiopathic amyloidosis.D. Chronic glomerulonephritis.E. Chronic pyelonephritis.16. A 55-year-old man, with a long history of a symmetrical type of severe obesity, developed acute heart insufficiency followed with lethal outcome. An autopsy revealed right ventricle wall burst with hemopericardium and excessive amount of fatty tissue under epicardium. Microscopically: adipose tissue from epicardium penetrates myocardium with muscle fibers atrophy. Name the pathological process?A. *Simple fatty heart.B. Fatty degeneration of myocardium.C. Ischemic disease.D. Essential hypertension.E. Acute myocardial infarction.

Mixed dystrophy1. The necrotic Peyer's patches of the ileum from the patient with typhoid fever are stained in a yellow-brown color. Which pigment impregnates the necrotic tissue? A. * BilirubinB. HemoglobinC. LipofuscinD. IndolE. Melanin2. During post-mortem of a patient arrived from a tropical country, it is discovered that there is a hemomelanosis of a liver, spleen and elements of the reticuloendo-thelial stroma. These changes are characteristic for which disease?A. *MalariaB. DysenteryC. Diabetes mellitusD. Exanthematic typhus

Page 30: Pat an Anew

30

E. Grippe3. A patient who suffered from cancer of the stomach died from cachexia. During the post-mortem the characteristic alteration in the heart were found. How would this condition be termed in the heart?A. *'Brown' atrophyB. 'Hairy' heartC. Solder plaque (bony heart)D. Tiger's heart ('tabby cat')E. Bovine heart4. A man with insufficiency of the mitral valve complained of a cough and sputum with a brownish colouring. Which pigment results in this color of the sputum?A. *HemosiderinB. MelaninC. HemoglobinD. HemomelariinE. Iron sulfate5. The post-mortem of a man who presented in the hospital with a history of a snakebite reveals expressed intravessels hemolysis. During dissection it is noted that the spleen, bone marrow and lymphatic nodes had a brown colouring. Microscopic examination showed that the cytoplasm of macrophages got a brown pigment. Which pigment accumulated in the tissues?A. * HemosiderinB. HematoidinC. HematinD. LipofuscinE. Bilirubin.6. The dissection of a patient who suffered from rheumatism and chronic rheumatic valvulitis revealed that mitral valve leaflet was thickened with rough stony deposits. Name the pathology presented with stony appearance of the valves?A. * Dystrophy calcificationB. Metastatic calcificationC. Metabolic calcification.D. FibrinoidE. Amyloidosis.7. An endoscopy was performed on a patient with a chronic stomach ulcer complicated with hemorrhage. This procedure revealed a brownish (coffee-like) liquid in the stomach. Which pigment results in the color of the stomach contents?A. * Hematin hydrochlorideB. HemosiderinC. BilirubinD. FerritinE. Porphyrin8. A 46-year-old man has an acute stomach ulcer complicated by gastric bleeding and vomiting. Gastric masses had a brown color and "coffee-like" appearance. Which pigment created such colouring?A. * Hematin hydrochlorideB. HemoglobinC. BilirubinD. HemomelaninE. Iron sulfide9. A 66-year-old patient complained of pain in the hands and feet joints. Physical examination revealed a deformation and painful of the joints. Laboratory tests showed the increased level of uric acid salts in the blood and urine. Which one of following is not being fully metabolized?A. *Nucleoprotein.B. Calcium.C. Chrornoprotein.D. Lipoprotein.E. Potassium.10. A man died from chronic sepsis. A post-mortem revealed an atrophy of the skeletal muscles and brown atrophy of both myocardium and liver. Which one of the following pigments accumulated in tissues?A. *LipofuscinB. LipochromeC. HemosiderinD. HemomelaninE. Melanin

Page 31: Pat an Anew

31

11. A 58-year-old male has been ill for many years with leukemia. A post-mortem exposed a brown color in the marrow, spleen, liver, and lymphatic nodes. The Perls' histochemical reaction was conducted. It was determined that the reticular, the endothelial cells and histiocytes of these organs contained granules of a dark blue color. Which pigment is responsible for the colouring?A. * HemosiderinB. BilirubinC. HematoidinD. HemomelaninE. Hematoporphyrin12. The post-mortem of a patient who suffered from malaria revealed jaundiced skin, sclera and mucous membranes. Also, the spleen was enlarged and had dark-grey color. This colour of the spleen is due to the presence of:A. *HemomelaninB. HemosiderinC. LipofuscinD. MelaninE. Hemoporphyrin13. A 56 year old patient died from chronic cardiac insufficiency as a result of rheumatic heart-disease. A post-mortem revealed that lungs were enlarged, dense with red-brownish coloring. What is the most likely diagnosis?A. *Brown induration lungsB. Acute bronchitisC. Honey-comb lungsD. Chronic bronchitisE. Chronic emphysema14. A patient with mitral valve insufficiency presents in his sputum cells, filled with brown pigment. The Perls' reaction is positive. Name this pigment.A. *HemosiderinB. HematoidinC. MelaninD. PorphyrinE. Bilirubin15. A 66-year-old male, with a history of hernatogenic tuberculosis was examined. This revealed hyperpigmentation of skin and mucous membranes, cachexia and insufficiency of the cardio-vascular system. Which disease caused such changes?A. *Addison's diseaseB. PhaeochromocytomaC. Simraond's diseaseD. Gushing diseaseE. Greves' disease16. A 52-year-old male with a history of sub-acute septic endocarditis is examined by a physician. A doctor revealed marked general pallor with icteric skin, sclera and visible mucous membranes. Blood test showed accumulation of indirect reacting bilirubin (unconjugated bilirubin). The yellow staining of the skin, sclera and mucous membranes indicates which one of the following?A. *Prehepatic jaundiceB. Fatty dystrophyC. HemosiderosisD. Hepatic jaundiceE. Posthepatic jaundice17. A 62-year-old female with a history of stomach cancer with plural metastases died from a cachexia. Select the characteristic changes of the heart expected to be revealed on dissection.A. * Brown atrophy of myocardium.B. Amyloid cardiomegaly.C. Dilatation cardiomyopathy.D. Hypertrophy cardiomyopathy.E. "Tabby cat" ("Tiger's heart").18. A patient developed a cyst in the cerebrum following a hemorrhagic stroke. Two years later the patient died from pneumonia due to a complication of influenza. During examination of the brain cyst it is noted that the walls have a rusty tint. Perls' reaction is positive. Name the process occurring in the wall of the cyst?A. * Localized hemosiderosisB. General hemosiderosisC. Local hemomelanosisD. Infiltration of bilirubinE. Primary haemochromatosis

Page 32: Pat an Anew

32

19. A patient with a long history of tuberculosis was examined at the hospital. Physical examination revealed a grayish-brown skin color, lowered arterial pressure, hypodynamia and a decline of the level of 17-oxycorticosteroids in the urine and blood plasma. A problem with the metabolism of which pigment is indicated by the clinical signs of this patient?A. *MelaninB. BilirubinC. LipofuscinD. LipochromeE. Hemosiderin20. A fragment of skin (1x2 centimeters) delivered for histological research. Grossly a small (0,5 cm in diameter) slightly elevated brown lesion, sharply demarcated from the surrounding normal skin, was recognized. Microscopically, a lesion presented with nevus cells nests, rich with brown pigment. This pigment had negative Perls' reaction. Name the pigment.A. *MelaninB. HematoidinC. HemosiderinD. BilirubinE. Hemomelanin21. A 5 5-year-old male with a history of bronchiectasis, pneuinosclerosis and cachexia died. During the post-mortem examination the heart was found to be diminished in size, flabby, with thinned walls. A section revealed brownish color of the heart's tissue. Which pigment was indicated in the myocardium?A. * LipofuscinB. HemosiderinC. HematoidinD. MelaninE. Lipochrome22. A post-mortem of a 44-year-old patient with a history of mitral stenosis reveals dense lungs that are a brown color. Which pathological process is most likely in the lungs?A. *HemosiderosisB. HemochromatosisC. IcterusD. HemomelanosisE. Lipofuscinosis23. A post-mortem was performed on a 55-year-old male, who over last eight years suffered from chronic form of malaria. At the dissection both grey matter of the cerebrum and a spleen had the ash-grey color. Which pigment is responsible for this discoloration?A. *HemomelaninB. LipofuscinC. HematoporphyrinD. MelaninE. Hemosiderin24. A 62-year-old male who has been ill with diabetes mellitus for 15 years died from a cerebral hemorrhage. Post-mortem revealed kidneys diminished in size with a fine-grained surface. The epithelium of the canaliculi of distal nephron's segment was high, with a light foamy cytoplasm. The Best's carmine staining demonstrated a bright red coloring of the cytoplasm's accumulations. These changes in the epithelium resulted from the accumulation of:A. *GlycogenB. LipidsC. HyalineD. ProteinsE. Amyloid25. At autopsy 68-year-old male is found to have cancer of the esophagus, accompanied with cachexia. Grossly, fatty tissue disappeared, both a liver and a heart were atrophic. Microscopy revealed brown-yellowish corn-like deposited next to nuclei of myocardial cells. These accumulations had negative Perls' reaction. Name the material of accumulations.A. *LipofuscinB. MelaninC. HemosiderinD. FerritinE. Hemomelanin26. A 55-year-old patient with a bilateral adrenal glands lesions presented with dark brown colouring of the skin. During histochemical examination of the skin the Perls' reaction was negative. Which pigment is responsible for this discoloration of the skin?A. *Melanin

Page 33: Pat an Anew

33

B. HemosiderinC. PorphyrinD. LipofuscinE. Biliverdin27. A 58-year-old female with a long history of chronic dysentery died. At autopsy, the stroma and parenchyma of the myocardium, kidneys, the mucous membrane of the stomach, and the connective tissue of lungs revealed violet color amorphous masses, which had positive K.QSS' reaction. Which pathological process developed in the patient's organs?A. *Metastatic calcificationB. Dystrophy calcificationC. Metabolic calcificationD. AmyloidosisE. Hyalinosis28. A 45-year-old male, with a long histoiy of rheumatism and mitral valve insufficiency, develops a chronic cough with rusty expectoration. What pigment colored sputum?A. *Hemosiderin.B. Melanin.С Hemoglobin.D. Malarial pigment.E. Iron sulfide.29. A 67-year-old male, with a long history of mitral valve's insufficiency, has been experiencing a cough with red-brownish coloring of a sputum. Cells with brown pigmentation and positive Perls' test were detected in the sputum. Which pigment responsible for the septum coloring?A. *Hemosiderin.B. Hematoidin.C. Melanin.D. Porphyrin.E. Bilirubin.30. A 38-year-old female with chronic stomach ulcer complicated with bleeding examined endoscopically. Stomach masses had coffee ground coloring. Which pigment responsible for this coloring?A. *Hematin chloride. В Hemosiderin.C. Bilirubin.D. Ferritin.E. Porphyrin.31. A 47-year-old male, with a history of secondary syphilis, has noticed foci of the skin depigmentation at his neck. Name the pathological process of the skin.A. *Leukoderma.B. Metaplasia.C. Leukoplakia.D. Dysplasia.E. Parakeratosis.32. a 38-year old female is found to have bilateral adrenal adenopathy resulted in brownish coloring of her skin. Perls' test of her skin's biopsy was negative. What pigment altered the color of the skin?A. *Melanin.B. Hemosiderin.C. Porphyrin.D. Lipofuscin.E. Biliverdin.33. A 70-year-old man has noticed recent swelling of joints of his hands and feet. Joints are painful and stiff. Laboratory tests revealed an increased level of urates in blood and urine. What is the most likely substance caused described pathology?A. *Nucleoproteins.B. Calcium.C. Chrornoproteids.D. Lipoproteins.E. Potassium.34. A 70 year-old male with a history of chronic shigellosis died. At post-mortem tissue samples were collected for histopoly. Microscopic investigation of hematoxylyn and eosin slides revealed amorphous violet deposits in stroma of the heart, kidneys, lungs and stomach mucosa. Koss' reaction was positive. What is the most likely pathological developed?A. *Metastatic calcification.B. Dystrophic calcification.C. Metabolic calcification.

Page 34: Pat an Anew

34

D. Amyloidosis.E. Hyalinosis.Necrosis. Postmortem changes.1. A male had a surgery due to "acute abdomen". During the operation it was noted that the peritoneum was dull and in the lumen of the rnesenteric artery superior a thrombus was detected. About 80 centimeters of the ileac intestine had a black colouring. Which process was diagnosed in the intestine?A. *GangreneB. Decubitus ulcerC. White infarctD. White infarct with a hemorrhagic crownE. Coagulative necrosis2. A patient with diabetes rnellitus presents to his physician with an acute pain in the right foot. At inspection the toe of foot had a black colour, the tissues of foot were edematous, with bed smell. Which form of necrosis was likely to be diagnosed?A. * Moist (wet) gangreneB. Decubitus ulcerC. SequesterD. Dry gangreneE. Infarct3. A 63-year-old male died of an endemic typhus. During the post-mortem it was revealed that the muscles of the abdominal wail and legs were dense with whitish-yellowish colouring. They resemble a candle. Name the pathological process?A. *Waxy necrosis (Zenker's necrosis)B. ApoptosisC. Fibrinoid necrosisD. Colliquative necrosisE. Caseous necrosis4. A 72-year-old-male had an infarct of the dextral hemisphere of the brain. One year later a computer tomography of the right hemisphere of the brain reveals a cavity with smooth walls and filled with liquid. Which pathological process is he most likely to have?A. * Post-infarct cystB. HydrocephalusC. Grey softening of a brainD. Infarct of a brainE. Hematoma5. A post-mortem revealed a thrombus in the left artery mesencephalicae and a large locus of grey softening in the tissues of the left hemisphere of a brain. Which pathological process is most likely to be present in the brain?A. * Ischemic infarctB. Coagulative necrosisC. AbscessD. Moist gangreneE. Sequestrum6. At a post-mortem of the 46-year-old male a large yellow - grey lesion was found in the left ventricle of the heart. A fresh thrombus was found in the coronary artery. What disease is he most likely to have?A. *Infarct of the myocardiumB. CardiosclerosisC. MyocarditisD. AmyloidosisE. Cardiomyopathy7. At autopsy a 60-year-oid male is found to have ischemic heart disease and atherosclerosis of the coronary arteries of heart. A section of the myocardium showed a white-yellowish focus, surrounded by the zone of hemorrhages in the apex, anterior and lateral walls areas of a left ventricle. Which is the most likely diagnosis?A. *Infarct of the myocardiumB. Post-infarction cardiosclerosisC. Diffuse cardiosclerosisD. MyocarditisE. Fatty dystrophy of the myocardium8. After a long staying in the bed a patient with circulatory deficiency got the skin and soft tissue darening above the sacrums. These tissues became swallowed. Sloughing off the epidermis in this area resulted in ulceration. Which complication is most likely?A. *Decubitus ulcerB. Dry gangreneC. Phlegmon

Page 35: Pat an Anew

35

D. InfarctE. Abscess9. Histological investigation of liver's biopsy revealed that some cells burn on small pieces with separate organellas and nuclei fragments surrounded by a membrane. The inflammatory reaction was missing. Select pathological process, the described changes are characteristic for:A. *ApoptosisB. NecrosisC. KaryorrhexisD. PlasmolysisE. Plasmorrhexis10. A male who had a long history of the intermittent claudication demonstrates the tissue of the foot fingers as being dry with a black colour, resembling a mummy. On small interval from this place the dichromatic line (red colour, is next to unchanged tissues, and white - yellow colour close to a tered tissues). Which type of a necrosis occurred?A. *Gangrene.B. Infarct.C. Sequester.D. Decubitus ulcer.E. Maceration.11. A 62-year-old male got a surgery due to the inguinal hernia. Macroscopic examination reveals that the wall of the intestine was a cyanotic, inflated, swallowed and coated with threads of a fibrin. Peristalsis was not heard. Which pathological process occurred in the wall of the intestine?A. * Moist gangrene.B. Dry gangrene.C. Coagulate necrosisD. Colliquative necrosisE. Decubitus ulcer12. A postmortem of a man, who died from typhoid revealed muscles on the abdominal wall and legs were dense, fragile, whitish-yellowish colour, resembling a candle. Which term best characterizes the muscles changes?A. *Zenker's necrosisB. Fibrinoid necrosisC. Caseous necrosisD. Colliquative necrosisE. Apoptosis13. A 48-year-old male, who had a history of hypertension for 12 years, present acute disturbance of the cerebral circulation. He developed a headache and alteration of the motion in the right extremity. Following right-handed hemiplegia resulted in fatal outcome. A postmortem revealed a systemic hyalinosis of the small arteries, thrombosis in the left arteria cerebri media. In the left parietal-temporal area a lesion was found, which is called:A. *Ischemic infarctB. HemorrhageC. Abscess of a brainD. Hemorrhagic infarctE. Edema of the brain14. A 45-year-old male suddenly died with the following findings revealed during the postmortem. In the back wall of the left ventricle of the heart a myocardial infarction was found. Which of the following microscopic changes in the myocardio-cytes can be seen in the locus of an infarct?A. *KaryolysisB. Fatty dystrophyC. Carbohydrate dystrophyD. CalcificationE. Protein dystrophy15. A postmortem of on a previously ill 48-year-old patient found an obturation of the lumen of the middle cerebral artery due to a thrombus. In the parietal-temporal area of the left hemisphere of the brain a locus of grey colour tissue with soft texture is found. Which tern best characterizes the brain tissue changes?A. *InfarctB. SequesterC. GangreneD. Caseous necrosisE. Fibrinoid necrosis16. A postmortem on the upper lobe of the right lung reveals the large triangle-like locus of the dark red dense tissue. Histological examination indicates necrosis of the walls of the alveolus's and the lumens filled with erythrocytes. Which is the most likely associated finding?A. *Hemorrhagic infarct

Page 36: Pat an Anew

36

B. CarnificationC. Lung's gangreneD. HemorrhageE. Atelectasis17. A postmortem on an elderly male with atherosclerosis reveals a thrombus in a branch of the internal carotid artery as well as a grey locus of a moist softening of the brain's tissue. Which pathological process was found in the brain?A. *Ischemic infarctB. Hemorrhagic infiltrationC. HematomaD. EncephalitisE. Tumor of a brain18. A surgery on a patient, with a history of syphilis revealed a locus of flabby tissue. Grossly, this locus was yellowish, dry, structures and gummy. The most likely diagnoses is:A. *Caseous necrosisB. InfarctC. Waxy necrosisD. Fibrinoid a necrosisE. Steatonecrosis19. The investigation of the liver's biopsy revealed that some separately arranged cells burn on small-sized pieces surrounded by a membrane. In some of them there were organelles, other had the fragments of dissolved nuclei. The inflammatory reaction around these cells missed. Name these changes:A. *ApoptosisB. AtrophyC. NecrosisD. HypoplasiaE. Dystrophy20. An ill elderly patient with a atherosclerosis, develops pain in the left foot. Grossly was found the foot enlargement, its tissues were black, flabby and macerated. The demarcation zone was not expressed. Which term best characterized the foot tissues changes?A. * Moist (wet) gangrene.B. Mummification.C. Coagulate necrosis.D. Dry gangrene.E. Sequestrum.21. A 62-year-old female with atherosclerosis was admitted to the hospitalized. At surgery gross examination revealed purulent peritonitis. During the operation a thrombus in the mesenteric arterias was found. Which was the most likely cause of the peritonitis?A. *Hemorrhagic infarct.B. Angiospastic ischemiaC. Ischemic infarctD. StasisE. Compressive ischemia22. The examination of a child, who had a history of measles, revealed reddish-black, uneven, swollen, slightly fluctuated lesions of cheaks and perineum area. Name the complication of measles?A. * Moist gangrene (noma)B. Dry gangreneC. Gas gangreneD. Decubitus ulcerE. Trophic ulcer23. A postmortem was performed on a female who died due to the cystadenocarci-noma metastases. The postmortem revealed large segments of a necrosis of the skin and soft tissues within cubitus area. Name the form of the necrosis.A. *Decubitus ulcerB. InfarctC. SequesterD. Caseous necrosisE. Zenker's necrosis24. A 58-year-old female with the history of atherosclerosis dies suddenly due to acute heart failure. Gross inspection of the left ventricle of the heart revealed a whitish-yellowish 6x5 cm, dense lesion with uneven boundaries and hemorrhagic zone next to it. Which is most likely diagnose?A. *Infarct of the myocardiumB. Postinfarction fibrosis

Page 37: Pat an Anew

37

C. Healed infarctD. MyocarditisE. Ischemic cardiomyopathy25. An ultrastructural examination of a salivary gland revealed within cells pieces of the nuclei surrounded by a membrane. Also condensate fragments of nuclear material and separate organelles were found. An inflammatory reaction around these cells was missing. Which term most correctly defines these alteration?A. *ApoptosisB. KaryorrhexisC. Coagulation necrosisD. KaryopyknosisE. Karyolysis26. A patient with tuberculosis has a kidney biopsy performed. Histological investigation revealed the caseous necrosis of the tissue accompanied by disorderly arranged fine grains of a chromatin. Which term most correctly defines describe lesion?A. * KaryorrhexisB. KaryolysisC. KaryopyknosisD. Mitotic activity of nucleiE. Apoptosis27. A postmortem of a 48-year-old male who had a history of typhoid fever revealed that the rectus abdominis at the wall was dense, a whitish colour, and resembled a candle. Which is the most likely diagnosis?A. * Waxy necrosisB. Fibrinoid necrosisC. Colliquative necrosisD. Caseous necrosisE. Apoptosis28. A 44 year old ill patient died due to the severe chronic heart failure. Pathologist diagnosed rheumatic granulomatous myocarditis. Microscopic evaluation of the myocardium indicated the presence of granulomas which consisted of macrophages with hyperchromatic nuclei and clear cytoplasm. Also necrosis was seen in the center of a lesion. Which is the most likely type of necrosis in the center of the lesion?A. * Fibrinoid necrosis.B. Waxy necrosis.C. Caseous necrosis.D. Colliquative necrosis.E. Fatty dystrophy.29. A 57-year-old patient has a long history of the type II diabetes mellitus. Physical examination revealed the alteration of the right foot tissues. They are dense, black with precise boundaries from normal tissues. Which term most correctly describe the lesion?A. *Dry gangrene.B. Wet (moist) gangrene.C. Gas gangrene.D. Decubitus.E. Trophic ulcer.30. A postmortem examination of a dead body revealed a cloudy corneas, dry skin with yellowish - brown lesion. Which term most correctly identifies describes a post-mortem alterations?A. *Cadaver desiccationB. Clotting of bloodC. Livor mortisD. Rigor mortisE. Algor mortis31. The postmortem of a 48-year-old male reveals in the right temporal lobe of the brain a large grey lesion with a softening, porridge-like texture. The basal arteries of the brain had numerous white - yellow thickenings of an intima which significantly decreased lumen. Which is the most likely diagnosis?A. *Ischemic infarctB. Abscess of a brainC. HematomaD. Hemorrhagic infarctE. Edema of the brain32. At post-mortem, a 60-years-old man, with a history of-typhoid fever, is found to have rectus muscles of the anterior abdominal wall dense, whitish, and look like a stearic candle. What is the most likely diagnose?A. *Zenker's necrosis.B. Fibrinoid necrosis.C. Colliquative necrosis.

Page 38: Pat an Anew

38

D. Caseous necrosis.E. Apoptosis.33. A 65-year-old female, with a long history of diabetes mellitus, presented her black, edematous and painful thumb of the right foot. Gross inspection revealed a focal epidermal detachment and malodorous discharge. What is the most likely clinico-pathologic form of necrosis?A. *Moist (wet) gangrene.B. Decubitus ulcer.C. Sequester.D. Dry gangrene.E. Infarction.34. A 5-year-old boy with measles presents to his pediatrician with necrotic changes of his cheeks. Gross inspection revealed that the cheeks soft tissues were edematous with reddish black fluctuated indistinctly outlined foci. What is the most likely complication of a measles?A. *Moist gangrene.B. Dry gangrene. С Gas gangrene.D. Decubitus ulcer.E. Trophic ulcer.35.A physical examination of 67-year old lady, with a history of femoral bone fracture, revealed a sequester formation accompanied with chronic inflammation of a bone marrow and adjacent tissues. What is the most likely disease caused such lesions?A. *Osteomyelitis.B. Reticulosarcoma.C. Multiple myeloma.D. Osteoclastoma.E. Periostitis.36, An elderly woman with a history of a stroke one year ago complains of a left limbs immobility. A computer tomography examination revealed a cavity filled with liquor, at right hemisphere of her brain. What is the most likely diagnose?A. *Postinfarction cyst.B. Hydrocephaly.C. Grey encephalomalacia.D. Cerebral infarction.E. Hematoma.Blood and lymph circulation disorders.1. The postmortem of a 48-year-old male, with a long history of a chronic heart failure, revealed an enlarged liver. Grossly, a liver had a motley pattern. Macro-scopically, the sectional view looked like a nutmeg on incision. Which term most correctly defines these alteration?A. * General venous plethoraB. General arterial plethoraC. AnemiaD. HemorrhageE. Bleeding2. A post-mortem of a 43-year-old female revealed multiple, hemorrhagic infarcts in lungs. Some lungs' vessels had reddish-brown dense masses within lumens, which were not attached to the vessel walls. Varicose phlebectasia of the legs with thromboses of some veins were also determined. Which pathological process occurred in this case?A. * Thromboembolism of pulmonary artery.B. Fat embolism of pulmonary' artery.C. Tissue embolism of pulmonary artery.D. Foreign bodies embolism of pulmonary artery.E. Gas embolism.3. A male patient died after a abdominal surgery. During the postmortem numerous thrombuses were found in the veins of the pelvis. Thromboembolic syndrome was diagnosed. Where it is necessary to search for thromboembolies?A. *Lungs arteryB. Portal veinC. Left ventricle of heartD. BrainE. Veins of the lower extremity4. The histological investigation of a liver detects venous plethora of center lobules, dystrophy and atrophy of hepatocytes in the venous plethora's area, fatty a dystrophy of hepatocytes on the periphery of a lobe. The replacement fibrosis in places of an atrophy of the hepatocytes was also revealed. Which pathological process does mis refer to?A. * 'Nutmeg' liver with precirrhotic phenomena.

Page 39: Pat an Anew

39

B. Biliary hepatic cirrhosis. С Fatty hepatosis.D. Hepatitis.E. Toxic dystrophy of a liver.5. A 35-year-old patient complains of repeated vomiting, diarrhea, decreased arterial pressure and tachycardia. He supposes this condition related to the food poisoning. Laboratory tests detected Salmonella's infection. Hematological examination revealed an increased munber of erythrocytes per unit volume. Which circulatory disturbance took place to create this pathology?A. *Clotting of a blood.B. Hemolysis of erythrocytes and compensatory induction of ahemogenesisC. General arterial plethoraD. PolycythemiaE. Hyperchromatic anemia6. A male with a history of myocardial infarct died of cardiovascular failure. A post-mortem revealed a replacement fibrosis, hypertrophy of the myocardium and dilatation of the cavities, especially the right ventricle. The liver was enlarged. Its surface was smooth. Grossly, a motley pattern with dark red dots on the grey a background was revealed. Histologically, central zones of the lobules were hy-peremic. On the periphery, around of periportal tracts hepatocytes demonstrated fatty dystrophy. Name these liver changes.A. *"Nutmeg" liver (chronic venous plethora)B. Pseudo 'nut-meg' liverC. AmyloidosisD. Cirrhosis of a liverE. Steatosis of a liver7. A male patient involved in the traffic accident received a wound in the neck due to broken glass. The bleeding was small, but a short time after the accident he died of acute dyspnea. A post-mortem of the heart revealed bubbles excretion when pericardium filled with water. Indicate which pathological process took place:A. *Air embolismB. Gas embolismC. Fat embolismD. ThromboembolismE. Foreign bodies embolism8. A 52-year-old male with long history of rheumatic heart disease died of chronic heart failure. A post-mortem revealed brown colour, enlarged dense lungs. Name the changes in lungs.A. *Brown induration of lungsB. Acute bronchitisC. Horny-comb lungsD. Chronic bronchitisE. Chronic emphysema9. A patient, who had fast elimination of 10 liters of an ascitic liquid from abdomen, suddenly lost his consciousness. What was the cause of this phenomenon?A. * Anemia of the brainB. Thrombosis of the cerebral arteriesC. Brain hemorrhageD. Arterial hyperemiaE. Thrombosis of the cerebral veins10. A young male with a history of rheumatic heart disease since childhood, gradually develops mitral stenosis, accompanied with episodes of heart failure. He presents to a hospital complaining of coughing with a reddish-brown sputum. Name probable changes at patient's lungs.A. *Brown induration of lungsB. Emphysema of lungsC. Atelectasis of lungsD. PneumosclerosisE. Bronchiectasis11. At autopsy an elderly female is found to have a blood clot in the femoral artery, which grossly had a striped pattern. Microscopy revealed a congestions of fibrin fibers and broken down red and white cells. Which is the most likely type of thrombus ?A. * Mixed thrombusB. Postmortem convolutions of a bloodC. ThromboemboliD. Hyaline thrombusE. Red thrombus12. A patient has a purulent otitis and thrombosis of a sigmoid sinus of a dura mater. Name the complication if thrombus breaks down and fragments circulate to distal vessels.A. *Thromboembolism of branches of pulmonary artery

Page 40: Pat an Anew

40

B. Thromboembolism of vessels of a brain with development of a grey softening of the brainC. Thromboembolism of vessels of a retina of an eye and development of blindnessD. Thromboembolism of coronary arteriesE. Local anemia13. A 43-year-old male, with a history of traffic accident and poly trauma, including multiple bone fractures, presented to the hospital. On the second day after the trauma, he complained of a pain in the right half of the chest, heavy breathing. At night, he died due to progressive heart and respiratory failure. Microscopic investigation revealed Sudan-positive orange drops in pulmonary and cerebral vessels that completely occlude the lumens of microcirculatory vessels. What is the most likely complication led to a patient's death?A. *Fat embolism.B. Gaseous embolism.C. Medicamentous embolism.D. Microbial embolism.E. Thromboembolism.14. A 62-year-old patient had a surgery due to acute appendicitis. She had been placed in a bed for 5 days. After she decided to get out of bed she experienced a shortage of air, her face became cyanotic and she lost her consciousness. After unsuccessful resuscitation the patient died. A post-mortem revealed a thromboem-bolism of the pulmonary artery. Which of the following is most likely source of mromboembolism?A. Thrombosis of veins of the lower extremityB. Thrombosis of a portal veinC. Thrombosis of mesenteric arteriesD. Thrombosis in a left ventricle of heartE. Ball-shaped thrombus of an auricle15. A male with a fracture of the shoulder bone carried on overlapped plaster bandage. Suddenly, an arm and the visible part of the forearm became cyanotic, cold, and edematous. Which of the following is most likely happened?A. *Local venous plethoraB. Local arterial plethoraC. Local anemiaD. StasisE. Thrombosis16. An elderly female develops acute disorder of the cerebral blood circulation, followed with coma, resulted in fatal outcome. A post-mortem revealed in the right hemisphere of the brain a large cavity, filled with blood. Which pathological process took place in the brain?A. *HematomaB. Hemorrhagic infiltrationC. Infarct of the brainD. DiapedesisE. Edema of the brain17. A male suddenly died after an open fracture of the clavicle. A post-mortem revealed in the right ventricle of the heart and pulmonary arteries the foamy blood. Which one was the cause of death?A. *Air embolismB. Bacterial embolismC. HemorrhageD. Tissue embolismE. Fat embolism18. A male with a history of the decompression sickness developed symptoms of acute cerebral circulation disorder and died soon. A post-mortem revealed in the left hemisphere of the brain the locus of a grey softening of the brain, which was 6x5x3 cm. Which one is most likely to cause the death of the patient?A. *Gas embolism.B. Fat embolism.C. Thrombosis.D. Thromboembolism.E. Atherosclerosis of the vessels.19. A post-mortem of an elderly man with a long history of the ischemic heart disease and heart failure revealed a nutmeg liver, brown induration of lungs, cyanotic induration of kidney and spleen. Indicate, what type of the blood circulation disorder is most likely?A. * Chronic general venous plethora.B. Arterial hyperemia.C. Acute general venous plethora.D. Acute anemia.E. Chronic anemia.20. A 50-year-old male with a myocardial infarction died from the heart failure. A post-mortem revealed the edema of the lungs and petechial hemorrhages at serous and mucus membranes. Microscopic examination indicated marked

Page 41: Pat an Anew

41

dystrophic alterations of the nephron's epithelium at proximal canaliculi of kidneys. Also, the centrolobular hemorrhage and necrotic zones were found in the liver. Name the type of the blood circulation disorder.A. * Acute general venous plethora.B. Arterial hyperemia.C. Chronic general venous plethora.D. Acute anemia.E. Chronic anemia.21. A neonate died of intoxication. A microscopic examination of an umbilical vein revealed a diffuse inflammatory infiltration of the vessel's wall. Also, its lumen was filled with thrombus, rich with leucocytes and bacterial colonies. Karyorrhexis was detected in many leucocytes. Which is most likely outcome of a thrombus in that case?A. * Septic autolysis.B. Aseptic autolysis.C. Organization and vascularization of the thrombus.D. Thromboembolism.E. Calcification of a thrombus.22. A 68-year-old female is admitted to the hospital for treatment of deep vein thromboses. The next day she suddenly died. At autopsy a large plug of laminated blood clot is found to occlude the main pulmonary artery. Which is likely to be found in her lungs?A. * ThromboembolismB. ThrombosisC. Tissue embolismD. Foreign bodies embolismE. Fat embolism23. A 77-years-old female with unstable angina presents to physician with slowly increasing heart failure symptoms. On the night of admission to the hospital she dies. A post-mortem examination revealed the enlarged liver with dense texture and the rounded edges. Sectional view had a nutmeg pattern with dark red dots on the yellowish background. Which pathological process resulted in liver's alteration?A. * Chronic venous plethoraB. Acute venous plethoraC. Arterial plethoraD. Arterial anemiaE. Chronic hemorrhage24. A post-mortem of a 53-year-old male with a long history of hypertension revealed the cavity in the occipital lobe of the brain. It measured 2x1 cm, had a brownish smooth walls and filled with transparent liquid. Which is the most likely diagnosis?A. * Cyst after hemorrhagesB. Softening of the brainC. Abscess of a brainD. Developmental defect of a brainE. Cyst after softening of the brain25. A 65-years-old patient, who presented with a 10 years history of ischemic heart disease, died due to heart failure. An autopsy revealed the cyanotic induration of both spleen and kidneys, brown induration of lungs and "nutmeg" liver. Which is the most likely type of the blood circulation disorder, resulted in such changes of internal organs?A. * General chronic venous hyperemiaB. Genera! acute venous hyperemiaC. General arterial hyperemia after an anemiaD. Arterial ischemia as a result of reallocating a bloodE. Local chronic venous hyperemia26. A 63-year-old patient with long history of ischemic heart disease and repeated myocardial infarction died due to progressive cardiovascular failure. A postmortem revealed an enlarged, dense spleen with the dark red colour of the sectional view. At the microscopic examination of spleen the pulp's sclerosis and atrophy of the follicles were found. Which term most correctly defines theses changes?A. *Cyanotic induration of a spleenB. Sago spleenC. Waxy spleenD. Porphyry spleenE. Septic spleen27. A young male fractures his pelvis and femur in a motor vehicle accident. On the third day he dies after respiratory distress and cerebral dysfunction. A histological examination revealed Sudan-positive orange globules scattered within cerebral cortex, kidneys and lung microcirculatory vessels. Which is most likely diagnoses?A. * Fat embolismB. Gas embolismC. Tissue embolism

Page 42: Pat an Anew

42

D. Microbial embolismE. Thromboembolism28. A cosmonaut died due to the air-tightless of the space-shuttle cabin. Microscopic examination of the vessels of the internal organs revealed multiple bubbles. Liver cells developed fatty dystrophy. In the brain and spinal cord multiple small, grey, soft lesions were discovered. Indicate the most probable cause of such alterations.A. *Gas embolismB. Air embolismC. Fat embolismD. ThromboembolismE. Tissue embolism29. A patient with a long history of rheumatic disease died of cardiopulmonary failure. A post-mortem revealed a stenosis of the mitral orifice as well as a brown induration of lungs. Which term most correctly defines this blood circulation disorder?A. *Chronic left ventricular failure.B. Chronic right ventricular failure.C. Acute left ventricular failure.D. Acute right ventricular failure.E. Portal hypertension.30. A post-mortem of a 59-year-old patient with long history of idiopathic hypertension revealed in brain a cavity with rusty colour walls. Which process preceded these changes?A. *HematomaB. DiapedesisC. Ischemic infarctD. PlasmorrhagiaE. Abscess31. A 65-year-old- patient presented with liver cirrhosis. The removal from his abdominal cavity of 10 liters of ascitic liquid resulted a collapse. Grossly his peritoneum was hyperemic. Define the type of hyperemia in peritoneum.A. * Post-anemic hyperemiaB. Collateral hyperemiaC. Complementary hyperemiaD. Inflammatory hyperemiaE. Caused by arteriovenous fistula32. A post-mortem of a 60-year-old female with a history of secondary hypertension revealed a cavity at the right hemisphere of the brain 4x2,5 cm, filled with red clots of blood and softened brain tissue. What term best characterized the brain changes?A. *Hematoma.B. Hemorrhagic impregnation.C. Ischemic infarct.D. Cyst.E. Abscess.33. An inexperienced scuba diver ascends from a depth of 55 meters to the surface in about 5 minutes. Shortly after surfacing he complains of severe muscle contraction and intense abdominal pain. What is most likely diagnose?A. *Gas embolismB. Air embolismC. Fat embolismD. ThromboembolismE. Tissue embolism34. A 78-year-old male, with a history of stroke 2 years ago, died of pneumonia as a complication of severe influenza. At post-mortem, gross investigation of his brain revealed a cerebral cyst with rusty color of its walls. Perls' test was positive. Name the process reveled within a cystic wall.A. *Local hemosiderosis.B. Common hemosiderosis.C. Local hemomelanosis.D. Infiltration of bilirubin.E. Primary hemochromatosis.35. A 54-years old male with a history of ischemic heart disease presents at hospital with recurrent myocardial infarction. Few days later, he died due to cardiac failure. Post-mortem revealed an enlarged solid spleen of dark cherry color on the cut surface. Microscopically, pulp sclerosis and follicles atrophy were found out. What is the most likely term to define spleen's alterations??A. *Cyanotic induration of spleen.B. Sago spleen.C. Lardaceous spleen.

Page 43: Pat an Anew

43

D. Porphyry spleen.E. Septic splenitis.36. A post-mortem of 77-year-old male, with a long history of cerebral atherosclerosis, revealed an atrophy of his cerebral cortex. What was the most likely cause of cortex alterations?A. *Insufficiency of blood supply.B. Pressure.C. Physical and chemical factors.D. Neurotic.E. Dysfunctional.37. A 56-year-old male with a history of cerebral atherosclerosis suddenly dies. At, a there is a thrombus within a middle cerebral artery's lumen. Gross inspection revealed a grey flabby focus at the parietotemporal part of the left hemisphere of his brain .What are the most likely alterations diagnosed in a brain tissues?A. *Infarction.B. Sequestrum.C. Gangrene.D. Caseous necrosis.E. Fibrinoid necrosis.38. An autopsy of a patient who died suddenly at emergency room revealed in his brain a cavity of irregular shape (5 x 3.5 cm) filled with blood clots and macerated cerebral tissue. An area of cavitary destruction had a rim of brown discoloration. This lesion was found within the subcortex nuclei at the right hemisphere of brain. What is the most likely definition of described pathology?A. *Hematorna.B. Hemorrhagic impregnation.C. Ischemic infarction.D. Cyst.E. Abscess.

Inflammation.1 A histological investigation of enlarged neck lymphatic node revealed a congestion of epithelial cells, lymphocytes and giant Pirogov-Langhan's cells. In the center of lesion a caseous necrosis was present. Which pattern of inflammation do the lymphatic node display?A. * TuberculosisB. RhinoscleromaC. SarcoidosisD. Glanders.E. Syphilis.2. A 3-year-old child died of a meningococca! fever. A macroscopic study revealed swallowed meninges which had yellow-green colouring. Which pattern of inflammation was most likely in meninges?A. *Fibrinous-purulent inflammationB. Serous inflammationC. Hemorrhagic inflammationD. Catarrhal inflammationE. Necrotic inflammation '3. A 37-year-old male presents with a fever, dyspnea and pain in the right part of the chest. The pleurocentesis gave 700 mis of thick yellow-greenish liquid. What pathological process was diagnosed in a pleural cavity?A. *Empyema of a pleuraB. BronchopneumoniaC. Serous pleuritisD. Hemorrhagic pleuritisE. Carcinomatosis of a pleura4. A 25 years old male presents with a 2-day history of itching and redness in the buccal area that appeared after shaving. Physical examination by family physician revealed vesicles filled with transparent fluid on the background the hyperemic buccal area. What is the most likely type of fluid in the vesicles?A. *Serous exudate.B. Transsudate.C. Purulent effluent.D. Mucous exudate.E. Hemorrhagic exudate.5. An autopsy of a 34-years old female, with a history of croupous pneumonia revealed opaque fluid in her pleural cavity. There was also a grey membrane on the visceral pleura. What is the most likely type of the pleura's inflammation?A. *Fibrinous.B. Catarrhal.

Page 44: Pat an Anew

44

C. Suppurative.D. Granulomatous.E. Hemorrhagic.6. A 4-year-old girl with 3 days history of diphtheria presents to the emergency with croup symptoms. Intensive care was unsuccessful and child died at the hospital. A post-mortem revealed thickened, edematous, dull mucous tunic of the larynx, trachea, and bronchi covered by grayish membrane easily separated from tissues underneath. What is the most likely type of inflammation?A. *Fibrinous.B. Serous.C. Suppurative.D. Mixed.E. Catarrhal.7.At post-mortem a 5-year-old boy is found to have a severe tracheobronchitis complicated with asphyxia. Grossly, a thick, gray, leathery membrane covered the mucous tunic of trachea and bronchi. The membrane loosely attached to underlying tissues and easily removed with forceps. The lumen of the segmental bronchi was blocked with gray masses of tissue debris, which microscopically consisted with necrotic tissues, neutrophils, fibrin and bacteria. What is the most likely type of inflammation?A. *Croupous.B. Catarrhal.C. Diphtheritic.D. Purulent.E. Serous8. An autopsy of a 50-year-old male, who died of dysentery, revealed the hypere-mic mucosa of the colon, coated with grey membranes, which can be removed from tissues underneath with some effort. Which type of an inflammation described in the intestine?A. *Diphtheroid inflammationB. Croupous inflammationC. Hemorrhagic inflammationD. Serous inflammationE. Catarrhal inflammation9. A 30-year-old male has a neck lymph node removed for examination. Histologi-cal investigation revealed granulomas, which consisted of epithelioid, lymphoid and rnultinuclear Langhans giant cells. Granulomas' centers were presented with necrosis. What causative agent needs to be revealed in the necrosis zone for support of the diagnosis of tuberculosis?A. *Mycobacterium tuberculosisB. Treponema pallidumC. StaphylococcusD. Frisch bacillusE. Salmonella10. An autopsy of a 53-year-old male, with a history of crupous pneumonia, revealed in his dextral pleural cavity 900 ml of cloudy, grey-yellowish colored liquid. Pleural membranes were found to be dim and plethoric. Name the clinical -morphological form of the inflammation in the pleural cavity?A. * Empyema.B. Fibrinous inflammation.C. Phlegmon.D. Chronic abscess.E. Acute abscess.11. An autopsy of a 53-year-old male, who died from chronic renal failure, revealed the colon's mucosa, covered with grey — yellow membranes, densely coherent to tissues underneath. The removal of the membranes resulted in ulcers formation. Name a type of an inflammation?A. *DiphtheroidB. SerousC. CatarrhalD. CroupousE. Purulent12. A 65-year-old female with a long history of chronic glomerulonephritis died from chronic renal failure. A post-mortem revealed on the surface of the epicardium and pericardium grey-whitish villous membranes. Which pathological process took place in the pericardium?A. *Fibrinous inflammationB. OrganizationC. Proliferative inflammationD. Hemorrhagic inflammationE. Arterial plethora

Page 45: Pat an Anew

45

13. An elderly man presented to his physician with the complicated nasal breathing. The histological investigation of a biopsy from his nose septum revealed granulo-matous inflammation in the mucosa with groups of large vacuolated histiocytes containing bacteria (Mikulicz's cells). What is the most likely diagnosis?A. *RhinoscleromaB. SyphilisC. TuberculosisD. MalleusE. Lepra14. An elderly man, with a history of an operation, presented with a sepsis, developed on a background of a decreased immune reactivity of an organism. A disease resulted in fatal outcome. Microscopical investigation of the abdominal wall revealed the diffuse segmentonuclear leukocyte's infiltration of intermuscular spaces, edema of a tissue and lysis of muscle fibers. Define the type of inflammation?A. * Phlegmon.B. Diphtheroid inflammation.C. Abscess.D. Necrosis.E. Catarrhal inflammation.15. A 64-years-old male died of rheumatism. An autopsy revealed epicardium covered with villiferous grey color membranes, which were easily separated from tissues underneath. The separation of membranes presented an edematous, hyperemic surface of epicardium. What type of an inflammation revealed in pericardium?A. *Fibrinous pericarditisB. Purulent pericarditisC. Hemorrhagic pericarditisD. Proliferative pericarditisE. Catarrhal pericarditis16. A 50-year-old male died of a pulmonary and cardiac insufficiency. An autopsy revealed in a left lung a 4 x 4 cm cavity, filled with pus. The wall of a cavity had a rough pattern, presented by pulmonary tissue. What is the most likely diagnose?A. *Acute abscessB. Chronic abscessC. Cavernous tuberculosisD. HydatidosisE. Fibrosing alveolitis17. A post-mortem of a man revealed in his abdominal cavity approximately 2.0 liters of a purulent exudate. A peritoneum was found to be dull, with a grayish tint. The intestinal serous membranes were covered with grayish membranes, which were removed easily. Which of the following is the most accurate diagnosis?A. * Fibro-purulent peritonitisB. Hemorrhagic peritonitisC. Serous peritonitisD. Tuberculous peritonitis E.18. A histological examination of the myocardium of a man, who died from heart failure, revealed a sclerosis of the pericardial connective tissue and diffuse infiltration by lymphocytes, macrophages, plasmocytes and single neutrophils. What is the most likely type of inflammation in a heart?A. * Interstitial productive.B. Granulomatous.C. Alterative.D. Exudative diffuse.E. Exudative focal.19. A 28-year-old male presented to the hospital with a fever and weakness, (asthenia). A biopsy was taken from one of his enlarged neck lymph nodes for histological examination. Microscopic investigation revealed the foci of necrosis, surrounded by epithelioid cells, giant multinuclear Langhans' cells and also lymphocytes. What is the most likely diagnosis?A. *TuberculosisB. Hodgkin's diseaseC. LympholeukemiaD. SarcoidosisE. Syphilis20. A microscopic examination of a kidney biopsy revealed foci with granular, eosinophilic masses in their centers. These lesions were surrounded by an infiltrate, consisted of lymphocytes, epithelioid cells and giant Langhans1 cells. What is the most likely pathological process?A. * Granulomatous inflammationB. Coagulative necrosisC. Caseous necrosis

Page 46: Pat an Anew

46

D. Alterative inflammationE. Proliferation and derivation of macrophages21. A young male presented to his physician with considerably enlarged, hypere-mic, painful tonsils. A gross examination of tonsils revealed dense, dirty-gray membranes on the surface. These membranes have spread to the hard palate and were intimately attached to tissues underneath. An attempt to remove the membranes resulted in bleeding. Which pathological process indicates these morphological changes?A. *Diphtheroid exudative inflammation.B. Croupous exudative inflammation.C. Catarrhal exudative inflammation.D. Purulent exudative inflammation.E. Hemorrhagic exudative inflammation.22. A 38-year-old man, with a history of chronic pyelonephritis, died from the chronic renal failure. Medical record showed that an auscultation in the hospital determined a patient's "pericardial frictions rub". An'autopsy revealed a dim, rough, and hairy - like epicardium. What type of pericarditis presented in that case?A. * CroupousB. DiphtheroidC. PurulentD. IchorousE. Serous23. A child presented to a hospital with symptoms of asphyxia. A physical examination revealed in the larynx whitish membranes, which obstructed lumen and were easily separated from tissues underneath.Name the most probable type of an inflammation in the larynx?A. *Croupous inflammationB. Hemorrhagic inflammationC. Diphtheroid inflammationD. Catarrhal inflammationE. Purulent inflammation24. A macroscopic investigation of trachea revealed a dim, hyperemic mucosa, covered with grey - white membranes. Which one is the most likely form of the inflammation?A. *FibrinousB. PurulentC. SerousD. ProliferativeE. Mixed25. An elderly man presented with a decomposition of some segments of spongy and cortical layers of an anticnemion bones. The cavities are filled with creamy greenish-yellow masses. What is the type of inflammation?A. *PurulentB. CatarrhalC. SerousD. ProliferativeE. Mixed26. A female patient presented to the hospital with a fever, asphyxia and pain in the right part of her chest. During a pleurocentesis procedure 700 ml of a viscid yellow-green liquid was removed. What is the most likely diagnosis?A. *Empyema of a pleuraB. Carcinomatosis of a pleuraC. Serous pleurisyD. Fibrinous pleurisyE. Hemorrhagic pleurisy27. An autopsy of the 58-year-old man, revealed in his liver a focus of tissue destruction, 4 cm in diameter, filled with a yellow-green liquid. What is the most likely diagnose?A. * AbscessB. PhlegmonC. AnthraxD. EmpyemaE. Granuloma28. A histological examination of biopsy from the nose mucous revealed epi-thelioid cells, plasmocytes, Mikulicz cells, and eosinophilic (acidophilic, Russell's) bodies. What is the most likely diagnosis?A. * RhinoscleromaB. SyphilisC. TuberculosisD. Droplet infectionE. Allergic rhinitis

Page 47: Pat an Anew

47

29. A physical examination of a 5-year-old boy reveals the fauces and the tonsils enlarged, plethoric, and coated with irremovable whitish membranes. Which one of the following characterizes the changes in the fauces and tonsils?A. * Diphtheritic inflammationB. Caseous necrosisC. Fibrinous necrosisD. Croupous inflammationE. Purulent inflammation30. A 49-year-old man, with 14 days history of acute lobar staphylococcal pneumonia in the left lower lobe of a lung, died of pulmonary and cardiac insufficiency. A post-mortem revealed in the upper lobe of a right lung an oval cavity, approximately 5 cm in diameter, filled with pus and connected with bronchus. Which complication of acute pneumonia took place in this case?A. * Abscess of the lungB. Gangrene of the lungC. Thromboembolia of a pulmonary trunkD. Bronchiectasis diseaseE. Acute bronchitis31. An autopsy of an elderly male revealed a cavity with compacted walls filled with a thick, greenish liquid, which had a stinking odor. What is the most likely disease?A. * Abscess of the lung.B. Gangrene of the lung.C. Infarct of the lung.D. Cavern of the lung.E. Tuberculoma of the lung.32. A 62-year-old man died of croupous pneumonia. A post-mortem revealed in the pleural cavity an opaque liquid and a grayish membrane on visceral pleura. What is the most likely type of inflammation on the visceral pleura?A. * Fibrinous inflammationB. Catarrhal inflammationC. Purulent inflammationD. Granulomatous inflammationE. Hemorrhagic inflammation33. A 5-year-old child has a fever and pain at swallowing. A physical examination revealed enlarged, dark-red palatine tonsils. They were coated with a grey - yellow membrane which was intimately attached to the surface of the tonsils. Which kind of inflammation described in the tonsils?A. * Diphtheritic inflammationB. Croupous inflammationC. Hemorrhagic inflammationD. Purulent inflammationE. Catarrhal inflammation34. An autopsy of a dead body revealed 200 ml of a viscid yellow-green liquid in the abdominal cavity. What is the most likely form of exudate inflammation?A. * Purulent inflammationB. Serous inflammationC. Fibrinous inflammation.D. Hemorrhagic inflammation.E. Ichorous inflammation35. A post-mortem of a young male revealed congested, thickened, opaque, yellowish-green meninges. What type of exudative inflammation these changes are characteristic for?A. * Purulent inflammationB. Serous inflammationC. Hemorrhagic inflammationD. Fibrinous inflammationE. Catarrhal inflammation36. A woman, with a history of her hands' skin thermal burn, presented to physician painful blisters, filled with opaque liquid. What is the most likely type of inflammation?A. * Serous inflammationB. Productive inflammationC. Croupous inflammationD. Granulomatous inflammationE. Diphtheritic inflammation37. A 44-year-old male had a skin biopsy procedure. Histological investigation of a tissue sample showed a caseous necrosis surrounded by infiltrate, consisted from lymphocytes, some giant cells and the growth of connective tissue. In addition, the endovasculitis was diagnosed. What is the most likely pathological process?

Page 48: Pat an Anew

48

A. * Productive granulomatous inflammation.B. Productive interstitial inflammation.C. Abscess.D. Catarrhal inflammation.E. Serous inflammation38. A 7-year-old child, with a history of angina, presented with enlarged paratra-cheal, bifurcation and neck lymph nodes. A histological examination of his neck lymph node revealed the foci of necrosis, surrounded by lymphocytes, epithelioid cells and Pirogov-Langhans' cells. Which one is the most likely pathology?A. *TuberculosisB. SarcoidosisC. RhinoscleromaD. MalleusE. Syphilis39. The histological examination of a lung tissue revealed a lesion presented with a locus of necrosis, surrounded by regular lines of epithelioid, lymphoid cells, plasma cells, macrophages and giant Pirogov-Langhans' cells.Define a kind of inflammation?A. *Tnberculosis inflammationB. Banal productive inflammationC. Inflammation at a lepraD. Exudative inflammationE. Alterative inflammation40. A 6-year-old child presented to infectious disease department with a body temperature 38°C, punctulated, bright red rash on a skin. The scarlet fever diagnosis is made. The fauces mucosa was brightly hyperemic, edematous; tonsils were markedly enlarged, with the dim yellowish-grayish centers and sites of black color. What inflammation underlies changes in fauces?A. *Pumlent-necroticB. FibrinousC. HemorrhagicD. SerousE. Catarrhal41. A rectoromanoscopy revealed edematous, reddish color mucosa of rectum and sigmoid colon, covered with a thick layer of mucus. Name the kind of inflammation?A. *CatarrhalB. PurulentC. MixedD. HemorrhagicE. Fibrinous42 A patient died with symptoms of the brain's edema and the dislocation of the stem. A post-mortem revealed thickened, opaque, grayish-green color pia mater. Which pathological process took place in pia mater of meninges?A. *Diffuse purulent leptomeningitisB. Meningococcal leptomeningitisC. Tuberculous leptomeningitisD. Malignant arachnoendomeliomaE. Hydrocephalus43. A 5-year-old child presented to a hospital with acute fever (up to 38° C), rhinitis, cough, lacrimation, and a patchy rash on the skin. His pharyngeal mucosa was edematous and hyperemic, with whitish maculae on the cheeks. What type of inflammation is the background of described changes?A. *CatarrhalB. PurulentC. FibrinousD. HemorrhagicE. Serous44. A 3-year-old child presented to a hospital with asphyxia. Examination of the larynx revealed whitish membranes, which occluded a lumen and easily extracted. Diphtheria was suspected. Which form of inflammation took place in the larynx?A. *Croupous inflammationB. Catarrhal inflammationC. Diphtheroid inflammationD. Serous inflammationE. Purulent inflammation45. A 47-year-old male presented with a sub-febrile temperature and enlarged lymph nodes. A biopsy investigation of a lymph node revealed numerous granulo-mas with a caseous necrosis in the centers, surrounded by epithelioid cells, giant multinuclear Langhans' cells and lymphocytes.

Page 49: Pat an Anew

49

What is the most likely diagnosis?A. * TuberculosisB. LymphogranulomatosisC. LymphosarcomaD. LympholeukemiaE. Simple lymphadenitis46. At a 45-year-old man the biopsy procedure of his nasal cavity's mucosa was performed. The diagnosis of rhinoscleroma made. What cells are typical for the disease?A. *Mikulicz's cellsB. Pirogovs' - Langhans' cellsC. Plasma cellsD. LymphocytesE. Schaumarm"s bodies47. An autopsy of a 53-year-old male showed many white, millet sized nodules in his lungs. A microscopic examination revealed granulomas with a necrosis in their centers and epithelioid, lymphoid, plasma cells, macrophages and numerous large multinucleated cells (Langhans' cells) on the periphery of a lesion.What is the most likely form of granuloma?A. *Giant - cell granulomaB. MacrophagalC. Foreign body granulomaD. Epithelioid cells E -48. A patient presented to the hospital with combustion of his right hand. A physical examination revealed a desquamation of epidermis with blisters formation. The blisters were filled with opaque liquid. What most probable inflammation described in that case?A. * SerousB. PurulentC. CatarrhalD. PutrefactiveE. Fibrinous49. A 43-year-old patient complains of hindered nasal breaming. Histological investigation of a biopsy of his nasal cavity mucosa revealed lymphoid cells, epithelioid cells, plasma cells and Mikulicz's cells. What is the most likely diagnosis?A. *RinoskleromaB. Glanders (malleus)C. TuberculosisD. LepraE. Syphilis50. A 54-year-old patient complains of complicated nasal breathing. A nasal mucosa was thickened. Histological investigation of a biopsy revealed Mikulicz's cells, epithelioid cells, plasma cells, lymphoid cells, and hyaline sphere- like aggregates. What is the most likely diagnosis?A. *SkIeromaB. Adenoviral rhinitisC. Allergic rhinitisD. Rhinoviral infectionE. Meningococcal nasopharyngitis51. A sample of cervix uteri was taken for histological examination. A microscopical investigation revealed a tissue's inflammatory infiltration with vascular walls involvement of small vessels, arteries and veins. In addition, there were also determined a presence of plasma cells, lymphocytes, epithelioid cells in the infiltrate and foci of sclerosis and hyalinosis of tissue. What is the most likely disease?A. *SyphilisB. TuberculosisC. LeukoplakiaD. Erosio colli uteriE. Condyloma52. A physical examination of tonsils and soft palate mucosa revealed white-grey color membranes, which are intimately attached to tissues underneath. An attempt to take out membranes results in formation of a deep tissue defect. Diagnose pathology on a mucosa of tonsils and a soft palate.A. *Diphtheritic inflammation В Serous inflammationC. Croupous inflammationD. Purulent inflammation

Page 50: Pat an Anew

50

E. Mixed inflammation53. Histological investigation of a biopsy revealed a granuloma, consisted of lymphocytes, plasma cells, macrophages with foamy cytoplasm (Mikulicz's cells) and numerous hyaline's sphere- like aggregates. About what disease it is possible to think?A. *RinoskleromaB. Lepra С SyphilisD. TuberculosisE. Actinomycosis54. A 40-year-old woman, with a history of the right palm's burn, presented to her physician an acute pain, reddening and swelling of a palm. In a few minutes there was a bubble, filled with transparent yellowish liquid. The display of what pathological process the described changes are?A. *Exudative inflammationB. Traumatic edemaC. Alterative inflammationD. Proliferative inflammationE. Vacuolar dystrophy55. A 44-year-old man presented to the policlinic with a local pain in occipital area of a head and rise of a body temperature there. Gross investigation of a lesion zone revealed the cone-shaped cyanotic infiltration with a yellow-greenish apex, which rose above the surface of skin. What is the most likely diagnosis?A. * FuruncleB. PhlegmonC. AbscessD. CarbuncleE. Empyema.56. A skin sample was taken for histological research. Microscopical investigation revealed granulomas, consisted of macrophages' knots with adding of lymphocytes and plasma cells. There were also determined large macrophages with fatty vacuoles, which contained packed layers of causative agents of a disease (Virchow's cells). For what disease the given description is characteristic?A. *LepraB. TuberculosisC. SyphilisD. RinoskleromaE Glanders (malleus)57. Histological investigation of a skin biopsy revealed granulomas, which consisted of epithelioid cells, surrounded mainly by T-lymphocytes. In addition, few giant rnultinucleated cells (Langhans* cells) were determined among epithelioid cells. The caseous necrosis was diagnosed within the centers of granulomas. There was lack of blood vessels. What is the most likely disease?A. TuberculosisB. SyphilisC. LepraD. RinoskleromaE. Glanders (malleus)58. A histological investigation of a liver biopsy revealed granulomas, consisted of plasma cells and lymphoid cells. Small vessels were characterized by endo - and perivasculitis. Some foci of caseous necrosis were also determined. Such granulomas are typical for:A. *SyphilisB. TuberculosisC. LepraD. RinoskleromaE. Glanders (malleus)59. A young man had a removal of mucous membrane new growths in his nasal cavity. A histological investigation revealed diffuse lymphocytes, plasma cells and macrophages infiltration. Name the type of inflammation.A. *Inflammation with formation of polypsB. Inflammation with formation pointed condylomaC. Interstitial inflammationD. granulomatous inflammation E.Exudative inflammation60. A 4-year-old child presented to his physician with a cold, high temperature, nausea and vomiting. He was hospitalized and died in two days. An autopsy revealed thickened, dim, edematous, congested, greenish-yellow pia mater of men-inges. What variety of exudative inflammation developed in pia maters?A. *Purulent

Page 51: Pat an Anew

51

B. CatarrhalC. HemorrhagicD. FibrinousE. Serous61. A young man was operated because of phimosis. Histological investigation of a removed prepuce revealed polymorphic infiltrates, consisted of randomly located plasmocytes, epithelioid and lymphoid cells. The vasculitis was diagnosed in vessels. What disease such changes are characteristic for?A. * SyphilisB. TuberculosisC. ActinomycosisD. Periarteritis nodosaE. Lepra62. Microscopic investigation an elderly man's lungs revealed the foci of necrosis, surrounded by the bank of epithelioid cells and lymphocytes. Between lymphocytes and epithelioid cells the rounded giant cells, with a considerable quantity of nuclei, located on a periphery of a lesion were determined. Give the name for these changes:A. *Tuberculous granulomaB. Sarcoid granulomaC. Syphilitic granulomaD. Lepromatous granulomaE. Scleromatous granuloma63. A 6-year-old child presented to department of infectious diseases with acute pain in a throat, difficulty at swallowing, rise of body temperature up to 39°C and edema of neck. A gross oral investigation revealed enlarged, hyperemic tonsils, covered with plenty of yellow membranes, which intimately attached to the mucous membrane. An attempt to take off membrane results in deep, bleeding defect formation. What type of inflammation takes place?A. *DiphtheriticB. PurulentC. SerousD. CroupousE. Hemorrhagic64. A fragment of a stomach mucosa was taken for histology during endoscopy procedure. A microscopic investigation revealed intact mucous membrane, covered by mucus, thickened, edematous and hyperemic with a numerous tiny hemorrhages. Define the type of acute gastritis?A. * Catarrhal (simple)B. ErosiveC. FibrinousD. PurulentE. Necrotic65. An operatively removed appendix was sent to pathology department. Macro-scopical investigation revealed markedly enlarged appendix. His serous membrane was dim, congested, covered by fibrin's membranes. The appendix' walls were thickened. On a cut section pus in the lumen was detected. Microscopical investigation showed a plethora of vessels, edema of all appendixes' layers and diffuses infiltration by leucocytes. What is the type of inflammation?A. *PhlegmonousB. ApostematousC. SimpleD. SuperficialE. Gangrenous66. A 18-year-old girl had a neck lymphatic node removal at the biopsy procedure. Histological investigation showed nodules, consisted of banks of epithelioid, lymphoid cells and large multinucleated cells (Langhans' cells), located between them. In the centers of nodes a caseous necrosis was determined. What causative agent needs to be revealed in the necrosis zone for making final diagnosis?A. *Koch's mycobacteriumB. Mycobacterium lepraeC. Treponema pallidumD. RickettsiaE. Fungi67. A young man complains of an itch and a skin reddening, which has appeared after shaving. Objectively: a cheek skin is hyperemic; the blisters, filled with a transparent liquid, were detected. What character of a liquid in blisters?A. * Serous exudateB. TransudateC. Purulent exudateD. Mucosal exudate

Page 52: Pat an Anew

52

E. Hemorrhagic exudate68. A 4-year-old boy presented to a hospital with pain in a throat at swallowing and malaise. A physical examination revealed grayish - whitish membranes on a pharynx and tonsils on a background of moderate edema and hyperemia. These membranes were intimately attached to tissues underneath. What pathology the described changes testify to?A. inflammationB. DystrophyC. NecrosisD. MetaplasiaE. Reganerative process69. A 68-year-old man died from meningitis. An autopsy revealed thickened, congested, dim, edematous meninges (pia maters), which were saturated by creamy greenish-yellow exudate. Define the type of inflammation?A. *PuralentB. SerousC. Fibrinous croupousD. Fibrinous diphtheriticE. Hemorrhagic70. Biopsy of an oral cavity's ulcer presented for a histological research. A microscopical investigation revealed areas of caseous necrosis, surrounded by plasmo-cytes, epithelioid, lymphoid cells and few giant multinuclear Pirogov-Langhans cells. There were also small vessels with signs of endo- and perivasculitis. Choose the most likely diagnosis.A. *SyphilisB. TuberculosisC. LepraD. RinoskleromaE. Glanders (malleus)71. An autopsy of 32-year-old male revealed slightly swollen and hyperemic lungs, covered with numerous millet-like grayish nodules. Histological investigation determined lesions consisted of lymphoid, epithelioid and Langhans giant cells. What is the most likely diagnosis?A. *TuberculosisB. Croupous pneumoniaC. BronchopneumoniaD. Interstitial pneumoniaE. Bronchiectatic disease72. An operatively removed appendix was sent for histological examination. Mac-roscopical investigation revealed thickened appendix. His serous membrane was dim, congested, with whitish, loose membranes; the lumen contained turbid, whitish-yellow exudate. Histological study showed the diffuse neutrophil's infiltration of the appendix wall. Choose the most likely type of appendicitis.A. *PhlegmonousB. Gangrenous С SimpleD. SuperficialE. ChronicImmunopathological processes1. A 15 year old boy died of the heart failure. He had a history of rheumatic heart disease in his medical record. Histological investigation of the left auricle of the heart revealed the rheumatic granuloma (the Aschoff-Talalayev body) with a central fibrinous focus associated, with a perimeter plasma cells, macrophages and giant cells. Which of following best describe the type of immune response?A. *Cell mediated (Type IV, delayed hyper sensitivity reaction)B. Anaphylactic (Type I, immediate)C. Cytolytic (Type II, immediate)D. Immune complex (Type III, immediate)E. Reaction of transplant rejection.2. A 4-yers-old boy presents with multiple malformation of craniofacial bones. He died soon due to sepsis, resulted from bronchopneumonia. Blood test revealed the immunoglobulins within normal range. At autopsy the thymical aplasia was identified. Name the mane cause of death.A. *Cellular immunodeficiencyB. Combined immunodeficiencyC. Secondary immunodeficiencyD. Acute leukemiaE. Chronic intoxication3. A 48-year-old women with asthma presents with viral respiratory infection. The infection caused a status asthmaticus with a fatal outcome. Histological investigation of lung tissue revealed a contracted bronchus and a luminal plug containing mucus and cell debris. The submucosa was edematous and contained a mixed inflammatory

Page 53: Pat an Anew

53

infiltrate, including a lot of eosinophils and degranulated must cells. What is the mechanism of hypersensitivity presented in this case?A. *Humoral immunity (Type I, armphylactic reaction)B. Inflammatory reactionC. Autoimmune reactionD. Immune complex reaction (Type III of reaction)E. Cell-mediated immunity (Type IV of reaction)4. A young man presents with the enlargement of the regional lymph nodes next to his inflamed wound, Histological investigation of the lymph node biopsy revealed the increased number of macrophages, lymphocytes and lymphoid follicules within cortex zone. Plasma cells number also increased. These alterations are most likely result of:A. * Antigen stimulationB. The acquired insufficiency of lymphoid tissue (secondary immunodeficiency state)C. Hereditary immunodeficiency stateD. Neoplastic transformationE. Hypersensitivity reaction5. A newborn died after 3 days of life, as a result of a lethal outcome of intrauterine pneumonia. At autopsy a newborn is sound have a great reduction of his thymic weight and it's sizes. Histological investigation of thymus revealed the unclear boundary between cortex and medulla zone as a result of lymphocytes amount reduction. A few Hassall corpuscles were found. They were focally keratinized and had concentric aggregated of epithelial cells. The gland was also exhibited lobe atrophy and sclerosis. What pattern of reaction did the thymus demonstrate?A. *Accidental thymus transformation (involution)B. Thymus atrophyC. Thymus hypotrophyD. Thymus aplasia (agenesis)E. Thymus neoplasia6. A 3-monm-old child died because of sepsis. An autopsy revealed thymic hy-poplasia, both sizes and mass of spleen reduction. Histologically, in spleen there was lack of the periarterial follicular T-zone and remarkable absence of cells at a red spleeny pulp. Lymph nodes investigation showed also absence of cells within paracortical (T-cells area). B-cells areas in immune system organs looked normal. What term best characterizes described above changes? A. Di George's syndromeB. Combined immunodeficiency syndromeC. Bruton's disease (X-linked agammaglobulinemia)D. Accidental thymus transformationE. Acquired immunodeficiency syndrome (AIDS)7. A morphological investigation of a kidney graft revealed a diffuse stromal infiltration by lymphocytes, plasmocytes, lymphoblasts, plasmablasts, and also a ne-crotic arteriitis. What pathology has developed in a graft?A. *Immunological graft rejectionB. GlomerulonephritisC. Ischemic kidney damageD. Tubular necrosesE. Pyelonephritis8. An 8-year-old girl, with a history of acute tonsillitis 3 weeks before the application, presented with nephrotic syndrome (proteinuria, haematuria and cylindruria). These testify the glomerular basal membrane's lesion. What is the mechanism of the basal membrane pathology?A. * Immune complex mechanismB. Granulomatous mechanismC. Antibody-mediated mechanismD. Reagin-mediated mechanismE. Cytotoxical mechanism9. At 46-year-old patient with autoimmune gastritis a biopsy was taken. Histologically, there was an infiltration by lymphocytes and macrophages of a gastric mucous membrane. What type of hypersensitivity presented in that case?A. * Type IY (cell-mediated hypersensitivity)B. Type II (reagin type)C. Type II (antibody-mediated cytotoxicity)D. Type III (immune complex cytotoxicity)E. Type V (granulomatous)10. Histological investigation of a lymph node revealed a considerable quantity of the enlarged lymphoid follicles with big germinal centers, and plenty of mitotic figures. What pathology these changes are characteristic for?A. *Antigen stimulation with follicle hyperplasiaB. Atrophy of lymphoid tissueC. Lymphosarcoma

Page 54: Pat an Anew

54

D. LymphogranulomatosisE. Lympholeukemia11. A child had a subcutaneous injection of tuberculin for a testing. Twenty-four hours later a physical examination revealed the expressed hyperemia and induration of tissues in the place of injection. What was the mechanism of these changes?A. "Cellular cytotoxicityB. Reagin cytotoxicityC. Antibody cytotoxicityD. Granulorhas formationE. Immune complex12. A 6-year-old child had the Mantoux test. In 3 days there was an inflammatory induration and a reddening of a skin 15 mm in diameter. That was considered as the positive test. What type of hypersensitivity reaction presented in that case?A. *Hypersensitivity of slow typeB. Immune complex hypersensitivityC. Complement-mediated cytotoxicityD. Hypersensitivity immediate type E.13. Physical examination of a pregnant woman with an Rh-negative blood revealed a high level of antierythrocyte antibodies. The cutaneous flap of her Rh-positive husband applied for depression of antibodies. In 2 weeks a flap was rejected. Microscopical investigation showed a disturbance of blood circulation, edema and cellular infiltration mainly by lymphocytes, neutrophils and macrophages. What is the most probable pathology?A. *Transplantation immunityB. Hypersensitivity reaction immediate typeC. Hypersensitivity reaction slow typeD. Granulomatous inflammationE. Interstitial inflammation14. A 3-year-old child died of an acute destructive staphylococcal pneumonia. Macroscopically, thymus mass reduced to 3,0 gr. Microscopical investigation of thymus showed reduction of lobules, considerable decline of lymphocytes quantity, stromal collapse, and inversion of layers with cystic enlargement of the HassaH's bodies. What is the most likely diagnosis?A. *Accidental thymus involutionB. ThymomegalyC. Thymus hypoplasiaD. Thymus dysplasiaE. Thymus agnenesia15. The experiment animal with a history of previous sensibilization received the next subcutaneous dose of antigen. In the place of injection a fibrinous inflammation with alteration of vessels walls, matrix and fiber structures of connective tissue (mucoid and fibrinoid swelling) and fibrinoid necrosis developed. What immu-nological reaction takes place?A. * Hypersensitivity reaction immediate typeB. Hypersensitivity reaction slow typeC. Reaction of transplantation immunityD. Normergic reactionE. Granulomatosis16. A man with a history of bronchial asthma died of asphyxia. Microscopical investigation of lungs revealed excess of mucus with lots of eosinophils in the bronchial lumen, sclerosis of interalveolar septas and alveoli dilatation. What mechanism of hypersensitivity reaction presented in that case?A. *Reagin reactionB. Cytotoxic reactionC. Immune complex reactionD. Cytolysis, conditioned by lymphocytesE. Granulomatosis17. The infiltration anesthesia by ultracain with adrenaline solution has been done to a young man. Then redness, edema of the skin with itchy blisters suddenly developed. What type of hypersensitivity took place?A. * AnaphylaxisB. CytotoxicityC. Immune complex damageD. Hypersensitivity slow typeE. Granulomatosis18. A 33-year-old patient with a history of acute tonsillitis presented with the urinary syndrome (haematuria, proteinuria and leukocyturia). Kidneys biopsy revealed an intracapillary proliferative glomerulonephritis. Electronic microscopy determined large subepithelial deposits. What is the pathogenesis of this disease? A.*Immune complex mechanism

Page 55: Pat an Anew

55

B. AtopyC. Cytotoxic, cytolytic action of antibodiesD. Cellularally caused cytolysisE. Granulomatosis19. A 10-month-old child has developed pneumonia. Clinical and laboratory tests revealed absence of mature B-lymphocytes in peripheral blood and in B-zones of lymph nodes, acute reduction of immunogiobulins in serum. In peripheral blood a total quantity of lymphocytes was normal. What kind of an immunodeficiency developed in that case?A. *Brutone sindromeB. Neselof syndromeC. Severe combined immunodeficiencyD. Di-George syndromeE. Viskotta-Oldrich syndrome20. A patient with a history of serious radioactive irradiation had bone marrow transplantation. In 2 months a rush on the skin and diarrhea developed. Clinical and laboratory tests showed hepatic insufficiency, local necrosis of the skin's epithelial cells, intestinal crypts and liver parenchymal cells. What was the reason for symptoms aggravation?A. * "Graft against the owner" diseaseB. Sepsis developmentC. An acute graft rejectionD. A chronic graft rejectionE. An exacerbation of a chronic hepatitis21. A 7-month-old child died of severe pneumonia complicated by sepsis. A postmortem revealed an absence of thymus. In lymph nodes there were no lymphoid follicles and cortical substance, in a spleen follicles were markedly reduced, germinal centers were absent. What is the most likely diagnosis?A. *Thymus agenesiaB. Thymus aplasiaC. Thymus atrophyD. Thymus hypoplasiaE. Acidental thymus involution22. A 1 -year-old child died of a relapsing pneumonia. A post-mortem revealed hypoplasia of a thymus and peripheral lymphoid tissue, atrophy of cerebellum cortex, angiotelectasis of bulbar conjunctiva. Immunohistochemistry revealed decreased level of immunoglobulins. What is the most likely disease?A. * Louis-Bar's syndromeB. Di-George syndromeC. Nezelof syndrome D.West's syndrome E. Brutone syndrome23. A 20-year-old woman died of postnatal sepsis. A post-mortem revealed enlarged congested spleen with abundant curettage from a cut section. Microscopical investigation showed hyperplasia and infiltration by plasmocytes of the red pulp and spleen's follicles. The red pulp was also rich with macrophages. What pathology resulted in spleen's alteration?A. * Antigenic stimulation of an organismB. Hereditary insufficiency of a peripheric lymphoid tissueC. Reaction of hypersensitivity of immediate typeD. Reaction of hypersensitivity of the slowed typeE. Autoimmunization24. Microscopical investigation of arterioles showed plasmatic saturation, mucoid and fibrinoid swelling, fibrmoid necrosis of walls, diffuse infiltration by lymphocytes, plasmocytes and monocytes. In addition, the focal proliferation of hystio-cytes, endotheliocytes and pericytes were determined. The final diagnosis was the "periarteritis nodosa". What type of inflammation took place in arterioles?A. * Acute immune inflammationB. Acute not immune inflammationC. Focal exudative inflammationD. Diffusive exudative inflammationE. Fibrinous inflammation25. During operation of kidney transplantation in 15 minutes after vascular anastomoses application, transplant became flabby, cyanotic and spotty; the pulsation was absent. Microscopical investigation of a kidney specimen revealed periendothelial accumulation of neutrophils in glomeruli vessels. In addition there were also a thrombosis and necrosis of some glomeruli. Specify the type of graft rejection.A. *Hyperacute rejectionB. Accelerated rejectionC. Acute intersticial rejectionD. Acute vascular rejection

Page 56: Pat an Anew

56

E. Chronic rejectionAdaptation and compensation.1. A young male complains of diminishing leg muscles in size and volume as a result of the complicated femoral fracture. The innervations of the muscles was not lost. Name the type of atrophy?A. * Disuse atrophyB. Degeneration atrophyC. Ischemic atrophyD. Atrophy due to compressionE. Atrophy due to chemical and physical influence.2. At autopsy a male is found to have big kidneys with marked dilatation of the renal pelvis and calyces filled with transparent liquid. A thinning of the renal parenchyma accompanied above lesions. Which is the most likely finding?A. *HydronephrosisB. GlomerulonephritisC. AmyloidosisD. TuberculosisE. Pyelonephritis3. An autopsy of an elderly male revealed the atherosclerosis of the brain arteries accompanied with the thinning of the brains cortex. Name the cause of the atrophy?A. *IschemiaB. CompressionC. Chemical and physical factorsD. Loss of innervationsE. Decreased workload4. A 62-yer-old male died at the hospital after developing heart failure. At autopsy the weight of the heart was SOOgm, with thickened right ventricular wall and dilated chamber. Name the alteration at the right ventriculum.A. * HypertrophyB. Hyperplasic alteration due to inflammationC. MetaplasiaD. AtrophyE. Sclerosis5. A young male presents with a large and lacerated wound covered with pus. The peripheral zone of the tissue defect was filled with large amount of granulation tissue. Name the type of healing?A. * Healing by first intentionB. Healing by second intentionC. Healing under a scabD. Covering a tissue defect with epithelial cellsE. Organization followed by scarring6. An autopsy revealed marked enlargement of a kidney. Gross examination of a kidney's section presented with dilation of both renal pelvis and calyces resulted from renal stones. What from diagnoses is most faithful?A. *HydronephrosisB. Simple cystsC. PyelonephritisD. Benign nephrosclerosisE. Polycystic kidney disease7. A physical investigation of an elderly man with history of esophagus cancer with stenosis but without metastasis revealed an atrophy of skeletal muscles and subcutaneous fat tissue. Skin was grayish-brown color, an epidermis was thinned, and size of the heart decreased. Myocardium and liver also had a brown coloring. What is the most likely diagnosis?A. * Alimentary cachexia.B. Myasthenia.C. Cancer cachexyD. Brown atrophy.E. The Addisona Illness8. At the man with long history of smoking the bronchial biopsy is taken. Microscopical investigation showed the thickened mucous membrane with chronic inflammation and transformation of ciliated epithelium into stratified one. What pathology such changes are characteristic for?A. *MetaplasiaB. Нуperplasia of epitheliumC. Planocellular cancerD. LeukoplakiaE. Hypertrophy of epithelium

Page 57: Pat an Anew

57

9. A patient with a history of frequent hemorrhoid bleeding died of acute myocar-dial infarction. A post-mortem revealed a red, succulent bone marrow of a hip diaphysis. What pathology developed in a bone marrow?A. *Compensatory hyperplasiaB. Vicarious hypertrophyC. Hypertrophy excrescencesD. Hormonal hyperplasiaE. Work hypertrophy10. A 59-year-old patient with a history of prostate glandular hyperplasia operated in the urology. An operation revealed a thickening of a urinary bladder wall up to 1 cm. It may be caused by:A. *Work hypertrophyB. Vicarious hypertrophyC. Hormonal hypertrophyD. Hormonal hyperplasiaE. Protective hyperplasia11. A 7-year-old child presented with a poliomyelitis. His somatic muscles are weak, their volume is reduced, and skin is dry and pale. Choose the most likely pathology, which takes place in the soft tissues?A. *Atrophy.B. Hypertrophy.C. Hyperplasia.D. Metaplasia.E. Hypoplasia.12. An elderly man with a long history of smoking presented with a cough accompanied by viscid mucous sputum, weakness after the insignificant physical activity and pale color of the skin. He lost 12.0 kg of weight for the last two months. Microscopical investigation of endoscopic biopsy showed squamous cell carcinoma. What pathology preceded the tumor formation?A. *Metaplasia.B. Hypoplasia.C. Hyperplasia.D. Necrosis.E. Sclerosis.13. A 48-year-old patient with a history of a hypertensive disease died of heart failure. An autopsy revealed an enlarged heart with dilated chambers. A thickness of the left ventricle wall was 2,5cm. Microscopically myocardial cells were considerably enlarged, with fatty dystrophy and hyperchromic barrel-shaped nuclei. What is the most likely pathology diagnosed in a heart?A. *Excentric hypertrophyB. MyocarditisC. Concentric hypertrophyD. Angiogenic cardiosclerosisE. Cardiomyopathy14. A 60-year-old man has a history of chronic bronchitis. Histological investigation of bronchus biopsy showed thinned mucous membrane, cystic transformation of mucous glands, and replacement of prismatic epithelium on stratified one. What of the listed pathological processes the most likely?A. * Metaplasia.B. Hyperplasia.C. Heterotopia.D. Heteroplasia.E. Displasia.15. Microscopical investigation of tissue from the edge of a chronic gastric ulcer showed necrosis, granulation tissue, abundant development of connective tissue and metaplasia of epithelium. What type of pathology takes place in that case?A. *Pathological regeneration with disorder of phases.B. Hypertrophy.C. Physiology regeneration.D. Reparative regeneration (substitution).E. Reparative regeneration (restitution).16. At a patient with a chronic cystitis biopsy of urinary bladder mucosa was taken. Microscopical investigation revealed a transitional epithelium with foci of stratified epithelium without keratinization. What process underlies the described changes in an epithelium?A. *MetaplasiaB. DystrophyC. HyperplasiaD. DysplasiaE. Hyperkeratosis

Page 58: Pat an Anew

58

17. After traumatic damage of a liver, subsequently there was complete hepatic structural and functional restoration. Name the type of regeneration?A. *RestitutionB. Incomplete regenerationC. Pathological regenerationD. Physiological regenerationE. Substitution18. At a patient with chronic bronchitis biopsy of bronchus mucous membrane was taken. Microscopical investigation revealed the areas of stratified epithelium without keratinization. What pathology takes place in the mucous membrane of bronchus?A. *MetaplasiaB. AtrophyC. HypeiplasiaD. Dysplasia E.19. A 42-year-old woman presented to the gynecologist with acyclic, profuse hem-orrhagic discharge. A histological investigation of a curettage material revealed increased quantity of glands and their cystic dilation. What is the most likely diagnosis?A. * Endometrial hyperplasiaB. Endometrial atrophyC. Endometrial hypertrophyD. MetaplasiaE. Organization20. A post-mortem of a 64-year-old woman with a long story of hypertensive disease revealed considerably diminished, dense kidneys with a fine-grained surface. What pathology such changes are characteristic for?A. * Blood insufficiency atrophyB. Atrophy from pressureC. Senile atrophyD. Dysfunctional atrophy E. Hypoplasia21. An elderly man died of cardiac insufficiency. Twenty years ago he had the right lung pulmonectomy because of a cyst. A post-mortem revealed an enlarged left lung. Diagnose pathology in the left lung.A. *Vicarious hypertrophyB. Neurohumoral hypertrophyC. Dysfunctional atrophyD. Dyscirculatory atrophyE. Neurotic atrophy22. A 42-year-old man had a lower extremity amputation. After a while painful knots appeared in a stump. Microscopical investigation revealed amputation neuromas. What pathology such changes are characteristic for?A. *Pathological regenerationB. Complete reparative regenerationC. InflammationD. Incomplete reparative regenerationE. Metaplasia23. A 38-year-old woman presented with complaints about the frequent uterine bleeding. The diagnostic curettage was performed. A histological investigation of a curettage material revealed increased quantity of coiled glands, some of them were with cystic dilation. What pathology these changes are characteristic for?A. *Glandular-cystic hypeiplasia.B. AtrophyC. MetaplasiaD. DysplasiaE. Hypertrophic vegetations24. A man had the right lung pulmonectomy because of a tumor 7 years ago. Then the volume of the left lung increased on 40%. What process developed in the left lung?A. *Vicarious hypertrophyB. Neurohumoral hypertrophyC. False hypertrophyD. Work hypertrophyE. Hypertrophic vegetation25. A histological investigation of an endometrium revealed coiled extended glands with 'saw-' and a 'spin-like' pattern. A strornal proliferation with hypeiplasia of its cells was also determined. What is the most likely diagnosis?A. * Glandular hyperplasia of endometriumB. Acute endometritisC. Leiomyoma

Page 59: Pat an Anew

59

D. Hydatidiform moleE. Placental polyp26. A patient presented with small knots on a thin peduncle in his rectum. Microscopical investigation of the biopsy material revealed a growth of connective tissue and glands. What is the most likely pathology?A. *Hyperplastic polyp.B. Hypertrophy.C. Metaplasia.D. Atrophy.E. Sclerosis.27. The biopsy of central bronchus mucosa of miner is taken. Microscopically, there is a transformation of a cylindrical epithelium to the mature stratified one. What pathology such changes are characteristic for?A.* MetaplasiaB. HyperplasiaC. LeucoplakiaD. DysphasiaE. Reparative regeneration28. The biopsy of a vaginal portion of uterus cervix is taken. There is an augmentation of quantity of cells, rising of mitotic activity and alteration of polarity of cells in basal part of an epithelial layer. What pathology such changes are characteristic for?A.* DysphasiaB. MetaplasiaC. AtrophyD. AcantosisE. Hyperplasia29. A 40-year-old man has a keloid scar after a treatment of the left arm burn. This process may be an example of:A. *Pathologic regenerationB. HyperplasiaC. MetaplasiaD. Reparative regenerationE. Vicarious hypertrophyEpithelium neoplasms1. A teenager male presents with slowly enlarging, painless nodule on his right hand's skin. Microscopic examination of removed lesion revealed an increased number of epithelial layers, with a stroma underneath, with developed a papilloma-tous pattern. Identify most likely type of atypia?A. * TissueB. Cellular.C. Metabolic.D. Functional.E. Hystochemical2. A 35 year-old female with a family history of colon cancer inquires about screening. Colonoscopy revealed a tumor and a tissue sample was taken for histology. Microscopic investigation has shown cells were arranged in glandular-like pattern. They exhibit cell pleomorphism, atypia, invasive growth and pathologic mitosis's. What is the most likely diagnose?A. *AdenocarcinomaB. Basal-cell carcinomaC. Solid carcinomaD. Mucous carcinomaE. Undifferentiated carcinoma3. A 50-year-old female is found to have a urinary bladder neoplasm. Biopsy investigation revealed that tumor consists of thin, branching out papillae, covered with few layers of transitional cell epithelium. What is the most likely diagnose?A. *PapillomaB. Basal cellC. Transitional cell carcinomaD. Squamous cell carcinomaE. Fibroadenoma4. A 60-year-old postmenopausal woman has been feel unhealthy and weak for about 3 month. A gross investigation of her cervix uteri revealed a lesion and a biopsy from this area was obtained. Microscopically, a lesion composed of atypical squamous cell, many of them displayed pathological mitoses. Keratin pearl formation was also observed histologically. What is the most likely diagnose?A. * Squamous cells carcinoma with keratin і sationB. Transitional cell carcinomaC. Squamous cells without keratinisation

Page 60: Pat an Anew

60

D. AdenocarcinomaE. Anaplastic carcinoma5. A 48-year-old woman complains to her doctor of uterine bleeding. Endometrial biopsy investigation revealed a presence of gland-forming cells with enlarged hyperchromic nuclei, cell atypia, abnormal mitoses. These cells were also determined at myometrium. What term most correctly identifies this pathological process?A. * Adenocarcinoma of uterus.B. Adenomatous endometrial hyperplasia.C. Acute endometritis.D. Placental polyp.E. Chorionepitelioma of uterus.6. A 39-year-old woman presented in oncology with a stomach cancer. Physical investigation revealed metastatic tumors in ovaries (Krukenberg tumors). What is most likely pattern of spread in that case?A. * Lymphatic retrogradeB. Lymphatic orthogradeC. GematogenousD. ImplantationE. Along epithelium-lined surfaces7. A 23-year-old female decided to have a surgeon's removal of a small nodule at her leg's skin. An operation material delivered to pathology department. A histological examination of a new growth has shown that parenchyma has been formed of integumentary epithelium with an increased amount of layers. Strorna together with epithelial proliferation forms papillae. What is the most likely form of atypism?A. *Tissue.B. Cellular.C. Histochemical.D. Functional.E. Metabolic.8. A 45-year old male with a family history of a gastric cancer inquires about screening. A gastroscopic examination of a patient revealed a pedunculated tumor mass 1.5 cm in diameter in the area of the lesser curvature of stomach. What kind of growth does the tumor have?A. *Exophytic.B. Expansive.C. Infiltrating.D. Appositional.E. Endophytic.9. A 70 year-old male is found to have a nodule at his right bronchus mucous membrane. A microscopic examination of the biopsy material from the tumor revealed the cell and tissue atypism, keratin pearls formation. What is the most likely pathologic process at biopsy material?A. *Malignant tumor.B. Benign tumor.C. Hyperplasia.D. Metaplasia.E. Hypoplasia.10. A 57-year-old postmenopausal woman is found to have a chronic nonhealing lesion at her portio vaginalis uteri.A biopsy was taken form this area. A histological examination of the biopsy material revealed a cellular atypism within epithelial layer, but basic membrane was unchanged. What is the most likely diagnosis?A. *Carcinoma in situ.B. Erosion.C. Adenocarcinoma.D. Papillorna.E. Endometriosis.11. A 15-year-old boy is found to have a small, dense, nodule at his right forearm skin. Grossly it had a papillary surface, which looked like a cauliflower. Microscopically, the tumor consists of many papillae. Parenchyma formed of integumentary epithelium with an increased amount of layers. The polarity of epithelial cells, their stratification and membrane wholeness are preserved. A connective tissue forms a stroma within a center of papillae. What is the most likely diagnosis?A. *Papilloma.B. Fibroma.C. Adenoma.D. Fibroadenoma.E. Cystadenoma.12. A 66-year old male lost his appetite and has been loosing his weight for about 5 months. X-ray study revealed a stomach neoplasm. Histological examination of a tumor's biopsy showed a great amount of signet-ring cells. Name the histological variant of cancer.

Page 61: Pat an Anew

61

A. * Adenocarcinoma.B. Solid carcinoma.C. Sarcoma.D. Mucinous carcinoma. E..Carcinoid.13. A 45-year-old woman has a small tumor of her left breast. Histological examination of a breast biopsy revealed poorly differentiated atypical epithelial cells. They formed trabecules separated from one another by connective tissue. The cells arid stroma proportion was approximately 1:1. What is the most likely histological variant of the cancer.A. * Solid carcinoma.B. Adenocarcinoma.C. Epidermoid carcinoma.D. Sccirrhous fmrocarcinoma.E. Small cell carcinoma.14. Histological investigation of a node in the removed mammary gland revealed complexes of atypical polymorphic epithelial cells, which had various sizes and forms. There were clear spaces at the centers of complexes. The cells had large nuclei, with presence of atypical mitoses. Diagnose the pathology,A. *AdenocarcinomaB. Squamous cell nonkeratinous carcinomaC. Solid carcinomaD. Fibroadenoma of breastE. Nondifferentiated polymorphocellular carcinoma15. A microscopical investigation of a tumour revealed signet-ring cells. Name a histological variant of a cancer.A. *Mucinous carcinomaB. AdenocarcinomaC. SarcomaD. Solid carcinomaE. Carcinoid16. A 34-year-old woman presented with a hoarseness of a voice, A laryngoscopy revealed a tumour of a larynx. The neoplasm had a grey-white color and papillary surface. Microscopical investigation showed a new growth of a connecting tissue, covered by a stratified epithelium with the marked keratinization. Cellular atypia was absent. Most likely the tumor is:A. *PapillomaB. FibromaC. PolypD. AngiomaE. Angiofibroma17. A microscopical investigation of a 50-year-old woman's endometrial curettage material with the clinical diagnosis « ovarian - menstrual cycle's disorder » revealed growth of glandular structures. These glands consisted of polymorphic cells with hypercliromic nuclei and mitoses figures. For what pathology the revealed histological changes are characteristic?A. *Adenocarcinoma of uterusB. Placental polypsC. Acute endometritisD. Glandular hyperplasia of endometriumE. Chorioepithelionia of uterus18. A 52-year-old female patient with a history of the chronic bronchitis and pneu-mosclerosis presented to the hospital for biopsy diagnostics. A microscopical investigation of a left bronchial's mucous from the suspicious site revealed cellular and tissue atypia. There were also found some structures in the form of cancer pearls". What is the most likely pathology presented in that case?A. * Squamous cell keratinous carcinoma of bronchusB. Chronic polypous bronchitisC. BronchiectasisD. Sharp bronchitisE. Squamous cell metaplasia of mucous of bronchus19. Histological investigation of a bronchial biopsy revealed a tumour which is constructed from nests of stratified epithelium's atypical cells with some characteristic "pearls". What is the most likely diagnosis?A. *Squamous cell keratinous carcinomaB. Squamous cell nonkeratinous carcinomaC. Solid carcinomaD. Mucous carcinomaE. Scirrhous carcinoma

Page 62: Pat an Anew

62

20. A microscopical investigation of a breast tumour revealed that the neoplasm was constructed of undifferentiated atypical epithelial cells. These cells formed trabecules, separated by layers of a connective tissue. A parity of cells and stroma was approximately 1:1. Name a histological variant of a cancer.A. *Solid carcinomaB. AdenocarcinomaC. Squamous cell carcinomaD. FibrocarcinomaE. Small cell carcinoma21. Histological investigation of the removed breast node revealed different sizes and the form complexes of atypical polymorphic epithelial cells among abundant stroma. The complexes had a clear space center. Cells were characterized by large nuclei, the increased number of nucleoli, nucleoli organizers and presence of atypical mitoses. What is the most likely diagnosis?A. *AdenocarcinomaB. Fibroadenoma of breastC. Solid carcinomaD. Squamous cell nonkeratinous carcinomaE. Nondifferentiated polymorphic cells carcinoma22. A post-mortem of a 59-year-old man, who died of a lung cancer, revealed plural metastases. What kind from the listed below metastases it is possible to regard as implantation (contact) one according to a mechanism of development?A. *Multiple tumorous nodules of pleuraB. Metastasis in prebronchial, paratracheal lymphatic knotsC. Metastasis in a brainD. Metastases in an adrenal glandE. Invasion of tumor from bronchus in an esophagus23. A 55-year-old man presented to a hospital with the atelectasis of a right lung's middle lobe, resulted from the obturation of a midlobar bronchus by a node of soft tissues. A bronchoscopy revealed a new growth within an obturation zone. A microscopical investigation of a biopsy sample showed the growths of an atypical glandular epithelium with pathological mitoses, which spread in tissues underneath and a cartilage. What is the most likely disease?A. *Bronchogenic carcinoma of lungsB. Dysplasia of epithelium of bronchusC. Inflammatory polypsD. Deforming bronchitisE. Sarcoma of bronchus24. A 65-year-old woman presented to the hospital with the menopausal bleeding. At histological investigation revealed in the curettage material from the mucosa of her cervix uteri revealed a new growth of atypical epithelium with formation so-called " cancer pearls ". What is the most likely diagnosis?A. *Squamous cell keratinous carcinomaB. AdenocarcinomaС Squamous cell nonkeratinous carcinomaD. Mucous cancerE. Nondifferentiated carcinoma25. A post-mortern of a 48-year-old woman with a history of an operated stomach tumour in the past revealed markedly enlarged, dense, whitish color ovary. Histo-logical investigation of the ovarian tissue showedutterly atypical epithelial cells, placed among layers and cords of a connective tissue. What if the most likely disease?A. *Krukenberg's carcinoma of ovaryB. Serous cystadenocarcinomaC. Pseudomucinous cystcarcinomaD. Malignant thecomaE. Malignant granular cell tumor26. A histological investigation of a lung biopsy revealed atypical cells which form plural acinar structures and produce mucus. What histological form of a cancer of lungs takes place at the patient?A. *High differentiated adenocarcinomaB. Low differentiated adenocarcinomaC. Nondifferentiated carcinomaD. DifferentiatedE. Glandular squamous cell carcinoma27. A gastroscopy of a 37-year-old man revealed a tumorous formation 1,5 cm in diameter in a small curvature of his stomach. What character of a tumor growth?A. *ExophyticB. ExpansiveC. Invasive

Page 63: Pat an Anew

63

D. InfiltratingE. Endophytic28. Histological investigation of a biopsy from a tumor of the right bronchus' mucous membrane revealed a cellular and tissue atypia, appearance of structures in the form of cancer pearls '. Define the pathological process.A. *Malignant tumorB. Benign tumorC. HyperplasiaD. MetaplasiaE. Hypoplasia29. A histological investigation of the biopsy from cervix uteri of a 45-year-old woman revealed signs of cellular atypia with intact basal membrane. What is the most likely diagnosis?A. * Carcinoma in situB. ErosionC. AdenocarcinomaD. PapillomaE. Endometriosis30. A physical examination of a 42-year-old patient revealed enlarged supraclavi-cular lymph nodes. A histological investigation of a lymph node biopsy showed the metastasis of a signet-ring cancer. Choose the most probable localization of a primary tumour.A. *Carcinoma of stomachB. Cancer of esophagusC. Cancer of thyroid glandD. Carcinoma of lungsE. Carcinoma of uterine cervix31. A female patient in a climacteric period presented with relapsing uterine bleedings. The diagnostic curettage of a uterus is executed. Microscopical investigation showed among blood glandular elements of different size and forms created by atypical cells with hyperchromatic nuclei numerous mitoses (including pathological). What is the most likely pathology?A. *AdenocarcinomaB. Glandular hyperplasia of endometriumC. ChoriocarcinomaD. Adenomatous polypusE. Signs of the interrupted pregnancy32. A bronchoscopy of a 65-year-old patient revealed a polyp-like formation, 1,0 cm in diameter, in the proximal part of the upper lobe bronchus of his right lung. A histological research revealed the tumour which consisted from fine lymphocyte-like cells with hyperchromic nuclei. The cells grew like layers and cords. Specify, what of the listed below kinds of tumors is the most authentic?A. *Nondifferentiated small cell carcinomaB. Nondifferentiated large cell carcinomaC. Squamous cell carcinomaD. AdenocarcinomaE Glandular squamous cell carcinoma33. A gastroscopic study of a 50-year-old patient revealed a crater-like lesion on small curvature in pre-pyloric zone of stomach. From a regional site of formation a biopsy is taken, биопсия. A histological investigation showed a tumor with glands-like structures of the various form and the sizes, growing into surrounding tissue. The cells had marked signs pf atypia. Name a histological variant of the presented tumor.A. * AdenocarcinomaB. Squamous cell carcinomaC. Scirrhous carcinoma of stomachD. Mucous carcinoma of stomachE. Solid carcinoma of stomach34. A patient presented to her physician with gradually developed plaque on a skin of a cheek. The formation had necrosis and an ulcer in the center. A histological investigation of a skin biopsy revealed a growth of atypical epithelial cells with a lot of pathological mitoses. What is the most likely diagnosis?A. *Carcinoma of skinB. SarcomaC. PapillomaD. Trophic ulcerE. Fibroma35. A 48- year-old man with a history of a chronic bronchitis has died of a cachexy. A post-mortem revealed in a lumen of the right bronchus an endophytic growth of a light- grey softish tissue. Microscopic investigation showed a development of an atypical stratified epithelium with a presence of an « epithelial [epidermic] pearl, pearly body ». What is the most likely diagnose.

Page 64: Pat an Anew

64

A. *Squarnous cell keratinous carcinomaB. Squamous cell nonkeratinous carcinomaC. AdenocarcinomaD. Nondifferentiated carcinomaE. Apudoma36. An autopsy of a 50-year-old man, who died of cancer intoxication, revealed the thickening of a stomach's wall to 1, 2 cm. The mucosa was fixed, with ill-defined gastric folds. On a cut view a tissue was homogeneous, whitish with chondroid density. For what macroscopical form of a tumour the described changes are characteristic?A. * InfiltrateB. NodeC. UlcerD. Ulcerous-infiltrationE. Cyst37. A histological investigation of a breast tumor's biopsy revealed solid layers of fine epithelial cells with polymorphic nuclei and a considerable quantity of pathological mitoses. In addition, the tumor had very little stroma and a lymphocytic infiltration. Name the kind of presented tumor.A. *Medullary carcinomaB. Scirrhous carcinomaC. Paget's diseaseD. AdenomaE. Adenofibroma38. A mucus membrane biopsy is taken from a bronchus of a 52-year-old patient. A histological investigation revealed cords of atypical epithelial cells, which grew into the tissues underneath. In the cord's center a concentric pink color fonnations were determined ("cancer pearls", "epithelial [epidermic] pearl, pearly body"). Name a kind of a tumor?A. *Squamous cell keratinous carcinomaB. Differentiated adenocarcinomaC. Squamous cell nonkeratinous carcinomaD. MelanomaE. Transitional cell carcinomaTumours derived from a mixture of tissues1. A 55-year-old female is found to have asymptomatic microscopic hematuria. A plain radiograph of the abdomen revealed a neoplasm at her right kidney. A gross investigation of removed kidney revealed a node 8cm in diameter at its upper pole. The cut surface of a tumor presented with multiple hemorrhages and necroses. Histologically, it consists of light cells that form alveolar .and papillary structures. The invasive growth of the tumor is moderate. Many cells have hyperchromic nuclei and atypical mitoses figues. What is the most likely diagnosis?A. *Renal cell carcinoma.B. Clear cell adenoma.C. Adenocarcinoma.D. Nephroblastoma.E. Acidophilic adenoma with malignancy.2. A 60-year-old man noticed worsening urinary tract symptoms and thus inquires about screening tests for kidney pathology. Investigation revealed a node 8 cm in diameter at right kidney's apex. After surgery, grossly tumor presented a patchy pattern with hemorrhages and zones of necrosis. Histological study showed large anaplastic tumor cells with abundant foamy cytoplasm and with small central densely staining nucleus. Abnormal mitoses were also found. What is the most likely diagnose?A. * Clear -cell carcinomaB. Clear -cell adenomaC. AdenocarcinomaD. NephroblastomaE. Acidophilic adenoma with malignization3. A 45-yer-old female examined by her gynecologist due to uterine bleeding. A neoplasm was diagnosed and then removed at surgery department. The uterine lesion presented with huge necrotic and hemorrhagic spongy masses. Microscopic investigation revealed large clear atypical epithelial cells and variety of abnormal dark cells, arranged around material blood spaces. Stroma was not defined. Atypical cells resembled cytotrophoblast and syncytiotrophoblast cells. What is the most likely diagnose?A. *ChoriocarcinomaB. Invasive hydatidiform moleC. AdenocarcinomaD. Cavernous haemangioma.E. Medullary carcinoma4. A 32-year-old woman with a history of abortion 8 months earlier presented to the emergency department complaining of uterine bleeding. Examination done by gynecologist revealed a neoplasm of the uterus. Grossly,

Page 65: Pat an Anew

65

tumor had a spongy structure with multiple hemorrhages. Microscopically, atypical clear epithelial Langhan's cells and abnormal syncytiotrophoblast cells arranged around maternal blood spaces were detected. What is the most likely diagnosis?A. *ChoriocarcinomaB. Squamous cell carcinoma without keratinisationC. AdenocarcinomaD. FibromyomaE. Hydatidiform mole5. A 45-year-old woman presented to oncology with a breast tumor. A biopsy revealed a tissue atypia with stromal predominance over neoplastic parenchyma. Breast ducts and ductules were variable, they had one or two layered epithelium without atypical mitotic activity. The intralobular stroma was dence. What is the most likely diagnosis?A. *FibroadenomaB. PapillomaC. Non-invasive carcinomaD. Invasive carcinomaE. Mastitis6. The patient on a face skin had a tumorous formation plaque-like form with an ulcer. What is the most likely diagnosis?A. * Basal cell epitheliomaB. CaremoidC. ThecomaD. Pinealoma C. Thymoma7. A physical examination of a 22-year-old woman, with few years' history of rny-asthenia, revealed a big tumor at anterior mediastinum. Histologically a tumor consisted of the oblong (spindle-shaped) cells with oval dark nuclei, which form bands and nests and has little Hassall's bodies. After a surgical removal of a tumor symptoms of a myasthenia have started to disappear progressively. Diagnose a tumor which has etioiogical and pathogenic connection with a myasthenia.A. * ThymomaB. Adenoma of thyroid glandC. Adenoma of parathyroid glandD. ParagangliomaE. Medulloblastoma8. A 48-year-old man has presented to his physician a plaque-like formation on a neck. Histological investigation of a skin biopsy revealed tumorous ceils located as nests, having round and oval form with narrow rim of basophilic cytoplasm. They reminded cells of a skin basal layer. Specify the tumor name.A. * Basal cell epitheliomaB. Epidermal cancerC. HidroadenomaD. TrichoepitheliomaE. Syringadenoma9. A physical investigation of a 25-year-old-woman revealed in her breast a dense node 1,0 cm in diameter. A biopsy research showed encapsulated growth of a connective tissue round a basal membrane of a mammary gland ducts. Glandular elements had different diameter, did not form lobes. What is the most likely diagnosis?A. *FibroadenamaB. FibromaC. Metastasis of a cancerD. AdenomaE. Fibrous cancer10. A physical examination of a 39-year-old woman revealed a soaking area by her breast nipple, a superficial ulcer with inflammatory hyperemia and skin edema. A histological research of a biopsy from this area revealed in basal layer of thickened epidermis atypical big cells with light and optically empty cytoplasm, with absence of intercellular bridges. Such cells are found and in the ostium of the big ducts of a gland. What is the most likely diagnosis?A. *Paget's deseaseB. Intraductal cancerC. Basal cell cancerD. Epidermoid carcinomaE. MelanomaMesenchymal (connective tissue) tumors1. A 30-year-old man has a node on his left leg the skin. Physical investigation revealed dense, mobile tumor, circumscribed by a connective tissue capsule. Grossly it cut section was presented with dense witish fibres. Microscopically the tumor composed of mature fibroblasts and a collagenous stroma. What is the most likely diagnosis?

Page 66: Pat an Anew

66

A. * FibromaB. MyomaC. HistiocytomaD. DermatofibromaE. Desmoid.2. A 14x6x5 cm neoplasm excised from a retroperitoneurn of a 66-year-old woman at surgery department. Microscopic investigation revealed atypical anaplastic cells, which contained round cytoplasmic vacuoles of lipid that scallop the nucleus. The majority of cells were pleomorphic, some of them were round with chromosomal abnormalities. What is the most likely diagnosis?A. *LiposarcomaB. LipomaC. MyosarcomaD. FibrosarcomaE. Mesothelioma3. A physical examination of 47-year-old woman of gynecology department revealed that her uterus contained dicrete, firm, white nodules. Histological examination excised lesion demonstrated a tissue atypia of a sample. It presented well-defferentiated mature cells of smooth muscules. What is the most likely diagnosis?A. *Leiomyoma.B. Carcinoma of the uterus.C. Fibromyoma.D. Chorionepithelioma.E. Leiomyosarcoma4. A 20-year-old man has had a slowly growing reddish nodule on his upper lip. He finally decides to have a surgeon remove it. Microscopically the nodule is composed of benign varying sized tiny blood vessels. What is the most likely diagnosis?A. * Capillary hemangioma.B. Venous hemangioma.C. Cavernous hemangioma.D. Hemangiopericytoma.E. Glomus-angioma.5. The patient, a previously dealthy man of 25 years, presented with a painless neoplasm in a soft tissues of his left thigh. Grossly, a tumor had uneven boundaries and on cut section it looked like a fish flesh. Microscopic investigation revealed immature connective tissue cells with pleomorphism, numerous mitotic figures and lym-phocyte infiltration at the edge of the tumor. What is the most likely diagnosis?A. *FibrosarcomaB. MyosarcomaC. FibromaD. CarcinomaE. Myoma6. A 40-year-old woman presented with a very slowly enlarging subcutaneous mass at the right side of the chest wall. Physical examination revealed a soft lobulated fluctuant swelling, not attached to the skin or underlying muscle. Histologically a neoplasm was well-encapsulated and consisted of mature cells with clear cytoplasm that varied considerably in size. What is the most likely diagnosis?A. *Lipoma.B. Fibroma.C. Hygroma.D. Papilloma.E. Hemangioma.7. A macroscopical investigation of operatively removed uterus revealed a tumour with a soft consistence, hemorrhages and necroses. The tumor cut surface reminds ' the fish meat '. Histological research has found an expressed cellular and tissue atypia; there were cells with pathological mitoses figures. What is the most likely diagnosis?A. Sarcoma.В Adenocarcinoma.C. Angioma.D. Fibroma.E. Lipoma.8. At the young man in a skin depth the dense, mobile tumour, is defined. A microscopical research revealed chaotically located fascicles of collagen fibers with a small amount of spindle-shaped cells. What tumour is removed?A. *Dense fibroma.B. Leiomyoma.C. Melanoma.

Page 67: Pat an Anew

67

D. Lipoma.E. Glomus-angioma.9-А 4-year-old child presented with a flat red color knot on his neck skin, which turns pale at pressing by glass the knot. What is the most probable diagnosis?A. HemangiomaB. Pigmented nevusC. MelanomaD. LeiomyomaE. Lymphangioma10. A 28-year-old man with a history of an elbow bruise 3 years ago presented with a tumorous growth in the area of an epiphysis of a humeral bone. The formation did not have accurate borders. A histological investigation of biopsy material revealed a considerable quantity of polymorphic cells of osteoblastic type with numerous pathological mitoses. Make the presumable diagnosis.A. *OsteosarcomaB. ChondrosarcomaC. Osteoid-osteomaD. FibrosarcomaE. Sinovial sarcoma11. A 48-year-old man presented to physician with mobile 1,0x0,7 cm formation under a skin of mandible. It had precise borders, dough-like consistence and slow growth. A histologic research of formation revealed fat tissue cells (lipocytes), which created lobules of different forms and the sizes, divided by thin layers of a connective tissue with vessels. What is the most likely diagnosis?A. *LipomaB. FibromaC. AngiomaD. LiposarcomaE. Fibrosarcoma12. A 33-year-old woman presented with a tumor-like formation on a white line of her abdomen, which during pregnancy has started to increase in sizes. A histological research revealed that a tumor is constructed of the differentiated connective tissue, in which collagen fibers prevail of cells. What tumor presented in this case? A. * DesmoidB. Dense fibromaC. FibrosarcomaD. Soft fibromaE. Dermatofibroma13. A 15-year-old young man presented with a tumorous formation in the central site of his wrist bone. The node grew slowly within last 3 years. A histologica! research of a removed neoplasm revealed mature chondrocytes without mitoses, which randomly located in chondral lacunas. Cartilages of a capsule had different form and the sizes due to variable quantity of chondral cells, between which there were basic substance with liquid layers of a connective tissue. What is the most likely diagnosis?A. *ChondromaB. ChondroblastomaC. ChondrosarcomaD. Teratoma E.14. A 50-year-old patient presented to the doctor with a ball-shaped, dense, motionless neoplasm, 2 cm in diameter, under a skin in the right parietal site of his head. A histological research of a removed neoplasm revealed a chaotic osteal beams pattern with a connective tissue between. What is the most likely diagnosis?A. *Cancellous osteoma (Osteoma spongiosum)B. Compact osteoma (Osteoma durum)C. OsteoporosisD. OsteomalaciaE. Osteosarcoma15. At the young woman in the area of her distal extremity of a femur the tumor, which quickly grew, is removed. Grossly, it had a motley pattern - from white-sulphur to brown-red color and a quaggy consistence. Microscopical investigation revealed the basic tissue component of a tumor presented with osteal and the ossi-form structures covered by atypical osteoblasts, with numerous thin-walled vessels and atypical mitoses figures. Make a diagnosis.A. *OsteocarcomaB. ChondromaC. OsteomaD. Саркома ЮингаE. Angiosarcoma16. A 16-year-old child presented with a painful sottish node in his femur diaphy-sis. The formation grows quickly with destruction of a spongy layer of a bone. Microscopical investigation revealed monornorphic round cells little

Page 68: Pat an Anew

68

bit bigger than mature lymphocytes, with jejunely light cytoplasm which contains glycogen. In some zones these cells form pseudo- rosettes with few mitoses. Between cells there are fibrinous membranes. What is the most likely diagnosis?A. *Ewing's sarcomaB. LimphomaC. RhabdomyosarcomaD. NeuroblastomaE. HemangiomaNeoplasms of the nervous system (including meninges) and melanin producing tissues1. An eye of 53-year-old patient, excised at surgery due to neoplasm, presented in pathology department. Gross investigation reveled a 1 *0,4 cm black lesion in the retina. Microscopic appearance of a tumor was characterized by nests of immature cell with eccentric nuclei, prominent macronucleoli and cytoplasm brown pigment. What is the most likely diagnosis?A. *MelanomaB. NeurinomaC. AngiosarcomaD. NeuroblastomaE. Ganglioneuroblastoma2. At autopsy a 8-year-old child is found to have a. poorly circumscribed tumor of cerebellum. Histologically investigation reveled crowds of small immature cells with hyperchromatic, round-oval nuclei and scant cytoplasm. A few rosettes were also found by pathologist at slide examination. What is the most likely diagnosis in that case?A. *Medullob!astomaB. AstrocytomaC. metastasis of cancerD. metastasis of sarcomaE. Glioblastoma3. An elderly man with a 2-year history of right leg amputation presents with 2 cm encapsulated tumor at the area of a past surgical trauma of soft tissue. Histology investigation revealed disordered orientation of mature nerve fiber bundles inter mixed with connective tissue. Higher magnification showed bundles of axons, Schwann cells, fibroblasts and perineuria! cells within tumor mass. What is most likely diagnosis?A. NeurinomaB. NeurofibromaC. Malignant neurinomaD. Soft fibromaE. Fibrosarcoma4. A 66-year-old woman present with right eye bad vision, ophtalmoscopy revealed a neoplasm of retina which was soon excised, at surgery together with an eye ball. Grossly, a neoplasm was soft, irregular in contour 1x1 cm in size and had a brown coloring. Under microscope a lesion demonstrated nodular aggregates of infiltrating cells. There cells contained large brown pigment nuclei with chromatin clumped at the periphery of nuclear membrane and prominent nucleoli. Atypical mitoses figures were also revealed. What is most likely diagnosis?A. *MelanomaB. SchwannomaC. Glomus tumorD. ParagangliomaE. Angiosarcoma5. A 38-year-old woman has a seizure while shopping and is taken to the hospital. A scan of a brain demonstrated a poorly circumscribed 5 cm tumor at right parietal lobe. A biopsy of this area contains of increased number of glial cell nuclei and an interening feltwork of file cell processes that give the background a fibril lary appearance. Abnormal mitoses and atypical central nervous system cells were absent within biopsy sample. What is the most likely diagnosis?A. AstrocytomaB. OligodendrogliomaC. GanglioneuromaD. EpendimomaE. Chorioid papilloma6. A 50-year-old woman presented with a pigmented painful skin lesion above the ankle. It had been present for many years but in recent months it had enlarged quite rapidly, its outline got irregularity. Microscopic investigation of a lesion biopsy revealed nests of atypical cells and single cells with eccentric nuclei, prominent macronucleoli and cytoplasmic brown pigment. What is the most likely diagnosis?A. * MelanomaB. Basal cell carcinomaC. Hemangioma.D. Haematoma.

Page 69: Pat an Anew

69

E. Carcinoid.7. The man of 45 years has completely lost hearing on the right ear. Physical investigation has not revealed any pathological changes in system of the right acoustic analyzer. The tomography of a brain has revealed a neoplasm, 7 cm in diameter, homogeneous, without precise contours in a site of cerebellopontine angle. Name a neoplasm.A. *Neurilemmoma B.GangliocytomaC. AstrocytomaD. Chorioidal papillomaE. Neuroblastoma8. The tumor of a brain is diagnosed for the patient with neurologic disturbances. During operation the tumor, which looks like the dense node bound to a firm cerebral membrane, is removed. Histological investigation revealed the neoplasm constructed from endothelium-like cells closely adjoining to each other. What is the most likely diagnosis?A. *MeningiomaВ. GlioblastomaC. Anaplastic meningeomaD. AstrocytomaE. Neuroblastoma9. The tumour of a brain is diagnosed for the patient with quickly increasing intrac-ranial hypertension. The removed tumor of a parietal -temporal part of brain had a soft consistence and a motley pattern of a cut surface. Histological investigation revealed neoplastic tissue constructed of polymorphic cells with sites of necroses and a hemorrhage. What is the most likely diagnosis?A. * GlioblastomaB. OligodendrogliomaC. NeuroblastomaD. AstrocytomaE. Meningioma10. The woman on her face skin had a pigmental formation in the form of a nodule which quickly grew. The biopsy is made. Microscopical investigation of biopsy sample revealed fields of spindle-shaped and polymorphic cells which contain a brown pigment. In addition, there were diagnosed numerous mitoses. What is the most likely diagnosis?A. *MelanomaB. NevusC. CancerD. PapillomaE. Dermatofibroma11. A physical investigation of a 60-year-old woman with one year history of a formation on a face revealed a brown skin plaque with irregular form and black impregnations. A histological research of a skin biopsy showed in epidermis and through all derma polymorphic big cells with pathological mitoses, large nucleoli and yellowy-brown pigment in a cytoplasm of many cells. The specified cells grow in a kind of fine groups and also alone. Make a diagnosis.A. * MelanomaB. NevusC. PapillomaD. XerodermaE. MelanosisDiseases of haemopoetic and allied systems1. A 34- year-old male is found to have weakness confusion, other neurologic manifestation and polyuria X-ray examination revealed multifocal destructive bone lesions throughout the skeletal system. Electrophoretic analysis revealed increased level of Ig in the blood and Bence Jones protein in the urine. What is the most likely diagnosis?A.*MyelomaB. Acute monocytic leukemiaC. Chronic myeloleukemiaD. LymphogranulomatosisE. Histiocytosis2. A 42-year-old female is admitted to the hospital for treatment a blood disorder. A disease complicated with pneumonia led to lethal outcome. An autopsy revealed hyperplasia of a bone marrow with "pus" - like appearance, splenomegaly (5 kg weight), hepatomegaly (6 kg weight), systemic lymph nodes enlargement. What is the most likely diagnosis?A* Chronic myeloleukemiaB. Chronic lymphatic leukemiaC. MyelomaD. Polycytemia vera (erythremia, Osier's disease)

Page 70: Pat an Anew

70

E. Lymphogranulomatosis3. An elderly male is admitted to the hospital for treatment of humeral bone fracture. His fracture's area X-ray showed a new growth and lytic zone within lesion. Histological examination of a biopsy revealed abnormal plasma cells. What is the most likely diagnosis?A. * MyelomaB. Chronic osteomyelitisC. Chondrosarcoma.D. Bone fibrous dysplasiaE. Metastasis of adenocarcinoma4. At autopsy an elderly female is found to have enlarged groups of a neck, axillary and mediastinal lymph nodes matted together. They were firm and rubbery. The cut surface was gray-white, producing a "fish-flesh" appearance. Microscopy revealed heterogeneous cellular infiltrate wich contained lymphocytes, classic and mononuclear Reed-Berezovsky-Sternberg cells. What is the most likely diagnosis?A. * LymphogranulomatosisB. Chronic lymphatic leukemiaC. LymphosarcomaD. RetikulosarcomaE. Mycosis fungoides5. A 67-year-old female presented with painless enlargement of lymph nodes. Histological examination of a biopsy sample revealed that the whole lymph node pattern was unclear, with heterogeneous cellular infiltrate. It included abnormal, immature cells admixed with lymphocytes, eosinophils, plasma cells and macro-phages. Binucleated Berezovsky-Reed-Sternberg cells, where surrounded by multiple cell types. What is the most likely diagnosis?A. * LymphogranulomatosisB. Acute myeloleukemiaC. Chronic myeloleukemiaD. Mycosis fungoidesE. Tuberculosis6. A 25-year-old patient presented with peripheral adenopathy, involving a single cervical lymph node. A biopsy sample investigation revealed an unclear node pattern with mixed cellular infiltration. Histologycal study under higher magnification showed numerous variants of Berezovsky-Reed-Stenberg cells, lymphocytes, ne-crotic area and mild diffuse fibrosis. What is the most likely diagnosis?.A. * LymphogranulomatosisB. Nodular lymphomaC. Burkitt's lymphomaD. Lymphocytic lymphomaE. Chronic lymphatic leukemia7. A 65-year-old female is noted to be anemic. He serum protein electrophoresis demonstration a large monoclonal Ig G kappa protein. In her bone marrow are increased numbers of atypical plasma cells. Her skull X-ray show multiple lytic areas. What is the most likely diagnosis?A.* MyelomaB. Polycytemia vera (erythremia, Osier's disease)C. Lymphocytic lymphomaD. ChondrosarcomaE. Bone fibrous dysplasia8. A thoracotomy of a 58-year-old woman revealed in her anterior mediastinum the enlarged and soldered together lymph nodes. Microscopical investigation revealed atypical cells with predominance of Hodgkin cells and giant Reed—Berezovsky -Sternberg cells. A sclerosis was absent. What is the most likely diagnosis?A. *Lymphogranulomatosis with low-spirited development of lymphoid tissueB. Lymphogranulomatosis with predominance of nodularis sclerosisC. Lymphogranulomatosis with predominance of lymphoid tissueD. LymphosarcomaE. Mixed-cellular variant of lymphogranulomatosis9. A tomography revealed enlarged lymphatic nodes. A histological investigation of lymph node's biopsy showed a circular growths of a connective tissue, which surrounded a granuloma - like formation, made from lymphocytes, plasmocytes and giant double- nuclear cells. What is the most likely diagnosis?A. *LymphogranulomatosisB. LymphosarcomaC. TuberculosisD. SarcoidosisE. Lymphatic leukemia10. The young man presented to his physician with enlarged neck lymph nodes. A microscopical investigation of a lymph node biopsy sample revealed the lymphoid tissue proliferation with presence of a giant Reed—Berezovsky -Sternberg cells, eosinocytes, zones of necrosis and sclerosis. What is the most likely diagnosis?

Page 71: Pat an Anew

71

A. *LymphogranulomatosisB. Chronic lymphatic leukemiaC. Multiple myelomaD. LymphosarcomaE. Histiocytosis11. A biopsy of the enlarged lymph node was taken. A histological investigation revealed a diffuse growth of lymphoid cells with adding of eosinocytes, atypical histiocytes, solitary giant Reed—Berezovsky - Sternberg cells with two and more nuclei, cell's necrosis and sclerosis. What is the most likely diagnosis?A. * LymphogranulomatosisB. Lymphatic leukemiaC. Burkett's lymphomaD. SarcoidosisE. Myeloleukemia12. An autopsy of a 67-year-old man revealed the systemic enlargement of lymph nodes with formation of tumorous conglomerates. The spleen was also enlarged with a motley pattern of a cut surface. There were plural, tiny, yellowish-white spots on a red background of a spleen's pulp. What is the most likely diagnosis?A. * LymphogranulomatosisB. SarcoidosisC. LymphosarcomaD. Carcinoma of lungE. Lymphatic leukemia13. Clinical investigation of a patient revealed the enlarged lymph nodes, spleen and liver. A microscopical study of the enlarged cervical (neck) lymph node showed the blurring of its structures and absence of lymphatic follicles. All microscopical views were presented by cells with round nuclei and narrow ring of a ba-sophilic cytoplasm. What is the most likely diagnosis?A. *Lymphatic leucosisB. LymphogranulomatosisC. LymphosarcomaD. MyeloleukemiaE. Multiple myeloma14. An autopsy of a 35-year-old woman revealed the enlarged spleen, (weight SOOgm), liver (weight 4000 gm) and lymph nodes. A bone marrow of a femur diaphysis was juicy (succulent), crimson-red color. Microscopical study of a liver defined dense infiltrates within portal tracts, consisted of immature blood cells. These cells had a round nuclei and narrow ring of a cytoplasm. What is the most likely diagnosis?A. *Chronic lymphatic leukemiaB. Chronic myeloid leucosisC. Generalizated form of lymphogranulomatosisD. Acute myeloblastic leucosisE. Acute lymphoblastic leucosis15. A microscopic investigation of the enlarged neck lymph node biopsy revealed the blurring of its structures, plenty of proliferating lymphocytes with adding of solitary giant Reed-—Berezovsky - Sternberg cells. What is the most likely diagnosis?A. *Lymphogranulomatosis, with predominance of lymphatic tissueB. Lymphogranulomatosis, with exhaustion of lymphatic tissueC. Mixed cell variant of lymphogranulomatosisD. LymphosarcomaE. Nodular sclerotic variant of lymphogranulomatosis16. At young men the increased cervical lymph node is removed. Microscopic investigation revealed the altered node's structure, an absence of lymphoid follicles, sites of a sclerosis and necrosis. The cellular infiltrate is polymorphic with a presence of lymphocytes, eosinocytes, and atypical one-nuclear cells and multinuclear giant cells (Reed—Berezovsky - Sternberg cells). What is the most likely diagnosis?A. * LymphogranulomatosisB. Acute lymphatic leukemiaC. Chronic lymphatic leukemiaD. Burkett's lymphomaE. Mycosis fungoides17. A liver biopsy was taken from a 66-year-old man, with a history of increased quantity of lymphocytes and pro-lymphocytes in his blood. A histological investigation of a liver sample revealed plural accumulations of the mentioned above cells, mainly between hepatic segments. For what disease above listed changes are characteristic?A. *Chronic lymphatic leukemiaB. Acute lymphatic leukemiaC. LymphogranulomatosisD. Chronic persistence hepatitis

Page 72: Pat an Anew

72

E. Hepatocellular carcinoma of liver18. A 4-year-old girl died due to a post-hemorrhagic anemia, resulted from a gas-tro-intestinal profuse bleeding. An autopsy revealed an anemia of her organs, the enlargement of the different groups of lymph nodes, thymomegaly, mild hepatomegaly, splenomegaly and bright red bone marrow. Microscopical study showed hyper cellularity of a bone marrow with monomorphic blast cells infiltrate, diffuse inflammatory tumor-like infiltrates in a liver, a spleen, lymph nodes, meninges and substance of a brain. What is the most likely diagnosis?A. * Acute lymphoblastic leucosisB. Acute myeloblastic leucosisC. Acute nondifferentiated leucosisD. Acute monoblastic leucosisE. Acute plazmoblastic leucosis19. A 14-year-old boy presented to a hospital with enlarged subrnaxillary and cervical lymph nodes. A biopsy procedure was performed. Microscopical investigation revealed the disorder of a lymph node typical structure, a heterogeneous cellular population with a presence of giant multinuclear cells and plural one-nuclear big cells. There were also eosinocytes, neutrophils and lymphocytes in the cell infiltrate. In addition, sites of necrosis and sclerosis were found. What is the most likely diagnosis?A. *LymphogranulomatosisB. Hyperplasia of lymphatic nodesC. Granulomatous lymphadenitisD. Purulent lymphadenitisE. Non-Hodgkin's lymphoma20. A radiological investigation of a man's head revealed in his maxillary and mandibular bones numerous round defects with smooth walls. A histological study showed osteolysis and osteoporosis accompanied with insufficient bone repair. The laboratory test of the urine detected the Bence Jones protein. What is the most likely diagnosis?A. *Multiple myelomaB. Chronic myeloleukemiaC. Chronic erythromyelosisD. Acute myeloleukemiaE. Acute nondifferentiated leucosis21. A physical examination of a young men's oral cavity revealed the atrophy of mucous membrane and red spots on his tongue (atrophic; Hunter's; Moeller's glossitis). Sclera had a yellow coloring. A blood test showed the color index above one. For what anemia these changes are characteristic?A. * Nutritional anemia due to vitamine B,2 deficiencyB. Asiderotic anemiaC. Acute posthemorrhagicD. Chronic posthemorrhagicE. Hemolytic anemia22. A physical examination of a 42-year-old man revealed enlarged lymph nodes. A histological investigation of a lymph node showed lymphocytes, histiocytes, re-ticular cells, small and big Hodgkin's cells, multinuclear Berezovsky -Reed-Sternberg cells (Sternberg-Reed cells) infiltration with solitary necrotic areas. What disease such changes characteristic for?A. * LymphogranulomatosisB. LymphosarcomaC. Chronic leucosisD. Acute leucosisE. Metastasis of carcinoma of lungs23. A gross examination of a dead body revealed the skin's pallor and a yellowness of a sclera. Livores mortis were not defined. The volume of blood in a heart and large vessels was reduced. A blood looked aqueous, hi a skin, mucosa and serous membranes there were petechial hemorrhages. The internal organs, especially a spleen, a liver and kidneys had a rusty color on a cut. A bone marrow of flat bones was a crimson-red and succulent In cortical [tubular, cylindrical] bones it looked like a crimson jelly. Name the disease, connected with a deficiency of vitamin В12.A. * Pernicious anemiaB. Drepancytic (sickle-cell) anemiaC. PanmyelophthisisD. Toxic anemiaE. Acute posthemorrhagic anemia24. A post-mortem of a 56-year-old woman revealed the Hunter's (atrophic; Moeller's) glossitis, atrophy of mucous membrane of a stomach and liver's hemosidero-sis. A bone marrow in all investigated bones was red. A microscopical study showed hyper cellular infiltration in a lamina propria of a stomach with a presence of lymphatic follicles. In a spinal cord there was a funicular myelosis and also haemopoiesis foci detected in a spleen. What is the most likely diagnosis?A. *Addison-Biermer anemia

Page 73: Pat an Anew

73

B. Fanconi's (congenital aplastic) anemiaC. Hypoplastic anemiaD. Chronic gastritis (type A)E. Chronic gastritis (type B)25. A post-mortem of a 4-year-old girl revealed plural petechial hemorrhages on her skin, serous and mucous membranes, large focal hemorrhage in a brain and necrotic tonsillitis. Microscopical study showed multiple cell infiltrates with prevailing lymphocytes in a bone marrow, a liver, a spleen, a thymus, lymph nodes, tonsils and a skin. What is the most likely diagnosis?A. *Acute lymphatic leukemiaB. Chronic lymphatic leukemiaC. Hodgkin's lymphomaD. Follicular non-Hodgkin's lymphomaE. Mycosis fungoides26. A post-mortem of a 15-year-old girl revealed enlarged neck, mediastinal and mesenteric lymph nodes, which were integrated in conglomerates. On a cut section, the tissue pattern of the nodes was non-uniform, with foci of necrosis. Microscopical investigation showed the uneven structure of lymph nodes, foci of sclerosis and necrosis. The cell population was also heterogeneous and included uninuclear atypical cells, giant multinuclear atypical cells, a significant amount of eosinocytes and neutrophils, and sparse lymphocytes. What is the most likely form of Hodgkin lymphoma (lymphogranulomatosis)?A. * Lymphogranulomatosis, mixed cell variantB. Lymphogranulomatosis, lymphohistiocytic variantC. Lymphogranulomatosis, nodular sclerosisD. Lymphogranulomatosis, variant with low-spirited development of lymphoidtissueE. Hodgkin's sarcoma27. A 63-year-old man, with 20 years history of working as the engineer for the service of electronic microscopes, died of a sepsis. An autopsy revealed plural hemorrhages in serous and mucous membranes, a general hemosiderosis, a fatty dystrophy of a myocardium, liver and kidneys, ulcerative -necrotic and purulent processes in a gastro -intestinal system. The red bone marrow was replaced by a fatty. What is the most likely diagnosis?A. *Hypoplastic anemiaB. Megaloblastic anemiaC. Posthemorrhagic anemiaD. Iron deficiency anemia28. A 38-year-old man, with a history of an ulcer, resulted in a stomach's resection, in his blood test had a normal quantity of erymrocytes, but reduced hemoglobin's concentration and decreased color index. An autopsy revealed pale skin and visible mucous membranes; bone marrow of long tubular bones was brightly red. Erythro-cytes in a smear had a normal form and the sizes. They look very pale because of bad staining by dyes. What pathological process took place in this case?A. * Hypochromic iron deficiency anemiaB. B12-folic acid deficiency anemiaC. Acute lymphoblast leukemiaD. Sicklemia sickle cell anemiaE. Aplastic anemia29. A 44-year-old man presented to a gastroenterologist with pains in his epigastrium. A physical examination revealed an icteritiousness of his skin and scleras, an alteration of a tongue's mucous membrane. A tongue grossly looked shining, smooth, with red spots. In peripheral blood's smear there were found enlarged erythrocytes (megaloblasts). A histological study of a gastrobiopsy from a body of a stomach showed a thinning of mucosa, a reduction of glands quantity, superfluous growth of a connective tissue. Specify, what of diagnoses is the most probable in this case:A. * B12-folic acid deficiency anemiaB. Chronic posthemorrhagic anemiaC. Hemolytic anemia.D. Chronic myeloid leukemia.E. Aplastic anemia30. A man, with a history of getting a high doze of ionizing radiation, presented to his physician with marked stomatorrhagia (gingival hemorrhage), spontaneous skin and mucosas' haemorrhages. A blood test showed a normochromal anemia and pancytopenia. The concentration of iron in blood's serum was normal. A histological investigation of a bone marrow puncture sample revealed a replacement of a hemopoietic tissue by the fatty tissue. What is the most likely diagnosis? A. * Aplastic anemiaB. B12-fo!ic acid deficiency anemiaC. Hemolytic anemiaD. Myelodysplastic syndrome

Page 74: Pat an Anew

74

E. Immune cytopenia31. A patient presented with an infiltrative, plaque-like polymorphic skin rash, which had various contours, sizes and a congested -cyanotic color. The lesions tended to peripheral growth and fusion. A microscopical investigation of a skin biopsy revealed massive lymphocytes' proliferation, which occupied the entire derma and a hypodermic fatty layer. What is the most likely diagnosis?A. *Limphomaof skinB. Systemic lupus erythematosusC. Mycosis fungoi'desD. Intradermal nevus

ATHEROSCLEROSIS1. A 46-year-old male suddenly died after developing a heart failure. An autopsy revealed in abdominal aorta the yellow color areas, which did not rise over its surface. Histological investigation of aorta showed the aggregations of cells with a foamy cytoplasm among smooth mussels and macrophages. These cells had a motley-orange coloring, when stained by a Sudan Ш. For what stage of an atherosclerosis such picture is characteristic?A. *LipoidosisB. LiposclerosisC. AtheromatosisD. UlcerationE. Atherocalcification2. A post-mortem of an elderly male revealed microscopic alteration of his coronary artery. There was narrowing of vessel's lumen due to a fibrous plaque with some admixture of lipids. Name the stage of atherosclerosis:A. *LiposclerosisB. LipoidosisC. Pre-lipoidosisD. AtheromatosisE. Atherocalcification3. At autopsy of 63-year-old male revealed an atherosclerosis of the brain's arteries and a thrombosis of the internal carotid artery's branch. Gross investigation showed a focus of moist softening in his brain's tissue. Define the pathological process in the brain.A. * Ischemic infarctionB. Hemorrhagic infiltrationC. HematomaD. EncephalitisE. Tumour of the brain.4. At autopsy of an elderly female an aorta tissue sample collected for histology. Microscopic investigation revealed in aorta's intima the accumulation of xanthoma's cells. At what disease such morphological picture is possible?A. * AtherosclerosisB. HypertensionC. Syphilitic mesaortitisD. Nonspecific aortoarteritisE. Nodular periarteritis5. A 54-year-old male died after developing a heart failure. An autopsy revealed a chronic venous hyperemia of the internal organs, hypertrophy of the left ventricle of a heart and focal cardiosclerosis. Gross investigation of aorta showed yellow-white plaques in the intima with fine-grained masses in their centers, which infiltrated the aortal wall. Give the name of this pathological process.A. * AtheromatosisB. LipoidosisC. LiposclerosisD. ArteriolosclerosisE. Calcinosis.6. A 66-year-old male suddenly died on his way to operation room. An autopsy revealed a hemorrhage into retroperitoneal fat, a saccular dilatation of the abdominal aorta wall with rupture in arterial wall. The aorta's defect had uneven edges and stony hardening of surrounding tissues. Name the most likely disease, resulted in described complication?A. * AtherosclerosisB. HypertensionC. Systemic vasculitisD. Visceral Syphilis E.

Page 75: Pat an Anew

75

7. At autopsy of 52-year-old male revealed yellowish areas at his aorta's intima, this did not bulge above its surface. Histological investigation showed the accumulation of cells with foamy cytoplasm, stained by Sudan III in yellow color. Name the most likely stage of atherosclerosis in aorta?A. *LipidosisB. LiposclerosisC. AtheromatosisD. Pre-lipoidosisE. Atherocalcification8. A post-mortem of a 65-year-old male revealed yellowish streaks and spots in the abdominal aorta's intima, which did not rise above endothelial surface. These areas were stained by Sudan III in orange color. What stage of atherosclerosis such changes are characteristic for?A. *LipidosisB. LiposclerosisC. AtheromatosisD. UlcerationE. Atherocalcification9. A 53-year-old female died from acute myocardial infarction. An autopsy revealed multiple whitish dense plaques in her coronal arteries' intimae, which bulged in and narrowed the vessel's lumen. Name the most likely stage of a coronary atherosclerosis?A. * LiposclerosisB. LipoidosisC. AtheromatosisD. AtherocalcificationE. Ulceration10. A 45-year-old male died in a traffic accident. A histological investigation of his aorta revealed a lipid infiltration of its intima with fats accumulation in myocytes and macrophages of aortal medial layer. Name the stage of atherosclerosis?A. *LipoidosisB. LiposclerosisC. AtheromatosisD. Pre-lipoidosisE. AtherocalcificationHYPERTENSION1. An elderly male, with 15 years history of essential hypertension, died from renal failure. What would be the most likely gross picture of his kidneys at autopsy?A. *Small, dense, a surface is fine-grained.B. Large pied and soft.C. Large redD. Large white and smoothE. Large with the plural thin-walled cysts.2. A 52-year-old male had a long history of hypertension. He presented to emergency care physician with complains of an acute, persisted for few hours rise of a blood pressure. What is the most likely alteration of the arterioles' walls due to hypertonic crisis?A. *Fibrinous necrosisB. HyalinosisC. SclerosisD. AmyloidosisE. Calcinosis3. An elderly woman, with a long history of bronchial asthma, ischetnic heart disease and hypertension, presented with hypertrophy of her heart's left ventricle. Name the cause of the heart alteration?A. * Arterial hypertensionB. Pulmonary insufficiencyC. Chronic ischemia of cardiac muscleD. Bronchial asthmaE. Emphysema of lights4. An elderly male, with 20 years history of hypertension, died from uremia. An autopsy revealed a heart hypertrophy and diffuse cardiosclerosis. There were also small, dense kidneys with granularity of their surface. Histological investigation of kidneys tissue showed the collapse of glomerular's arterioles and sclerosis. Some glomeruli were replaced with pinkish homogenous masses, negative to the Kongo-red staining. Tubules were atrophic. Name the kidney pathology?A. * Primary-scarring kidneysB. Secondary - scarring kidneysC. Amyloidosis

Page 76: Pat an Anew

76

D. Chronic glomerulonephritisE. Chronic pyelonephritis5. A 63-years-old patient, with a history of hypertensive disease, died from cardiac insufficiency. At autopsy, the heart enlargement and dilated ventricular cavities were revealed. Microscopical investigation showed marked hypertrophy of cardiomyocytes, with their fatty dystrophy and hyperchrotnic barrel-like nucleuses. What pathological process is the most likely in a heart?A * Eccentric hypertrophyB. MyocarditisC. Concentric hypertrophyD. Angiogenic cardiosclerosisE. Cardiomyopathy6. A 64-years-old patient died from the brain's hemorrhage. An autopsy revealed markedly decreased kidneys (6x3x2cm in sizes and 60, 0 grams weight). They were dense, anemic with an even, fine-grained surface. On a cut, there was a constant thinning of the kidneys' cortex. Changes in kidneys are the implication of:A. * Arteriolosclerotic nephrosclerosisB. Atherosclerotic nephrosclerosisC. Secondary- scarring kidneyD. Amyloidal- scarring kidney.E. Gouty kidneys7.-A 64-years-old male died from uremia. At autopsy revealed reduced in sizes kidneys, with 50.Og weight. Their surfaces were regularly granulated; the cortexes were thin. A microscopic study showed considerably thickened walls of glomeralar arterioles due to deposition of homogeneous unstructured pink masses. The lumens of these vessels were markedly narrowed; nephrons were diminished with sclerotic changes, tubules were atrophic. For what disease the described changes are characteristic?A. * HypertensionB. Chronic glomerulonephritisC. Amyloidosis of kidneysD. Pyelonephritis with scarring of kidneysE. Acute necrotic nephrosis8. A 65-year-old male long time was ill by hypertensive disease and died from chronic kidney insufficiency. The autopsy showed, that both kidneys are considerably decrease in sizes, their surfaces are granulated; histologically -most glomeruluses are containing hyaline, part of them in sclerotic condition, other in hyperplasia; in stroma - the fields of sclerosis, arteriolo- and arteriosclerosis, elastofibrosis of large kidney arteries branches. What is the name of the exposed changes?A. * Arteriolosclerotic nephrosclerosisB. Atherosclerotic nephrosclerosisC. Secondary-restricted kidneyD. Chronic pyelonephritisE. Amyloidal- scarring kidney.9. An autopsy of 48-year-old patient, who died from the complications of hypertensive disease, revealed small, dense kidneys with fine-grained surface. Parenchyma and cortex matter were atrophied. Give the name for such kidneys.A. * Primary-scarring kidneysB. Amyloidal-scarring kidneysC. Secondary-scarring kidneysD. Pyelonephritic-scarrmg of kidneys E.10. An autopsy of an elderly woman, with a long history of hypertensive disease, revealed small size and weight (80 grams), firm kidneys. They had grey color and granular surface. On a cut, there was uniform thinning of a cortex. How it is possible to name the changes in kidneys?A. *Primary- scarring kidneysB. Pyelonephritic scarring of kidneysC. Secondary - scarring kidneysD. Amyloidal- scarring kidneysE. Diabetic glomerulosclerosis11. A 63-year-old man, with a long history of hypertensive disease, died from a hemorrhage in the brain. An autopsy revealed reduced in sixes, firm kidneys with fine-grained surface and thinned cortex matter. These changes in kidneys are characteristic for:A. *Primary-scarring kidneysB. Secondary-scarring kidneysC. Amyloidal-scarring kidneysD. Pyelonephritic-scarring of kidneysE. Infarct of the kidney

Page 77: Pat an Anew

77

12. An autopsy of a 61-year-old female revealed the thickening of her heart's left ventricle up to 2.5 cm. Her kidneys were small, contracted and firm. The surface of the kidneys had a granular appearance. On a cut, there was a thinning of a cortical substance. Microscopical investigation showed arteriolosclerosis, glomerulosclerosis and interstitial sclerosis. What is the most likely disease?A. * HypertensionB. AtherosclerosisC. Ischemic heart diseaseD. RheumatismE. Amyloidosis of kidneysISCHEMIC HEART DISEASE1. An autopsy of a 49-year-old patient, who died from lungs edema, revealed in myocardium a yellow-grey, large focus and a fresh blood clot in a coronal artery. What is the most likely diagnosis?A. *Myocardium infarctionB. CardiosclerosisC. MyocarditisD. AmyloidosisE. Cardiomyopathy2. An autopsy of a 58-year-old female revealed in myocardium a large, dense, grey focus, which histologically consisted of the rough connective tissue fibers. It was surrounded by the hypertrophied muscular fibers. What changes arose up in a heart?A. *Postinfarction CardiosclerosisB. Ischemic stage of myocardial infarctionC. Necrotic stage of myocardial infarctionD. Diffuse CardiosclerosisE. Myocarditis3. An autopsy of a 62-year-old male, with a history of ischemic heart disease, revealed an atherosclerosis of coronal arteries and signs of hypertensive disease. On a cut of the heart, in the area of the apex and left ventricle's frontal and lateral walls, there was a well defined yellowish focus, surrounded by hemorrhages. What is the most likely pathological process in the cardiac muscle?A. * Myocardium InfarctionB. Postinfarction CardiosclerosisC. Diffuse CardiosclerosisD. MyocarditisE. Fatty dystrophy of myocardium4. A 32-year-old man suddenly died during the emotionally strained work. An autopsy revealed uneven myocardium blood supply. Histochemical investigation detected a decrease of the amount of glycogen. Electron-microscopical study showed a destruction of mitochondria, the contractures of myofibrils. What is the most likely disorder of a blood circulation?A. *Acute ischemiaB. Chronic ischemiaC. Vacating arterial hyperemiaD. Acute vein hyperemiaE. Angioneurotic arterial hyperemia5. An autopsy of a 56-year-old male, with a history of ischemic heart disease, revealed the edema of lungs. What pathological changes could cause a pulmonary disorder?A. * Acute insufficiency of left ventricleB. Acute general anemiaC. Acute insufficiency of right ventricleD. Ischemia of small circleE. Blood stasis6. A 48-year-old man, with a history of transmural heart infarction of the left ventricle's myocardium, died from the veritable rupture of. heart (the heart tamponade). What process in an infarct zone could promote a heart's wall rupture?A. *Autolysis with melting of myocardium tissue (myomalacia)B. Substitution of connective tissue in area of infarction (organization)C. Rising of blood pressure in the small circulatory circleD. Scar formation with thinning of wall of the left ventricle7. A patient, with a history of myocardial infarction, develops symptoms of blood circulation insufficiency after physical exercises. There is marked cyanosis and edema of subcutaneous tissue of his lower extremities. What changes have developed on a place of a myocardial infarction at the recovered person?A. *CardiosclerosisB. Intracellular regenerationC. Myocarditis

Page 78: Pat an Anew

78

D. Atrophy of myocardium E.8. A 36-year-old man, with acute transmural infarction of the left ventricle's myocardium, died from the rupture of the heart and tamponade. What process could be a reason of the heart rupture in the zone of infarction?A. *Autolysis with melting of myocardium fabric (myomalacia)B. Substitution of connective fabric in area of infarction (organization)C. Rising of blood pressure in the small circulatory circleD. Scar formation with thinning of wall of the left ventricle9. A 58-year-old male, with a history of hypertensive disease, developed a long attack of substernal pain. The infarction of myocardium was diagnosed. A patient died soon. An autopsy revealed flabby myocardium with uneven blood filling. Histological and histochemical examinations showed disappearance of glycogen granules and decreased activity of oxidizing enzymes. What stage of myocardial infarction presented in that case?A. *Ischemic stageB. Necrotic stage.C. OrganizationD. Recurrent myocardial infarctionE. Acute relapsing myocardial infarction10. A 64-year-old patient, with long history of atherosclerosis and myocardial infarction, developed the attack of substernal pain. A patient was hospitalized in 3 days and died soon from progressive cardiovascular insufficiency. An autopsy revealed in the back wall of the left ventricle and interventricular septum of heart a white color focus, about 3 cm in a diameter. It was fibred, falling back, with a clear boundary. Give the name for these changes:A. *Focal cardipsclerosisB. Myocardial ischemiaC. Myocardial infarctionD. MyocarditisE. Dystrophy of myocardium11. A 52-year-old male, after repeated intramural heart infarction of myocardium, gradually returned to health and further supervision of a district internist. In 2 years he died in a motor-car accident. Define a pathological process in myocardium, which wound be revealed at the autopsy?A. *Focal cardiosclerosisB. Diffuse cardiosclerosis.C. Atrophy.D. Necrosis.E. Hyperplasia12. A patient developed substemal pains in the 7 o'clock in the morning. He presented at the Emergency Department in 8 o'clock in the morning, where electrocardiograph investigation revealed a myocardial infarction. Ten minutes later he died. What most reliable morphological sign of myocardium infarction would be found at histological investigation after the autopsy?A * Disappearance of glycogen in myocardial cellsB. Vacuolar dystrophy of myocardial cellsC. Fatty infiltration of myocardial cellsD. Necrosis of myocardial cellsE. Weakening of myofibril of myocardial cells13. A patient died from progressive cardiac insufficiency. An autopsy revealed a flaccid, dilated in diameter heart. A cut surface investigation showed the irregular blood filling of a myocardium, resulted in the patchy pattern of its tissue. A histological study determined a hyperemia of myocardium and stromal accumulations of small mononuclear cells. The described morphological changes present:A. *Nonpurulent interstitial myocarditisB. Vein plethoraC. Fatty dystrophy of myocardiumD. CardiosclerosisE. Myocardial infarction14. A 56-year-old patient presented to the hospital with symptoms of acute myocardial infarction. A diagnosis was confirmed by the EKG and laboratory tests. For 5th days the condition acutely worsened. A progressive cardiac insufficiency resulted in patient's death. A dissection confirmed the diagnosis of myocardial infarction, complicated by the heart's wall rupture and tamponade of pericardium. What process developed in the area of myocardial infarction?A. *Aseptic autolysisB. OrganizationC. EncapsulationD. Septic disintegrationE. Petrification15. A 57-year-old man, with a long history of an alcohol abuse, died at the increasing phenomena of chronic heart failure. An autopsy revealed the weight of a heart 580 grams; a languid, clay color myocardium, with intensive

Page 79: Pat an Anew

79

diffuse interstitial fibrosis. Coronal arteries were intact. A microscopical study of myocardium showed a combination of hydropic and fatty dystrophy of cells; atrophy and hypertrophy of cardiomyocites. There were also some foci of a cells lysis, accompanied with sclerosis. What kind of cardiomyopathy described in that case?A. *AlcoholicB. HypertrophicC. DilatativeD. RestrictiveE. Metabolic16. A 66-year-old patient presented in the hospital with the acute recurrent myocardial infarction of front-lateral wall of the left ventricle. On the 4th day of disease, an acute, marked difficulty in breathing, a cough with considerable quantity of a foamy sputum discharge and facial cyanosis developed. A patient died from progressive cardiac insufficiency. An autopsy revealed enlarged grey-pink color lungs. A foamy liquid flowed down from the cut surface. What pathological process in lungs caused a death?A. *Edema of the lungsB. Lungs infarctionC. Hydro thoraxD. PneumoniaE. PneumosclerosisRHEUMATIC DISEASES1. An autopsy of 48-year-old female revealed a mitral stenosis with valve's incompetence. Histological investigation showed the post- inflammatory cardiosclerosis and Aschoff-Talalayev nodes (granulomas). What is the most likely diagnosis?A. *RheumatismB. Systemic sclerodermaC. DermatomyositisD. Nodular periarteritisE. Lupus erythematosus2. An autopsy of a 34year-old patient with a long history of rheumatism, revealed the epicardial surface of the heart with shaggy exudate formed by grey strands which easily separate from underlying tissues and described as 'bread-and-butter' pericarditis ('hairy heart'). What is the most likely diagnosis?A. *Fibrinous pericarditisB. Purulent pericarditisC. Hemorrhagic pericarditisD. Proliferated pericarditisE. Catarrhal pericarditis3. A histology investigation of a mitral valve sample of a patient who died from complications of rheumatism revealed a mucoid swelling, a damage of the endothelial cells and also thrombi at the valve closure line. Name the type of the rheumatic endocarditis?A. * Acute warty endocarditisB. Diffuse endocarditisC. Fibroplastic endocarditisD. Relapsing warty endocarditisE. Polyps-ulcerated endocarditis4. A 9-year-old boy presented with painless, firm 1-2 mm nodules at the skin around ulnar and knee joints (at extensor's area). Biopsy investigation revealed a central area of fibrinoid necrosis of the connective tissue surrounded by lymphocytes and macrophages. What disease these nodules are characteristic for?A. * Rheumatism.B. Rheumatoid arthritis.C. Systemic scleroderma.D. Nodular periarteritis.E. Lupus erythematosus.5. A 28-year old female died from uremia. A post-mortem revealed an enlarged kidneys, which had a patchy pattern with hemorrhages on there surface. Histological investigation showed eosinophilic deposits (hematoxilin bodies), "wire loop" lesions in the basement membrane of the glomerular tuft, hyaline thrombi and foci of fibrinoid necrosis and also 'onion skin' sclerosis at the spleen's vessels. What is the most likely diagnosis?A. * Lupus erythematosus.B. Rheumatism.C. Systemic scteroderma.D. Rheumatoid arthritis.E. Nodular periarteritis.6. A 56-year old male with a long history of a rheumatic heart insufficiency died with symptoms of hemiplegia shortly before death. Histological examination of his mitral; valve revealed severe sclerosis, nodular collection of

Page 80: Pat an Anew

80

inflammatory cells and vegetations composed mainly of platelets and fibrin. Name the most likely type of endocarditis?A. *Relapsing warty endocarditisB. Acute warty endocarditisC. Diffuse endocarditisD. Fibroplastic endocarditisE. Polyps-ulcerated endocarditis7. Microscopic investigation of the heart auricle from a patient with a history of mitral stenosis revealed the Achoff-Talalayev's bodies (granulomas). What was the most likely cause of the heart insufficiency supported by the histology results?A. *RheumaticB. AtheroscleroticC. SyphiliticD. InnateE. Septic8. A 32-year-old female with a long history of rheumatic valve's defect presented at the hospital with tachypnea and dyspnea, leg's edema, ascites and hepatomegaly. She died from the chronic heart insufficiency. An autopsy revealed a mitral stenosis. What was the most likely factor of the mitral stenosis morphogenesis?A. *Leaves union between itselfB. Sclerosis and bulge of leavesC. Sclerosis and shortening of leavesD. Shortening of tendon filamentsE. Presence of small blood thromboses on the surface of valve9. An autopsy of 61-year-old male with a history of rheumatoid arthritis revealed enlarged dense kidneys, which had yellowish-whitish color and waxy appearance. Grossly, foci of scars were recognized at kidneys surface. Microscopically, at the slides stained by Congo red, homogeneous pink masses at capillaries of glomeruli tufts, arterioles walls and arteries, basal membranes of tubuli and in stroma were found. Name the described complication of rheumatoid arthritis?A. *Secondary amyloidosis of kidneys.B. Postinfective glomemlonephritis.C. Quickly progressive glomerulonephritis.D. Acute necrotic nephrosis.E. Fibroplastic glomerulonephritis.10. A 48-year-old female after exposure to cold presents to her physician with painful deformed fingers joints, which bones were restricted in their movements. Physical examination revealed small firm nodules near the joints. Histological investigation of the nodules biopsy showed the centrally located core of fibrinoid necrosis with surrounding rim of macrophages and hystiocytes. What is the most likely diagnosis?A. *Rheumatoid arthritis.B. Dermatomyositis.C. Rheumatism.D. Gout.E. Deformed arthrosis.11. A post-mortem of 40-year old female who died from uremia revealed enlarged kidneys, which had a patchy pattern of their surface. Histological investigation of kidneys showed eosinophilic deposits (hematoxylin bodies), "wire loop" lesions in the basement membrane of the glomerular tuft, hyaline thrombi and foci of fibrinoid necrosis. Besides these, Libman-Sacks endorcarditis was also determined. What is the most likely pathology in kidneys?A. *Lupus erythematosus nephritis.B. Rheumatoid glomerulonephritis.C. Choleric glomerulonephritis.D. Sclerotic kidney.E. Terminal glomerulonephritis12. A 33-year old female died from chronic kidney failure. A post-mortem revealed multiple scars and infarcts in kidneys and spleen. Histological investigation showed alteration of small and medium sized arteries presented with sclerosis and mild endothelial proliferation. A severe lymphocytes and histiocytes infiltrates were also recognized at the perivascular tissues. What is the most likely disease caused these alterations?A. *Nodular periarteritisB. AtherosclerosisC. Hypertonic diseaseD. Morphine's DiseaseE. Visceral syphilis13. A post-mortem of a 25-year old woman who died from chronic kidney failure revealed a reddish malar rash ('butterfly rash') and small (up to 0,2 cm) pale tan spreading vegetations over the mitral valve surface. Histological

Page 81: Pat an Anew

81

investigation of kidneys showed foci of fibrinoid necrosis, eosinophilic deposits (hematoxylin bodies), "wire loop" lesions in the basement membrane of the glomerular tuft and karyorrhexis. What is the most likely diagnosis?A. *Lupus erythematosus.B. Nodular periarteritis.C. Rheumatism.D. Rheumatic arthritis.14. Histological investigation of the mitral valves of the heart revealed the focal desquamation of endothelial cells replaced by thrombi. The connective tissue had mucoid swelling areas and also zones of sclerosis and revascularization. Name the type of valve's endocarditis?A. *Relapsing warty endocarditisB. Diffuse endocarditisC. Acute warty endocarditisD. Fibroplastic endocarditisE. Polyps-ulcerated endocarditis15. A 62-year-old woman presents to her physician with considerable deformation of metacarpal phalangeal and feet joints. Histological examination of the soft tissues adjacent to the joints revealed a mucoid swelling of the connective tissue, areas of a fibrinoid necrosis surrounded by palisading epithelioid macrophages and sclerosis. Few 'rice bodies' were found within a synovia! cavity. What is the most likely diagnosis?A. * Rheumatic arthritisB. RheumatismC. Behterev's DiseaseD. Hematogenic tuberculosisE. Gout16. A 44-year-old woman presents to her physician ulnar deviation of the hands and flexion-hypertension ('swan neck" or "walrus flipper") deformities of the fingers. Her metacarpal phalangeal joints are easily exposed to a dislocation and a subluxation. Microscopical examination revealed nodular proliferations of synovium, cartilage destruction and "pannus" formation. What is the most likely diagnosis?A. *Rheumatoid arthritisB. Rheumatic arthritisC. Osteoarthritis.D. Lupus erythematosus. E-17. An autopsy of a child, who died of a heart failure, revealed the dilated heart chambers. Microscopic investigations showed the hyperemia of the myocardial stroma, edema, and diffuse interstitial - infiltrates consists of hystiocytes, lymphocytes, some scattered neutrophils and eosinophils. What is the most likely diagnosis?A. *Diffuse interstitial exudative myocarditisB. Focal interstitial exudative myocarditisC. Nodular productive myocarditisD. Interstitial productive myocarditisE. Alternative myocarditis18. A 35- year- old female presented her physician intermittent episodes of ischemia of her fingers, marked by pallor, paresthesias and pain, accompanied by tightening and thickening of the skin and poliarthralgia. Histological investigation of the skin biopsy from affected areas revealed mild epidermal atrophy, hyalinosis of collagen fibers within derma, scattered perivascular lymphocytes' infiltrates. Underlying skeletal muscles expressed interstitial edema, loss of cross-section striation, nidal necroses followed with petrification. What is the most likely diagnosis?A. *DermatomyositisB. Systemic sclerodermaC. Lupus erythematosusD. Nodular periarteritisE. Rheumatism19. A post-mortem of a 19-year old female revealed multiple furuncles on her skin, a warty endocarditis of the heart valves (Libman-Sacks endocarditis), a focal hemorrhage under the endocardium, ulcerative stomatitis, esophagus ulcers, pneumonia, nephritis, and a spleen hyperplasia with a perivascular sclerosis. Morphological investigation of her brain showed areas of necrosis and signs of vasculitis within thalamus. What is the most likely diagnosis?A. *Lupus erythematosusB. Rheumatism.C. Septic endocarditis.D. Ischemic heart disease.E. Hypertensive disease.20. The woman of 45 years within several years has difficulties at swallowing. She also notes the limitation of fingers movements at printing on the computer keyboard, though joints are not painful. Physical investigation revealed her "stony face" (no wrinkles) owing to tightening of the facial skin and restricted motion of the mouth.

Page 82: Pat an Anew

82

The skin biopsy showed a widespread fibrosis of a derma without inflammatory infiltration. Diagnose disease on the listed clinical and morphological data.A. * SclerodermaB. Lupus erythematosusC. DermatomyositisD. AmyloidosisE. Rheumatic arthritis21. A 43-year-old woman presents to her physician with pain and immobility of the bones in metacarpal phalangeal and feet joints. Similar complains in symmetric pattern were about ulnar and knee joints, though not so severe. Physical examination revealed pastous skin over the joints, a partial ankylosis of metacarpal phalangeal and feet joints, and also ulnar deviation of the hands and flexion-hypertension ('swan neck"or"walrus flipper") deformities of the fingers. In addition, movable firm, rubbery and tender 1cm hypodermic nodules were found in the phalangeal joints area. An aspirate of joint fluid showed increased turbidity and presence of white "grains" ("rice bodies"). Immunofluorescence also revealed the rhematoid factor. What is the most likely diagnosis?A. *Rheumatic poliartritisB. Systemic disease of connective tissueC. GoutD. Osteoartrosis E-22. A post-mortem of the patient who died from uremia revealed deformation of a spine column with severe restriction of its mobility. Articular cartilages of spine joints were destructed with persistent chronic inflammation in tissues of joints. The joints cavities filled with connective tissue, in some places leading to ossification and ankylosis formation. In an aorta, heart, lungs a chronic inflammation and a focal sclerosis were discovered. In kidneys an amyloidosis was recognized. What diagnosis in this case is most probable?A. *Ankiloid spondiloartritis (the Behterev's disease)B. The Pedget's disease (deforming ostosis)C. Rheumatoid arthritis.D. Parathyroid osteodystrophy.E. Osteopetrosis (marble disease). ;23. At the patient with suspicion on a systemic disease a biopsy from a site of the skin tightening and restricted motility was taken. A histology investigation revealed all kinds of disorganization of connective tissue fibers with mild cellular reaction and also transition in excessive sclerosis and a hyalinosis. What is the most likely diagnosis?A. *ScierodermaB. Nodular periarteritisC. Lupus erythematosusD. PsoriasisE. DermatomyositisTHE RESPIRATORY SYSTEM DISEASES1. An autopsy of the miner, who had worked in coal mine more of 10 years, revealed in a lung whitish fibrous fibers and nodules 0,2-0,3 cm in diameter. Histological investigation of nodules showed bundles of interacting concentric pink collagen with apparent hyalinosis. There was a minimal inflammatory reaction and also noticeable amount of a brownish dust. What is the most likely type of pneumoconiosis in this case?A. *SilicosisB. TalcosisC. Asbestosis D.Siderosis E. Berylliosis2. Histological investigations of the bronchus wall biopsy material, from the patient with a history of chronic bronchitis, revealed granulation tissue and diffuse inflammatory infiltrate. What kind of a bronchitis was diagnosed?A. *Chronic polypous bronchitisB. Chronic mucopurulent bronchitisC. Chronic mucous bronchitisD. Chronic purulent bronchitisE. Chronic deforming bronchitis3. A 72-year old female died of severe grippe. A post-mortem revealed "the big motley lungs". Histological investigation showed acute hyperemia, hemorrhages, an edema of a pulmonary tissue, erythrocyte-rich exudate that, filled the bronchi and alveoli. What is the most likely kind of pneumonia?A. *Hemorrhagic bronchopneumoniaB. Catarrhal bronchopneumoniaC. Purulent bronchopneumoniaD. Desquamative bronchopneumoniaE. Fibrinous bronchopneumonia4. A 52-year old female, with a history of the chronic glomerulonephritis and chronic renal failure, presented to the hospital with coughing spells accompanied by thick phlegm and breathlessness. Bronchoscopy revealed congested,

Page 83: Pat an Anew

83

edematous, bronchial mucus membrane wim small hemorrhages. A bronchial lumen was narrowed by thick mucus. Name the process in bronchi?A. *Secondary acute catarrhal bronchitisB. Primary acute catarrhal bronchitisC. Chronic catarrhal bronchitisD. Destructive - ulcerous bronchitisE. Catarrhal - purulent bronchitis5. A 68-year old patient, with a history of the chronic inflammatory disease of lungs since his childhood, presented to the hospital with coughing accompanied by scanty sputum (expectoration). The complications of pulmonary and a heart failure resulted in his death. What changes in heart have been found at post-mortem?A. *Right ventricular hypertrophy and dilatationB. Left ventricular hypertrophy and dilatationC. Ventricular hypertrophyD. Ventricular dilatationE. Heart without gross change6. A female patient, with 10 years history of cough with a purulent sputum and dyspnea, died from respiratory and heart failure. A post-mortem gross investigation of a dead body revealed her fingers reminded drum sticks. A dissection of lugs showed a bronchial alteration with a saccular deformity and purulent inflammation. What is the most likely diagnosis?A. *Bronchiectatic diseaseB. TuberculosisC. Chronic bronchitisD. AbscessE. Acute bronchitis7. A patient died in 3 weeks of the pneumonia onset. Macroscopic investigation of lungs revealed the whole of a left inferior lobe became enlarged, consolidated and airless. Grossly lungs got the appearance of a grayish brown, dry surface with fibrin fibers on the pleura. Histological investigation showed the collapsed and bloodless alveolar capillaries, the exudate within alveoli consisted mainly of neutrophil polymorphs. What is the most likely diagnosis?A. * Croupous pneumoniaB. Focal bronchopneumoniaС .[nfluenzal pneumonia гриппознаяD. Fibrinous pleuritisE. Intestinal pneumonia8. The biopsy is taken from a suspicious site at the mucous the right bronchus of a 58-year old male, with a history of the chronic bronchitis, pneumosclerosis and cardiopulrnonary insufficiency. Flistological investigation revealed cellular and tissue atypia, presence of a "cancer pearls" structures. What pathological process associated with histological changes listed below?A. *Squamous cell keratinous carcinomaB. Chronic polypous bronchitisC. BronchiectasisD. Acute bronchitisE. Squamous metaplasia of the bronchial epithelium9. Histological investigation of a biopsy of a bronchus revealed a tumor which is constructed from clusters of atypical cells of a laminated (stratified) squamous epithelium, some areas with typical "pearls". What is the most likely diagnosis?A. * Squamous cell keratinous carcinomaB. Squamous cell nonkeratinous carcinomaC. Solid carcinomaD. Mucinous carcinomaE. Scirrhous carcinoma10. A 42-year old male, with a history of the amyloidosis, died of chronic renal failure. A post- mortem revealed in the lower lung lobe multiple dilated bronchi filled with abundant foul sputum. A gross investigation of a cut surface showed a honey-comb pattern of a lungs tissue. A microscopic study demonstrated the chronic inflammation within bronchial wall and also a replacement of elastic and muscular fibers by a connecting tissue. These changes in a lung are regarded as:A. *BronchiectasisB. BronchopneumoniaC. Chronic bronchitisD. Chronic pneumoniaE. Abscesses of lungs11. A 53-year old male, with a history of the chronic diffusive bronchitis, presented to the hospital symptoms of cardiopulmonary insufficiency, then resulted in the lethal outcome of the disease. An autopsy revealed increased volume and hyperinflation of lungs, which covered a mediastinum with their edges and keep their form

Page 84: Pat an Anew

84

when removed from the body and put at autopsy table. Gross investigation showed a pale grey colour of lungs and a crunch sound while making an incision of a pulmonal tissue. A pressing a finger on a lungs tissue creates a fossa. A mucopurulent exudate was determined within bronchi's lumen. What is the most likely diagnosis?A. *Chronic diffuse obstructive emphysemaB. Chronic focal emphysemaC. Intestinal emphysemaD. Primary idiopathic. emphysemaE. Vicarious compensatory emphysema18. A 48-year old male, with a history of common cold after acute cooling, presented to his physician with symptoms of lung a heart insufficiency. He died soon at the hospital. A post-mortem revealed the right lung enlarged, firm, hypoventilated, with a liver-like consistency and fibrin fibers on the pleura. On a cut section pulmonary tissue had a gray coloring and looked granular. A turbid liquid flowed down the surface. Histological investigation showed an acute inflammation with the exudate within alveoli consisted mainly of neutrophil polymorphs. What is the most likely diagnosis?A. *Croupous pneumoniaB. Focal pneumoniaC. Intestinal interalveolar pneumoniaD. Staphylococcal pneumoniaE. Idiopathic fibrosing alveolitis19. A 52-year old male presented to hospital with symptoms of acute pneumonia, complicated on the 6th day by pulmonary edema. The latter one resulted in the patient's death. A post-mortem revealed the upper part of the right lung enlarged, firm, with fibrin fibers on the pleura. On a cut section pulmonary tissue had a gray coloring and looked granular. A turbid liquid flowed down the surface. Histological investigation showed the exudate within alveoli consisted of fibrin, neutrophil polymorphs, macrophages, and disintegrated red cells. What is the most likely diagnosis?A. *Croupous pneumoniaB. Staphylococcal bronchopneumoniaC. Virus pneumoniaD. Hypostatic pneumoniaE. Adult respiratory distress syndrome20. An autopsy of the middle age man with a long history of bronchiectasis revealed markedly enlarged adrenals volume at the expense of a cortical layer. Adrenals were pale, dense, and sebaceous. Microscopic investigation showed the unstructured, amorphous, eosinophilic, Congo red positive deposits by reticular stroma and within vessels walls. These changes indicate a development of:A. *AmyloidosisB. Mucoid swellingC. Fibrinoid swellingD. LipidosisE. Hyalinosis21. A post-mortem revealed the left lung enlarged, firm, with fibrin fibers on the pleura. On a cut section pulmonary tissue had a gray coloring. A turbid liquid flowed down the surface. What is the most likely diagnosis?A. *Croupous pneumoniaB. Focal pneumoniaC. Intestinal pneumoniaD. Cancer of lungsE. Bronchiectatic disease22. A 42-year old woman, with a history of common cold after acute cooling, presented to his physician with symptoms of cough, chest pain, fever, and difficulty in breathing. She died on the 5th day staying at the hospital. A post-mortem revealed the inferior lobe the right lung enlarged, firm, with membranous fibrin fibers on the pleura. On a cut section pulmonary tissue had a gray coloring and looked granular. What is the most likely diagnosis?A. *Croupous pneumoniaB. Influenzal pneumoniaC. BronchopneumoniaD. Caseous pneumoniaE. Measles pneumonia23. Histological investigation of pulmonary segment, removed from a coal worker's lung, revealed multiple roundish, nodules composed of concentric whorls of dense, hyaline fibers of collagen. What is the most likely disease?A. *SilicosisB. TuberculosisC. BronchitisD. Fibrosing alveolitisE. Cancer of lungs

Page 85: Pat an Anew

85

24. A 46-year old male, with a history of the chronic bronchiectasis, died from uremia. An autopsy revealed enlarged, firm kidneys, with sebaceous (waxy) cut surface. To what disease there correspond such changes?A. *Amyloidosis of kidneyB. GlomerulonephritisC. Acute tubular necrosisD. PyelonephritisE. Arteriolosclerotic nephrosclerosis25. A 54-year old male, with a history of destructive purulent bronchitis died of multiple organ failure. An autopsy revealed cardiomegaly, a sebaceous (waxy) kidney and a sago spleen. What process has complicated bronchitis?A.* Secondary amyloidosisB.Senile amyloidosisC. Tumor-like amyloidosisD. Primary amyloidosisE. Generalized hyalinosisD. SilicoanthracosisE. Asbestosis35. An autopsy of a patient, who had been working within 20 years the coal worker, revealed grayish-black dense,sclerotic lungs. The vast areas of collagen tissue new growth and multiple macrophages, carrying black pigment within cytoplasm, were recognized under microscope. What is the most likely disease?A. *AnthracosisB. AnthracosilicosisC. SilicoanthracosisD. TalcosisE. Siderosis36. An autopsy of the 60-year-old man with a long history of over consumption of alcohol revealed the right lung enlarged, firm, with fibrin fibers on the pleura. On a cut section pulmonary tissue had a gray coloring. Histological investigation showed the exudate within alveoli consisted of fibrin and disintegrated erythrocytes. What is the most likely diagnosis?A. *Croupous pneumoniaB. Focal pneumoniaC. Intestinal pneumoniaD. Primary tuberculosis of lungE. Caseous pneumonia37. An autopsy of the 47-year-old man revealed the right lung firm with dry hypoinflated granular pulmonary tissue. Visceral pleura presented with membranous gray-brown fibrin fibers on the pleura. What is the most likely diagnosis? What is the most likely diagnosis?A. *Croupous pneumoniaB. TuberculosisC. BronchopneumoniaD. Intestinal pneumoniaE. Pneumofibrosis38. A 42-year old man presented to the hospital with symptoms of malaise, complicated with a collapse one hour later. He died soon, not coming to consciousness. A post-mortem revealed the lungs enlarged, congested, cut with a sound of crunch. Whole of a right inferior lobe had a liver-like texture with some fibrin fibers superposed on the pleura. On a cut section pulmonary tissue exhibited a gray-brown coloring. What is the most likely diagnosis?A. *Croupous pneumoniaB. BronchopneumoniaC. Acute venous hyperemia in lungsD. 1'oxic shock syndromeE. Acute myocardial infarction39. An autopsy of the 69-year-old man with a long history of chronic bronchitis revealed increased pulmonary volume and hyperinflation, pale grey color, soft texture lungs that kept their form when removed from the body and put at autopsy table. Histological investigation showed markedly dilated proximal acini, thinned and somewhere ruptured interalveolar septas. For what disease of lungs such morphological picture is characteristic?A. *Emphysema of lungsB. Chronic intestinal pneumoniaC. Atelectasis of lungsD. Bronchiectatic diseaseE. Pleural empyema40. An autopsy of a patient, who had been working within 20 years the coal worker, revealed enlarged, slightly firm and hypoinflated lungs. At the gross investigation of the lungs sectional view multiple, round, grayish-black, diffuse nodules were discovered. Histological study showed nodules composed of concentric whorls of dense collagen fibers, some of them- with hyaline degeneration. What is the most likely disease?

Page 86: Pat an Anew

86

A. *Nodular form of silicosisB. Miliary tuberculosis of lungC. Bronchiectatic diseaseD. Croupous pneumoniaE. Diffusive - sclerotic form of silicosisDISEASES OF ALIMENTARY SYSTEM1. An operation of a male with a medical history of a hematemesis revealed a stomach ulcer penetrated into the muscular layer. Ulcer edges were dense. A bleeding vessel was detected at the ulcer's bottom. What is the type of an ulcer?A. * Chronic ulcer with hemorrhageB. Chronic ulcer with penetrationC. Ecute ulcer with bleedingD. Chronic ulcer with perforationE. Chronic ulcer with malignisation2. A removed appendix delivered to pathology department. Macroscopic investigation revealed its thickening, enlargement, dull and hyperemic serous membrane. A yellowish-green liquid lumen was visible on a cut section within its lumen. What is the most likely type of the appendicitis?A. * Suppurative appendicitis.B. Catarrhal appendicitis.C. Superficial appendicitis.D. Gangrenous appendicitis.E. Apostematous appendicitis3. A microscopic investigation of a colon biopsy revealed a superficial defect of a mucosa, decrease of goblet cells number with lack of mucus in them. In addition, considerable lympho- plasmocyte infiltration with presence of segmented leucocytes and eosinocytes was defined. Choose the most probable and exact diagnosis.A. *Nonspecific ulcerative colitis in exacerbationB. Crohn's diseaseC. Chronic ischemic colitisD. Dysentery in the third stageE. Amebiaz of intestine with forming of ulcers4. Microscopic study of appendix determined intensive leukocyte infiltration hyperemia, stases throughout all its layers. What is the most likely type of appendicitis?A. * Suppurative appendicitisB. Gangrenous appendicitisC. Superficial appendicitisD. Simple appendicitisE. Chronic appendicitis5. Macroscopic investigation of a removed appendix revealed its thickening, enlargement, dull and hyperemic serous membrane with whitish membranaceous incrustation. A whitish-yellow, opaque, viscous liquid detected on a cut section within its lumen. What is the most likely type of the appendicitis?A. * Suppurative appendicitis.B. Gangrenous appendicitisC. Simple appendicitisD. Superficial appendicitisE. Chronic appendicitis.6. The removed appendix thickened and covered by fibrinopuralent incrustation. A purulent exudate infiltrated all appendix layers; the destruction of mucosa membrane was evident. What is the most likely diagnosis?A. Flegmonous-ulcerative appendicitisB. Simple appendicitisC. Suppurative appendicitisD. Gangrenous appendicitisE. Superficial appendicitis7. An autopsy of a 48-year-old woman with a history of the stomach neoplasm revealed the markedly enlarged, firm, white color ovary. Microscopic investigation showed markedly atypical epithelial cells placed among the layers and fibers of a connecting tissue. What is the most likely diagnosis?A. *Krukenberg tumor of ovary.B. Serous cystadenocarcinomaC. Pseudomucinous cystadenocarcinoma.D. Malignant thecoma.E. Malignant granulosa cell tumor.8. A post-mortem of a male with a history of anemia accompanied by vomiting by dark gastric contents reveled in a stomach about 1 liter of liquid blood and bloody clots. A gross investigation showed an on small curvature of a stomach an oval solitary ulcer with the elevated firm edgesand a smooth floor. What is the most likely diagnosis?

Page 87: Pat an Anew

87

A. *Chronic gastric ulcerB. Chronic atrophic gastritisC. Acute gastritisD. Acute gastric ulcerE. Chronic hyper-trophic gastritis9. A 48-year old female presents to her physician enlarged supraclavicular lymph nodes. Microscopic investigation of a biopsy from a lymph node revealed the metastasis of the signet-ring cancer. Choose the most probable localization of a primary tumor.A. *Cancer of stomach.B. Cancer of oesophagus.C. Cancer of thyroid.D. Cancer of lung.E. Cancer of cervix uteri.10. A histological investigation of a gastric biopsy revealed a thinning of a stomach mucosa with reduction of a glands quantity. This was accompanied with a considerable growth of a connecting tissue, dilation of a glands lumen, lymphocytes and plasmocytes infiltration of the mucosa. What is the most likely diagnosis?A. * Chronic atrophic gastritisB. Chronic superficial gastritisC. Chronic atrophic gastritis with an intestinal metaplasiaD. Phlegmon of stomach E.11. Microscopic study of appendix determined intensive leukocyte infiltration throughout all its layers. What is the most likely type of inflammation?A. * Suppurative appendicitisB. Apostematous appendicitisC. Gangrenous appendicitis.D. Superficial appendicitis.E. Flegmonous-ulcerative appendicitis12. A histological investigation of the removed stomach ulcer revealed in its floor the fibrinopurulent exudate, a zone of the fibrinoid necrosis, a granulation tissue with a fibrous tissue underneath. What is the most likely diagnosis?A. * Chronic ulcerB. Acute ulcerC. Acute erosionD. Phlegmon of stomach E.13. A macroscopic investigation of the appendix, 9 cm in length and 0.9 cm in thickness, revealed dull and hyperemic serous membrane. Microscopic study determined edema, stases in capillaries and venules, fine hemorrhages throughout the mucosa. Besides these, the necrotic foci with leukocyte infiltration around them were also verified. What is the most likely diagnose?A. * Superficial appendicitis.B. Simple appendicitis.C. Suppurative appendicitisD. Flegmonous-ulcerative appendicitisE. Gangrenous appendicitis.14. A gastroscopy of a 44-year old patient, with a history of a pain in epigastrium after meal, revealed a hyperemia of the stomach mucosa with the stomach folds reduction. Histological study showed a thinning of the mucosa, a reduction of the glands quantity accompanied with the growth of a connecting tissue, lymphocytes and plasmocytes infiltration. Specify, what of the listed diagnoses is the most probable?A. *Chronic atrophic gastritis.B. Acute catarrhal gastritis.C. Acute suppurative gastritis.D. Chronic superficial gastritis.E. Giant hypertrophic gastritis.15. A physical examination of a fauces of a 12-year-old child with tonsillitis revealed the reddening and slight edema of mucosa, with reactive enlargement of tonsils, dotted by pinpoints of exudates emanating from the tonsillar crypts.What clinical-morphological variant of tonsillitis is the most probable in this case?A. *Lacunar tonsillitisB. Catarrhal tonsillitisC. Purulant tonsillitisD. Fibrinous tonsillitisE. Necrotic tonsillitis16. A 44-year old patient, with a history of the chronic duodenum peptic ulcer, died of peritonitis. An autopsy revealed multiple steatonecroses of a retroperitoneal tissue and a pancreas. A gross investigation of a duodenum

Page 88: Pat an Anew

88

demonstrated an ulcerative defect, which was 5 mm in diameter and 10 mm deep. The edges presented necrotic masses. Diagnose the complication of a duodenal peptic ulcer?A. *PenetrationB. HemorrhageC. StenosisD. Perforation E.17. A histological investigation of a gastric biopsy of a 50-year old woman revealed a thinning of a stomach mucosa with reduction of a glands quantity, foci of an intestinal metaplasia; a plethora, an edema and a stromal sclerosis. These were accompanied with a diffuse leucocytes, lymphocytes and plasmocytes infiltration of the mucosa. What is the most likely diagnosis? A. *Chronic atrophic gastritis in an active phaseB. Chronic atrophic gastritis in a nonactive phaseC. Chronic superficial gastritisD. Acute catarrhal gastritisE. Acute fibrinouse gastritis18. A gross investigation of a stomach revealed a deep defect of a gastric wall with a lesion of a muscular layer. The proximal edge was undermined and distal one was flat. A histological study of the removed stomach showed in its floor the zone of the fibrinoid necrosis, a granulation tissue and massive fibrosis, which replaced the muscular layer. What is the most likely diagnosis?A. *Chronic ulcer in exacerbationB. Chronic ulcer in remissionC. Acute ulcerD. ErosionE. Cancer-Ulcer19. Microscopic study determined edema, diffusive leukocyte infiltration of appendix wall and also a mucosa! lesion with alteration of its muscular layer. What is the most likely diagnose?A. * Flegmonous-ulcerative appendicitisB. Suppurative appendicitisC. Gangrenous appendicitisD. Superficial appendicitisE. Apostematous appendicitis20. A gastroscopy of a 55-year old patient revealed a diffusive swelling, hyperemia and solitary small hemorrhage of the stomach mucosa. These were accompanied with considerable quantity of a muddy, viscous grey exudate on a gastric surface. What gastritis has developed at the patient?A. *Catarrhal gastritisB. Hemorrhagic gastritisC. Flegmonous gastritisD. Fibrinous gastritisE. Corrosive gastritisDISEASES OF THE LIVER1. A microscopical investigation of a liver biopsy revealed a dystrophy, necroses of hepatocytes, a sclerosis, accompanied with destruction of a hepatic beam and a lobular structure, a formation of false lobes and regenerative nodes. What is the most likely diagnosis?A. * Cirrhosis of liverB. Chronic gepatozC. Chronic hepatitisD. Massive diffuse necrosis of liverE. Acute hepatitis2. An autopsy of a 53-year old patient, with a history of a chronic alcoholism and repeated attacks of an alcoholic hepatitis, revealed a firm, yellow color liver with pointed edge. A liver surface was coarsely scarred with multiple fine knots on a cut surface. What is the most likely diagnosis?A. * Cirrhosis of liver.B. Cancer of liver.C. Fatty dystrophy of liver.D. Chronic hepatitis.E. Acute hepatitis.3. A 62-year old patient, with a history of a cholelithiasis accompanied with cholangitis and cholangiolitis, presented to the hospital liver cirrhosis. What is the most likely type of cirrhosis presented?A. *BiliaryB. InfectionC. ToxinsD. Nutritional deficiencyE. Circulatory

Page 89: Pat an Anew

89

4. A patient presented to the hospital an ascites, twice enlarged spleen, a varicose esophagogastric veins and veins around and within the rectum. A histological investigation of liver biopsy revealed micronodular cirrhosis. What process has complicated cirrhosis?A. * Portal hypertensionB. Cardiac insufficiencyC. Hepatotenal syndromeD. Hepatic-cellular insufficiency E.5. A 62-year old patient, with a history of chronic virus hepatitis, died of an acute posthemorrhagic anaemia resulted from an esophageal varicose veins bleeding. An autopsy revealed the reduced, dense liver with coarsely scarred surface. A microscopical investigation shpwed regenerative false nodules separated by fibrous tissue, contained the remnants of portal tracts. What morphogenetic type of cirrhosis took place in this case?A. *Portal cirrhosis.B. Postnekrotic cirrhosis.C. Mixed cirrhosis.D. Viral cirrhosis.E. Biliary cirrhosis.6. A 42-year-old male, with a history of rheumatic stenosis of the left atrioventricular orifice resulted in chronic heart insufficiency, presented at the hospital with dyspnea at small physical exercises, tachycardia, a cyanosis of his lips, bubbling rales in the inferior segments of lungs, leg's edema. What histologicalchanges will be characteristic for a liver?A.*Necrosis of hepatocytes in the center of lobule, fatty dystrophy on periphery B. Necrosis of hepatocytes in the center of lobule, hyalin dystrophy on peripheryC. Necrosis of hepatocytes in the center of lobule, hydropic dystrophy on peripheryD. Fatty dystrophy of hepatocytes in the center of lobule, necrosis on peripheryE. Hydropic dystrophy of hepatocytes in the center of lobule, necrosis on periphery7. A 52-year-old patient presented with fatigue, loss of appetite, muscle and joint aches, an increase in body temperature to 38°C. On the 7th day a patient exhibited a yellow color in the skin, urine and around the whites of the eyes and also an acute pain in right hypochondrium. A histological investigation of his liver biopsy revealed a destruction of the liver's beam architecture,-vacuolar and ballooning dystrophies of hepatocytes, some cells were necrotic. The Councilman's bodies were also determined. On the periphery of lobes were registered the enlarged quantity of multinuclear hepatocytes. What form of virus hepatitis is most likely?A. * Icteric hepatitisB. Malignant hepatitisC. Chronic hepatitisD. Cholestatic hepatitisE. Anicteric hepatitis8. An autopsy of a male, with a history of a drug abuse, revealed vacuolar dystrophy of hepatocytes, a Councilman's bodies, periportal inflammation consisted mainly of chronic inflammatory cells, lymphocytes, plasma cells and histiocytes. What is the most likely etiology of the disease?A. *ViralB. BacterialC. ToxicD. ParasiteE. Fungus9. The liver biopsy is taken from the patient with symptoms of a parenchymatous icterus and a portal hypertension. Histological investigation revealed a fatty dystrophy of hepatocytes, a destruction of a hepatic beam and a lobular structure, a formation of false lobes and regenerative nodes. These were accompanied with a porto- portal fibrous septae formation and periportal lympho- macrophageal infiltrates. What is the most likely diagnosis?A. * Cirrhosis of liverB. Alcoholic hepatitisC. Chronic hepatosisD. Viral hepatitisE. Toxic dystrophy10. An autopsy of a male, with a history of a chronic alcohol abuse, revealed a small, firm, micronodular liver. Histological investigation showed a fatty dystrophy of hepatocytes, a formation of small regenerative (false) nodes separated by fibrous septae with lympho- macrophageal infiltration. What is the most likely diagnosis?A, *Alcoholic cirrhosis.B. Chronic active alcoholic hepatitis.C. Chronic persistent alcoholic hepatitis.D. Toxic dystrophy of liver.E. Fatty hepatosis.

Page 90: Pat an Anew

90

11. A patient presented with vomiting, loss of appetite and jaundice. A histological investigation of his liver biopsy revealed vacuolar and ballooning dystrophies of hepatocytes and necrosis of some cells. The Councilman's bodies were also determined. Besides, a destruction of the liver's beam architecture within lobes and inflammation consisted mainly of chronic inflammatory cells, lymphocytes, plasma cells and histiocytes were registered. What form of virus hepatitis is most likely?A. Viral hepatitisB. Purulent hepatitisC. Cirrhosis of the liverD. Toxic dystrophy of the liverE. Malaria12. A 35-year-old man exhibits clinical symptoms of the virus hepatitis В. А puncture liver biopsy revealed: a destruction of the liver's beam architecture with polymorphism of hepatocytes and plural mitosis figures. Hepatocytes were enlarged, filled by vacuoles which contained a transparent liquid. What kind of a dystrophy is characteristic for the described case?A. * Hydropic dystrophyB. Fatty dystrophyC. Hyalin dystrophyD. Carbohydrate dystrophy E.13. An autopsy of a female, with a history of a rheumatic disease with a mitral stenosis, resulted in decompensation stage, revealed a general venous plethora, small, firm and micronodular liver. Gross investigation of a cut surface showed a lobular structure and yellowish-brown color of hepatic tissue. Define the most probable pathology diagnosed in a liver.A. *Portal cirrhosisB. Fatty dystrophyC. Billiary cirrhosisD. Postnecrotic cirrhosisE. AtrophyDISEASES OF THE URINARY SYSTEM7. An autopsy of a 59- year-old man revealed the coral-like stone filled all renal pelvis of a right kidney. The kidney was enlarged, hyperemic with the capsule, taken out hardly. The pelvis and calyxes cavities were dilated, filled by a turbid, greenish- yellowish viscous liquid. The mucosal layer was dim with some hemorrhages. A cut surface investigation showed motley pattern of the kidney's tissue, with yellow spots in up to 1cm in diameter. What complication to nephrolithiasis has developed?A. *Chronic pyelonephritis with exacerbationB. Tumour of kidneyC. Postinfectious glomerulonephritisD. Primary amyloidosisE. Rapidly progressive glomeralonephritis.8. Histological study of a renal biopsy revealed a necrosis and a marked dystrophy of the convoluted tubules epithelium, tubulorrhexis, stromal edema and hyperemic medulla. Name the pathological process?A. *Necrotic nephrosisB. GlomerulonephritisC. PyelonephritisD. Cystic diseaseE. Kidney stone disease9. A 59-year-old man with a long history of bronchiectasis disease, a pneumorrhagia, presented with facial and back edema, and proteinuria of 33mg/l. A pulmonary bleeding soon resulted in his death. A post-mortem revealed the enlarged firm kidneys with a sebaceous (waxy) cut surface. Histological investigation showed the accumulation at glomeruli and under tubular epithelium homogeneous eosinophilic, Congo-Red positive masses, which gave a metachromasia reaction with the methyl violet. Name process which has developed in kidneys?A. *AmyloidosisB. HyalinosisC. Fatty dystrophyD. Mucoid swellingE. Fibrinoid swelling10. An autopsy revealed substantial enlargement of a right kidney. A gross investigation of a renal cut section showed a stone in it with marked dilation of the pelvis and calyces by urine and thinning of the renal parenchyma. What of diagnoses is the most likely?A. * Hydronephrosis.B. PyelectasiaC. Pyelonephritis.D. Cyst of kidney E.

Page 91: Pat an Anew

91

11. An autopsy of a 44-year-old female revealed a substantial enlargement of a right kidney reminding grape clusters. The cut surface showed cavities varying in size from 0, 5 to 3 cm in diameter, which were filled by serous liquid and colloid masses. Kidney parenchyma between cavities was thinned to 0, 1 cm. What is the most likely diagnosis?A. *Renal cystic diseaseB. Chronic pyelonephritisC. Acute pyelonephritisD. NephrolithiasisE. Dysphasia of kidneys12. A post-mortem of a patient with a long history of cystitis and a dyskinesia of ureters revealed morphological signs of uremia. Kidneys were contracted, with irregularly scarred surface. In the renal pelvis fine urate stones and sand were registered. Histological investigation showed 'the thyroid kidney' and areas of an interstitial inflammation. What is the most likely diagnosis?A. *Chronic pyelonephritisB. Acute pyelonephritisC. Atherosclerotic-contracted kidneyD. Primary- contracted kidneyE. Amyloidal- contracted kidney13. An autopsy of the 58-year-old patient revealed an enlarged kidneys, which had a well defined yellow-grey with red specs cortical layer. Kidney's medulla colored in a dark red. Histological investigation revealed proliferation of capsular epithelial cells and podocytes with crescents formation, sclerosis and hyalinosis -of the glomerular tufts, stromal fibrosis. What is the most likely kidneys disease?A. *Extracapillary productive glomeralonephritisB. Intracapillary productive glomerulonephritisC. Extracapillary exudative glomerulonephritisD. Intracapillary exudative glomerulonephritisE. Interstitial nephrite14. A 58-year-old patient with a history of purulent osteomyelitis died of chronic renal insufficiency. A post-mortem revealed the enlarged firm kidneys of white-yellow color with a sebaceous (waxy) cut surface. What is the most likely diagnosis?A. * Amyloidosis of kidneysB. Chronic glomerulonephritisC. Subacute glomerulonephritisD. Septic nephriteE. Acute necrotic nephrosis15. A post-mortem revealed the enlarged kidneys with a lobulated surface due to plural cavities, with the smooth wall, filled with a transparent liquid. What is the most likely disease?A. *Renal cystic diseaseB. Necrotic nephrosisC. PyelonephritisD. GlomerulonephritisE. Infarct16. A physical examination of a young woman revealed marked edemas and high proteinuria. A histological investigation of kidneys biopsy determined a disappearance of podocytes, a declining of heparansulfate in a basal membrane of glomerular capillaries. What is the most likely disease?A. * Idiopathic nephrotic syndromeB. Postinfectious glomerulonephritisC. Rapidly progressive glomerulonephritisD. Chronic glomerulonephritisE. Acute necrotic nephrosis17. A post-mortem of a 52- year-old man, with a long history of chronic glomerulonephritis, resulted in his death, revealed markedly reduced, firm, finegrained kidneys. Microscopical investigation showed fibrinous inflammation of serous and mucosas, dystrophic changes of parenchymatous organs and a brain edema. What complication has led to the serous and mucosas lesions?A. *UremiaB. AnaemiaC. SepsisD. DIC-syndromeE. Thrombocytopenia18. A young man died of a burn disease. A post-mortem revealed the brain edema, liver and kidneys enlargement. Gross investigation showed the thickened pale-grey cortical layer and hyperemic medulla. Histological study demonstrated focal tubular necrosis at different points along the nephron with a disruption of tubular basement

Page 92: Pat an Anew

92

membrane interstitial edema with leukocytic infiltration and a hemorrhage. What of the listed diagnoses the most authentic?A. * Necrotic nephrosis.B. Tubulointerstitial nephritis.C. Pyelonephritis.D. Gouty kidney.E. Myeloma kidney.19. Histological investigation of a kidney biopsy showed sclerosis, lymphocyte-plasmocyte infiltration of the renal pelvis walls and calyces; tubules dystrophy and atrophy. The intact tubules were dilated, stretched by colloid-like masses. The epithelium was flattened. In whole, the microscopic picture of tissue sample had 'a thyroid kidney' pattern. What diagnosis is most probable?A. * Chronic pyelonephritisB. Sharp pyelonephritisC. GlomerulonephritisD. NephrosclerosisE. Tubulointerstitial nephritis20. The patient treated for a cold by the big doses of paracetamol, presented symptoms of an oliguria and a/otemia. In 5 days he died of an acute renal insufficiency. Histological investigation of kidneys revealed the diffusive edema of an interstitial tissue of a kidneys cortex, its infiltration by lymphocytes, the eosinophils, some neutrophils. Almost intact glomerules were found along with a destruction of a tubular epithelium. Name the most probable type of a kidneys lesion?A. * Tubulointerstitial nephritisB. Acute glomerulonephritisC. Nephrotic syndromeD. PyelonephritisE. Necronephrosis21. A macroscopic investigation of the removed kidney reveled renal swallowing, congestion and a capsule easily taken out. Renal pelvis and calyxes were dilated, filled by turbid urine. Their mucosa was dim, with the areas of hemorrhages. The cut surface of a kidney had a motley pattern; yellow-grey zones were surrounded by plethora and hemorrhages areas. To what disease there corresponds such macroscopic picture of kidneys?A. *Acute pyelonephritisB. Acute glomerulonephritisC. Amyloidosis of kidneysD. NephrolitiasisE. Renal cystic disease22. Histological investigation revealed proliferation of Bowman's capsule epithelial cells, podocytes and macrophages, resulted in crescents formation, which compressed glomeruli. Necrotic capillary loops had fibrinous thrombi within their lumens. A sclerosis and hyalinosis of some glomeruli were detected. In addition, the expressed nephrocytes dystrophy, an edema and stromal infiltration of kidneys was observed. What is the most likely kidneys pathology?A. * Rapidly progressive glomerulonephritisB. Postinfectious glomerulonephritisC. Chronic glomerulonephritisD. Chronic pyelonephritisE. Amyloidosis of kidneys23. A 53-year-old patient with a history of a chronic glomerulonephritis died of chronic renal insufficiency. A post-mortem revealed smaller than normal, firm kidneys. Their capsule was taken out hardly, baring a granular renal surface. On a cut section a cortical and cerebral layers were thin with dry, anemic, grey color kidneys tissue. How is called such kidneys?A. * Secondary-scarring kidneysB. Primary- scarring kidneysC. Atherosclerotic - scarring kidneysD. Amyloidal- scarring kidneysE. Pyelonephrotis - scarring kidneys24. A 5-year-old girl with a history of an acute respiratory virus infection presented with widespread swelling, a massive proteinuria, a hypoalbummemia, a lipidemia. A microscopic investigation of the renal biopsy revealed an absence of small podocytes processes within vascular glomerulus. What is the most probable diagnosis?A. *Lipoid nephrosisB. Postinfectious glomerulonephritisC. Rapidly progressive glomerulonephritisD. Focal segmental glomerulosclerosisE. Membranous glomerulonephritis

Page 93: Pat an Anew

93

25. An autopsy of a patient, who died from poisoning of ethylene glycol, revealed enlarged edematous kidneys. Their capsule was taken out very easily. Gross investigation showed the wide pale-grey cortical layer and dark red medulla. What disease has developed at the patient?A. * Necrotic nephrosisB. Acute pyelonephritisC. Acute glomerulonephritisD. Acute tubulointerstitial nephritisE. Lipoid nephrosis26. A 12-year-old boy with a history of acute tonsillitis two weeks ago presented with edematous face in the mornings, increasing of arterial pressure, urine in a kind of "meat slops"('coca-cola' colour). Immunohistochemistry of kidneys biopsy revealed immune complexes on basal membranes of capillaries and within glomerular mesangium. What disease has developed at the patient?A. * Acute glomerulonephritisB. Acute interstitial nephriteC. Lipoid nephrosisD. Acute pyelonephritisE. Necrotic nephrosis27. The young man presented in hospital with a headache and giddiness. Within last year he often had high arterial pressure. Medical drugs almost did not help. Kidneys biopsy investigation revealed the extra capillary epithelial proliferation with a crescent formation. For what disease described pathology is characteristic?A. * Rapidly progressive glomerulonephritisB. Acute glomerulonephritisC. Lupus glomerulonephritisD. Wegener's granulomatosisE. Chronic glomerulonephritis28. A 48-year-old woman died of renal insufficiency. A post- mortem revealed enlarged flaccid kidneys with wide, swallowed, dim cortical layer. A yellow-grey with red specks cortex was delimited from dark red cerebral substance. Microscopic investigation showed a proliferation of Bowman's capsule epithelial cells, podocytes and macrophages, resulted in crescents formation. For what disease described changes are characteristic?A. * Subacute glomerulonephritis.B. Acute glomerulonephritis.C. Chronic glomerulonephritis.D. Acute pyelonephritis.E. Amyloidosis of kidneys.29. A macroscopical investigation of a removed kidney revealed in proximal area of an ureter the concrement which obturated its lumen. The kidney was markedly enlarged; a parenchyma was atrophic with considerably dilated pelvis and calyces. Microscopical study of kidney's sample showed the diffusive sclerosis, an atrophy of glomeruli and tubules. Survived tubules were cystically dilated. What complication of a nephrolithiasis developed at the patient?A. * Hydronephrosis.B. Pyonephrosis.C. Pyelonephritis.D. Glomerulonephritis.E. Chronic paranephritis.30. An autopsy of the 68-year-old man revealed a sallow-grey colour of his skin with petechial hemorrhages. His face and tongue had a powdery 'uremic frost'. Fibrinous hemorrhagic laryngitis, a tracheitis, a shaggy pericardium and a catarrhal gastroenterocolitis were also diagnosed. For what syndrome the given complex of morphological changes is characteristic?A. *Chronic renal insufficiencyB. Acute hepatic insufficiencyC. Acute renal insufficiencyD. Chronic heart insufficiency E.31. A man died of renal insufficiency. A post- mortem revealed enlarged flaccid kidneys with wide, yellow-grey with red specks cortex. Microscopic investigation showed a proliferation of Bowman's capsule epithelial cells, resulted in crescents formation. Capillary loops exhibited necrotic alterations and fibrin thrombi within their lumens. What is the most likely diagnosis?A. * Subacute glomerulonephritisB. Acute glomerulonephritisC. Lipoid nephrosisD. Chronic glomerulonephritisE. Amyloidosis of kidneys

Page 94: Pat an Anew

94

32. A post-mortem of a patient with a history of chronic renal insufficiency revealed the enlarged, firm, sebaceous (waxy) kidneys with multiple retractions on their surface. Histological investigation showed many glomeruli were replaced by the Congo-Red positive masses. The same substance was found on the basal membranes of capillaries, in mesangium and also within arterial walls and kidneys stroina. What of the listed diagnoses the most likely?A. * Amyloidosis of kidneysB. Acute glomerulonephritisC. Chronic glomerulonephritisD. Subacute glomerulonephritisE. Lipoid nephrosis33. A 60-year-old woman died of a chronic renal failure. An autopsy revealed the irregularly contracted and markedly lobulated kidneys with a fibrous tissue alternated with an intact parenchyma. Renal pelvises were dilated, their walls -thickened. Histological investigation showed sclerosis, lymphocyte - plasmocyte infiltration of the renal pelvis walls, and interstitium. What diagnosis is most probable?A. *Chronic pyelonephritisB. Acute pyelonephritisC. Acute glomerulonephritisD. Tubulointerstitial nephritisE. Chronic glomerulonephritis34. The kidney biopsy is taken from the man who is in clinic with symptoms of a mercury poisoning. Histological investigation revealed focal tubular necrosis, an edema, hyperemia, hemorrhages and leukocytic infiltration in a stroma. What condition has developed at the patient?A. * Acute necrotic nephrosisB. Acute glomerulonephritisC. Chronic renal insufficiencyD. Acute pyelonephritisE. Chronic pyelonephritis36. A 42-year-old man died of renal insufficiency. Microscopic investigation revealed a proliferation of Bowman's capsule epithelial cells, podocytes and macrophages, resulted in crescents formation, necrosis of capillary loops and fibrin thrombi within their lumens. In addition, the histological study showed sclerosis and hyalinosis of glomeruli, tubular atrophy and stromal fibrosis of kidneys. What of the listed diagnoses the most likely?A. * Subacute glomerulonephritisB. Acute glomerulonephritisC. Chronic glomerulonephritisD. Focal segmental sclerosisE. Membranous nephropathy37. A patient died from uremia. A post- mortem revealed enlarged flaccid kidneys with wide, swallowed with red specks cortex. Medulla had a dark red coloring. Microscopic investigation showed epithelial crescents which compressed capillaries. Nephrocytes exhibited a dystrophy; a stromal edema and infiltration were also recognized. What is the most likely diagnosis?A. * GlomerulonephritisB. PyelonephritisC. NephrolithiasisD. Nephrotic syndromeE. Amyloidosis of kidneys38. The elderly man, with 15 years history of a membranous -proliferative glomerulonephritis, constantly received hemodialysis therapy. The last half a year he did not receive a treatment. He presented to the hospital with extremely severe condition, without consciousness, with a smell of urea from his body and edemas. A marked pleuritis, pericarditis and peritonitis were also diagnosed. In a whole, all symptoms were regarded as a uremia. What kind of an inflammation is the most probable?A. *Fibrinous inflammationB. Purulent inflammationC. Catarrhal inflammationD. Hemorrhagic inflammationE. Serous inflammation39. A postmortem of a 58-year-old male revealed in kidneys asymmetrical, coarse, corticomedullary scars. Histological investigation of a kidney showed atrophy of cyst-like dilated tubules, filled with colloid eosinophilic masses. Interstitium was characterized by pronounced periglomerular sclerosis. What diagnosis is most probable?A. *Chronic pyelonephritis with scarringB. Chronic glomerulonephritis with scarring of kidneysC. Amyloidosis of kidneysD. Tuberculosis of kidneysE. Kidneys at hypertension

Page 95: Pat an Anew

95

40. At the patient operated concerning a widespread tumour of an abdominal cavity, the considerable bleeding and declining of arterial pressure took place. After operation the acute renal insufficiency, resulted in the patient's death, developed. An autopsy revealed the wide light pink cortical layer, which was clearly delimited from dark red pyramids. Histological investigation showed the absence of nuclei in epithelium of convoluted tubules, tubulorrhexis, venous hyperemia. Nuclei of the cells of vascular glomeruli and straight tubules were unchanged. What pathology of kidneys has developed at the patient?A. * NecronephrosisB. InfarctC. GlomerulonephritisD. PyelonephritisE. Nephrosis41. A 49-years-old- patient with a 10 days history of dysentery presented with raised body temperature, a pain in lumbar area, a fever and a considerable quantity of leucocytes in urine. Histological investigation of a puncture renal biopsy revealed hyperemia, interstitial leukocytic infiltration and numerous inflammatory cells (mainly neutrophiles) with a desquamated epithelium, filling renal tubules. What complication has developed at the patient?A. *Acute pyelonephritisB. Chronic pyelonephritisC. PyelitisD. GlomerulonephritisE. Necrotic nephrosisENDOCRINE SYSTEM DISORDERS1. A 23-years-old male presented to his physician with excess of the somatotropic hormone, the enlarged nose, lips, ears, a mandible, hands and feet. What is the most likely diagnosis?A. *AcromegalyB. Pituitary nanismC. Itsenko-Cushing diseaseD. Addison's diseaseE. Adiposogenital dystrophy2. A histological investigation of the removed enlarged thyroid gland revealed irregular form and sizes follicles. The follicular epithelium proliferated both into follicles lumen and outside. Layers of the connective tissue separated various sites of the thyroid. What is the most likely diagnosis?A. *Macro-microfollicular goiterB. Diffuse goiterC. Autoimmune thyroiditisD. Follicular carcinomaE. Riedel's thyreoiditis3. A 42-year-old man presented with a thyroid gland enlargement in two times. A physical investigation revealed firm, irregular lobulated gland. Histological investigation showed a diffusive infiltration of thyroid gland by lymphocytes, plasmocytes with follicles formation and the enhanced growth of a connective tissue. What is the most likely diagnosis?A. * Hashimoto's goiter.B. Endemic goiter.C. Sporadic goiter.D. Diffuse toxic goiter.E. Riedel's thyreoiditis4. A 52-year-old man with a history of hematogenic tuberculosis presented to the hospital with hyperpigmentation skins (melanosis) and mucosas, a cachexia, cardiovascular insufficiency. What disease caused such changes?A. *Addison's diseaseB. PheochromocytomaC. Simmonds' diseaseD. Itsenko-Cushing diseaseE. Graves' disease5. A 48-years-old woman presented with insomnia (inability to get enough sleep, a bad dream), a fatigue, irritability, exophthalmos (protuberance of eyes) and tachycardia. The thyroid gland was enlarged. She died soon due to cardiovascular insufficiency. Histological investigation of the thyroid tissue revealed a proliferation of an epithelium with formation of papillas, a colloid colliquation, lymphocyte and plasmocyte infiltration and formation of lymphatic follicles with the germinal centres. What is the most likely diagnosis?A. * Diffuse toxic goiter. (Basedow's disease)B. Endemic goiterC. Sporadic goiterD. Hashimoto's thyreoiditisE. Benign tumor of thyroid gland

Page 96: Pat an Anew

96

6. A 48-year-old male, with a long history of living in the mountain district of Central Asia, presented with a thyroid gland enlargement, resulted in complicated swallowing. A physical investigation revealed an increasing of a body mass, slowness, drowsiness, and puffy face. A microscopic study of a thyroid gland showed irregular follicles with a hypochromic colloid. What of diagnoses is the most probable?A.,* Endemic goiter.B. Basedow's goiter.C. Hashimoto's thyreoiditis.D. Sporadic goiter.E. Riedel's thyreoiditis.6. A 56-year-old woman with a long history of parathyroid glands disease, died from increased renal insufficiency. A post-mortem revealed a bones deformation of extremities, a vertebral column, and ribs. Bones were fragile, soft and easily deformed or cat. They had some motley tumorous formations. Kidneys were contracted. Histological investigation revealed lacunar bone resorption. In addition, a microscopic study demonstrated giant cell granulomas within the centres of tumorous formations, accumulations of erythrocytes and a hemosiderin. Name the described disease.A. *Parathyroid osteodystrophy.B. Metastases of cancer in a boneC. Chronic renal insufficiencyD. Paget's diseaseE. Myeloma7. A 42-year-old woman presented with the obesity, chiefly of the trunk and face, steroid diabetes, an arterial hypertension and secondary dysfunction of ovaries. An autopsy. revealed a hypertrichosis, a hirsutism, strias on a skin of hips and a abdomen. In the anterior lobe of the pituitary gland a tumour was found. Histological investigation showed the basophilic adenoma of the pituitary and hyperplasia of a fascicular layer in adrenals. What of the listed diagnoses is the most likely?A. * Itsenko-Cushing diseaseB. Itsenko-Cushing syndromeC. Simtnonds' diseaseD. Adiposogenital dystrophyE. Pituitary nanism8. A histological investigation of a thyroid gland, presented to pathology, revealed a mild atrophy of a parenchyma, sclerosis, diffusive infiltration of a stroma by lymphocytes and plasmocytes with formation of lymphoid follicles. What is the most likely diagnose?A. *Autoimmune thyroiditisB. Parenchymatous goiterC. Toxic goiter D; ThyroiditisE. Riedel's thyreoiditis9. A 55-year-old woman died from uraemia. A post-mortem revealed a parathyroid gland adenoma, bones deformation of extremities, a vertebral column, and ribs. Bones were soft, with slight porousness. On a cut they had a motley pattern with multiple cysts. Histological investigation revealed marked bone remodelling and lacunar resorption of osteoid and fibrous tissues. What is the most likely diagnosis?A. *Parathyroid osteodystrophy.B. Chronic osteomyelitisC. OsteoporosisD. Fibrous dysplasiaE. Paget's disease10. A 62-year-old man with a 14 years medical history of a diabetes mellitus presented with significantly worsened eye vision for the last two years. What characteristic processes resulted in worsening of the eye vision?A. *MicroangiopathyB. MacroangiopathyC. Calcinosis of hyaloid bodyD. Dimness of cornea E.11. A 45-years-old male presented to the hospital with enlarged nose, ears, a mandible and feet. What is the most likely diagnosis?A. *AcromegalyB. NanismC. Cerebro-hypophyseal cachexiaD. Adiposogenital dystrophy E.12. A male patient presented with obesity of the trunk and face. He also had a long medical history of an arterial hypertension. The death resulted from a hemorrhage in a brain. A morphological investigation revealed a basophilic adenoma of a pituitary body and hyperplasia of adrenals cortex. What is the most likely preliminary diagnosis?A. * Itsenko-Cushing diseaseB. Diabetes mellitus A. Acromegaly

Page 97: Pat an Anew

97

D. Pituitary nanismE. Adiposogenital dystrophy13. A thyroid gland of a 48-year-old male was removed at surgery department. Histological investigation revealed various size follicles which were lined by tall columnar epithelium. The latter one was pilled up at places forming papillary infoldings. Colloid appeared lightly staining, watery and finely vacuolated. The stroma showed increased accumulation of lymphoid cells.A. *.Basedow's goiter. В Hashimoto's thyreoiditisC. Riedel's thyreoiditisD. De Quervain's thyroiditisE. Nodular goiter14. A 45-years-old male presented to the hospital with gradual enlargement of feet, right hand, nose and lips. The adenoma of a pituitary body was diagnosed. What is the most likely disease?A. * AcromegalyB. NanismC. Basedow's disease.D. Addison's diseaseE. Diabetes mellitus15. A histological investigation of a thyroid revealed a considerable infiltration of a gland by lymphocytes with formation of lymphoid follicles, a destruction of parenchyma, growth of connective tissue fibers. What is the most likely disease presented in that case?A. * Hashimoto's goiterB. Colloid goiterC. Endemic goiterD. Diffuse toxic goiterE. Parenchymatous goiter16. A 64-year-old patient died of a cachexia. A post-mortem revealed an adenoma of parathyroid glands, bones deformation of extremities, a vertebral column, and ribs. Bones were soft, with slight porousness and easily deformed or cat. What of the listed diagnoses is the most likely?A. *Parathyroid osteodystrophy.B. OsteopetrosisC. ChondrodysplasiaD. OsteomyelitisE. Fibrous dysplasia17. A histological investigation of a thyroid gland biopsy, presented to pathology, revealed lymphoid structures with the germinal (growth) centers among the follicles filled with a colloid. What is the most likely disease presented in that case?A. * Hashimoto's goiter.B. Endemic goiterC. Sporadic goiterD. Basedow's goiterE. Riedel's thyreoiditis18. An autopsy of the man revealed a bronze coloring of a skin and mucosa of an oral cavity. In addition, caseous masses were defined in adrenals. What is the most likely disease in that case?A. * Addison's diseaseB. Itsenko-Cushing syndromeC. Basedow's disease.D. AcromegalyE. Pheochromocytoma19. A 64-year-old man died with symptoms of a hypoglycemic coma. A postmortem revealed reduced, dense pancreas. Histological investigation of pancreas showed the growth of a connecting tissue, an atrophy of Langerhans islets. What disease caused such changes in a pancreas?A. *Diabetes mellitus.B. Mucoviscidosis.C. Acute pancreatitis.D. Cancer of pancreas.E. Hypoplasia of pancreas.20. A 64-year-old man, with a medical history of a diabetes mellitus, presented augmented symptoms of renal failure. What disease caused such changes in a pancreas?A. *GIomerulosclerosis.B. Necrosis of renal tubular epithelium.C. Amyloidosis.D. Proliferative glomerulonephritisE. Membranous nephropathy.

Page 98: Pat an Anew

98

21. A morphological investigation of a thyroid removed due to thyrotoxicosis struma revealed a diffuse infiltration of a gland by lymphocytes with a destruction of parenchyma and an area of connective tissue. What is the most likely diagnose?A. * Hashimoto's thyreoiditisB. Anaphylactic reactionC. ThyrotoxicosisD. Cancer of thyroidE. Adenoma22. A patient presented to surgery with symptoms of hypothyrosis. Macroscopic investigation of the. removed thyroid gland revealed its considerable enlargement, dense-elastic texture and lobulated surface. Histological investigation showed the diffuse infiltration of lymphocytes and plasmocytes of gland's parenchyma with formation of lymphoid follicles. The latter ones had hyperplastic germinal centers with atypia and a metaplasia of a follicular epithelium. In addition, the sclerotic alterations of a parenchyma were also determined. What is the most likely diagnose?A. *Autoimmune thyroiditisB. Diffuse toxic goiterC. Colloid goiterD. Nodular goiterE. De Quervain's thyroiditis23. At operation of excision of a diffusive thyroid gland struma a patient died from acute adrenals insufficiency. An autopsy, besides the atrophy of adrenals, revealed a left ventricle hypertrophy in a heart, a hypertrophy and an intracellular edema of cardiomyocytes, a serous edema and lymphoid infiltration of a myocardial stroma. What is the figurative name of a heart pathology presented at post-mortem?A. *Thyrotoxyc heartB. Tabby cat heartC. Cor bovinumD. Cor pulmonaryE. Brown atrophy of heart24. An autopsy of a 40-year-old woman revealed the obesity, chiefly of the trunk and face ("moonface" and "buffalo hump"), strias on a skin of hips and abdomen. She had in her medical records an osteoporosis with spontaneous fractures of bones, an amenorrhea, an arterial hypertension, a hyperglycemia. What of the listed diagnoses is the most likely?A. * Itsenko-Cushing syndromeB. Addison's diseaseC. Friderichsen-Waterhouse syndromeD. Conn's syndromeE. Sipple's syndrome25. A renal puncture biopsy was taken from a patient with elevated blood sugar levels and high amounts of glucose in his urine. Histological investigation revealed extension of mesangium with focal accumulation of membranous substances and periglomerular sclerosis of some glomerules. In addition, there were found a hyalinosis and plasmatic infiltration of arterioles; lymphocytes, histiocytes and some leucocytes infiltration of a stroma; glycogen deposits within nephrocytes of narrow segment. What is the most likely diagnosis?A. *Diabetic glomerulonephritisB. Arteriolosclerotic nephrocirrosisC. PyelonephritisD. Acute glomerulonephritisE. Subacute glomerulonephritisPATHOLOGY OF PREGNANCY1. A 29-year-old woman presented to maternity department with termination of pregnancy at 20 weeks of gestation. The fetus and fetal membranes were delivered with some blood clots. Histological investigation revealed fetal membranes, chorion villi and decidual tissue. Name the described pathology of pregnancy.A. *Spontaneous complete abortionB. Premature birthC. Hydatidiform moleD. Invasive hydatidiform moleE. Artificial abortion2. A young woman presented to the hospital with an acute pain. Her uterine tube was removed at gynecology department. Macroscopic investigation of a tube revealed its local dilation at the middle third and blood clots accumulations within the lumen. A histological research showed chorion villi, surrounded by areas of erythrocytes with some leucocytes. What is the most likely diagnosis?A. *Tubal pregnancyB. Acute purulent salpingitisC. Hemorrhage in tube of the uteriD. Hemorrhagic salpingitis

Page 99: Pat an Anew

99

E. Tuberculous salpingitis3. A 35-year-old woman had an operation of uterine tube removal at gynecology department. A histological research of a tube's wall reveled chorion villi and decidual cells. What is the most likely diagnosis?A. *Tubal pregnancyB. Placental polypC. ChoriocarcinomaD. Paper-doll fetusE. Lithopedion4. A- post-mortem of a pregnant woman revealed a brain edema, hemorrhagic pneumonia, motley liver with a hemorrhage. Microscopic study showed disseminated blood clotting of vessels, plural fine necroses, and hemorrhages in internal organs. In addition, in kidneys there were found cortical necroses and a fibrinoid necrosis of a tubules epithelium within a nephron. What is the most likely diagnosis?A. *EclampsiaB. Pre-eclampsiaC. SepticopyemiaD. SepticemiaE. Acute renal insufficiency5. A 25-year-old woman with a medical history of recent childbirth presented to gynecology department with metrorrhagia. The histological investigation of a curettage material revealed a pathologic growth, consisted of fibrin clots,chorion villi and decidual tissue, undergoing organization. What is the most likely diagnosis?A. * Placental polypB. Glandular hyperplasia of the endometriumC. Hydatidiform moleD. ChoriocarcinomaE. EndometritisDISEASES OF THE REPRODUCTIVE SYSTEM1. A 45-year-old woman with a medical history of a menstrual cycle disturbance had curettage of the uterine cavity to obtain endometrial tissue for histopathologic analysis. Microscopic investigation revealed the increased quantity of the endometrial glands, their form irregularity. Some of glands were dilated or had a saw-like pattern. What is the most likely diagnosis?A. * Cystic glandular hyperplasia of endometriumB. Placental polypC. Atypical hyperplasia of endometriumD. Glandular polyp of endometriumE. Endometrial adenocarcinoma2. An autopsy of an elderly man revealed enlarged, soft, elastic, slightly nodular prostate. A gross investigation of a cut section showed separate nodes parted by layers of a connective tissue. The compressed surrounding prostatic tissue created a plane of cleavage about them. A microscopic study found the increase of glands quantity. There were variable number of prostatic nodes and glands in them.A. * Glandular nodular hyperplasiaB. Fibrpmuscular (stromal) hyperplasiaC. Mixed nodular hyperplasiaD. AdenocarcinomaE. Undifferentiated carcinoma3. A 30-year-old woman, with a history of a chronic endocervicitis, passed routine inspection at the gynecologist with no complaints. A histological investigation of a biopsy, taken from the pars vaginalis neck of the uteri, revealed the high cylindrical epithelium, secreting mucus and branched out glands underneath. What is the most likely diagnose?A. * EndocervicosisB. Squamons metaplasiaC. Glandular metaplasiaD. LeukopJakiaE. Erythroplakia4. A 68-year-old man presented with difficulty of a miction. The prostate was operatively removed. A microscopical investigation revealed an increase of glandular and muscular elements numbers. A lobular structure of prostate was also altered. What process in a prostate is the most probable?A. * Mixed nodular hyperplasiaB. Glandular hyperplasiaC. Fibromuscular hyperplasiaD. ProstatitisE. Adenocarcinoma5. A 42-year-old woman presented with acyclic, plentiful, uterine bleedings. A histological investigation of a curettage material revealed increased quantity of glands and their cystic dilation. What is the most likely diagnosis?A. * Hyperplasia of endometriumB. Atrophy of endometriumC. Hypertrophy of endometrium

Page 100: Pat an Anew

100

D. MetaplasiaE. Organization6. A 42-year-old woman presented with dysfunctional uterine bleeding. A histological investigation of a curettage material revealed increased quantity of coiled glands and their cystic dilation. What is the most likely diagnosis?A. * Cystic glandular hyperplasiaB. AtrophyC. MetaplasiaD. DisplasiaE. Hypertrophic vegetations7. A microscopic investigation of an endometrium revealed coiled extended glands with 'saw-' and a 'spin-like' pattern. A stromal proliferation with hyperplasia of its cells was also determined. What is the most likely diagnosis?A. * Glandular hyperplasia of endometriumB. Acute endometritisC. LeiomyomaD. Hydatidiform moleE. Placental polyp8. A histological investigation of curettage of the cervical canal of the uteri revealed a prismatic epithelium and a significant amount of gland-like structures. They developed from the prismatic epithelium cambial elements, originated from the neck of a uterus. What is the most likely diagnose?A. *Proliferative endocervicosisB. Simple endocervicosisC. Endocervicosis, stage of healingD. Cervical adenomatosisE. Cervical polyps9. A 52-year-old woman, with a medical history of the menstrual cycle disturbances and hemorrhages, had curettage of the uterine cavity. Histological investigation revealed the increased quantity of the endometrial glands. Many glands had a morphological pattern of a proliferation phase, clumped together and had intraglandular papillary protrusions. Some glandular cells exhibited the signs of nuclear and cytoplasm polymorphism. What is the most likely diagnosis?A. * Atypical hyperplasia of endometriumB. Cystic glandular hyperplasia of endometriumC. Glandular hyperplasia of endometriumD. Glandular polyp of endometriumE. Adenocarcinoma of endometrium10. A physical examination of a 68-year-old man, with complains to an acute ischuria (retention or suppression of the urine), revealed the nodular enlargement of a prostate. A microscopical investigation showed that the majority of knots were constructed of polymorphic glandular structures, with some retention cysts. The latter ones had thick secreta. What is the most likely diagnose?A. *Glandular hyperplasia of prostateB. Fibromuscular hyperplasia of prostateC. Hyperplasia of prostate mixed formD. Adenoma of prostateE. Carcinoma of prostate11. A 24-year-old woman, on a 3rd day after childbirth, presented with a diffusive edema and painful palpation of the right breast. Physical examination revealed hyperemia of the breast and a fervescence. Histological research of a gland's tissue showed diffusive leucocytes infiltration of stroma, an interstitial edema and hyperemia of vessels. What is the most likely diagnose?A. *Acute phlegmonous mastitisB. Acute apostematous mastitisC. Acute serous mastitisD. Chronic purulent mastitis E12. A 48-year-old woman with a grease bloody flux from her genital tracts had a diagnostic curettage of her uterus cavity. Microscopical research of the uterus curettage material revealed a thinning of a mucosal layer, reduction of the endometrial glands number, a fibrosis of a stroma and some lymphoid cells infiltration. What is the most likely diagnosis?A. *Chronic atrophic endometritisB. Acute purulent endometritis D Chronic cystic endometritisE. Chronic hypertrophic endometritis13. A gynecologic investigation of the 36-year-old woman revealed bright red brilliant maculae on a uterus neck. They bleed easily at a touch. Biopsy investigation showed that a tissue sample was covered by a cylindrical epithelium with papillary growths. There was also a growth of glands within the thickness of a uterus neck tissue. What pathology of a uterus neck was diagnosed?

Page 101: Pat an Anew

101

A. * PseudoerosionB. Cervical erosionC. EndocervicitisD. Glandular hyperplasiaE. Leukoplakia14. A 23-year-old woman presents with her menstrual cycle disorder went trough a transvaginal puncture of Douglas' cul-de-sac resulted in aspiration of blood and some other material. A microscopic investigation of the puncture fluid revealed blood, some decidual cells and chorion villi. What is the most likely diagnosis?A. *Ectopic tubal pregnancyB. SalpingitisC. Rupture of Fallopian tubeD. Hydatidiform moleE. Carcinoma uteriPERINATAL PATHOLOGY1. A 33-year-old woman gave birth to a dead fetus with gestational age more than43 weeks. The fetal skin was dry and chap. An autopsy revealed a generalhypotrophy and nuclei of ossification (Beclard's nuclei) in a proximal epiphysis oftibial and humeral bones. Amniotic fluid, an umbilical cord and placentalmembranes were dyed by meconium. Name the period of the described perinatalpathology?A The antenatal periodB. The prenatal periodC. The intranatal periodD. The postnatal periodE. The progenesis period

GAMETOPATHY1. A 3-month-old child died of pneumonia. A macroscopical investigation revealed upward slant of palpebral fissures of the eyes, a sinking down dorsum of nose, dolichouranic, a low locating of small auricles. An autopsy revealed a defect of a heart and the main vessels' development. A genetic study showed a chromosome 21 trisomy. What of the listed diagnoses is the most likely?A * Down's syndromeB. Patau's syndromeC. Shereshevsky-Turner's syndrome (gonadal dysgenesis)D. Edwards' syndrome E.AVITAMINOSIS.1. A 3-year-old child, with history of alimentary dyspepsia, presented with progressing gingival hemorrhage, petechial hemorrhages on a skin and a mouth mucosa. There was also a peridental infection. The described changes characteristic for:A * Avitaminosis СB. Avitaminosis DC. Avitaminosis BlD. Avitaminosis B6E. Avitaminosis A2. A 9-month-old child presented with retarded dentition, the teething order disturbance, a reconfiguration of the upper jaw in a horizontal direction (dolichouranic). Microscopical investigation of his teeth revealed the irregular mineralization of the enamel and crumpled enamel prisms. Some of them were rich with vacuoles. There were also expansion of a predentin zone and a presence of solitary denticles. What is the most likely disease in that case?A *Rachitis, early stageB. Rachitis, late stageC. OsteomalaciaD. A goutE. Hypervitaminosis D3. A 1, 5-year-old child presented with focal thickenings of ribs, wrists, and a curvature of legs. A stomatologist pointed to retarded dentition, the teething order disturbance, the irregular mineralization of the enamel and dentine, a reconfiguration of the upper jaw in a horizontal direction (dolichouranic). What is the most likely disease presented in that case?A * RachitisB. A dystrophic calcificationC. A metabolic calcificationD. A metastatic calcificationE. Osteomalacia

Page 102: Pat an Anew

102

INFECTION DISEASES1. A post-mortem of a patient, with a history of a drug abuse, revealed red-purple papular nodules and blotches on the skin of his lower extremities (Kaposi's sarcoma). In addition, acute pneumocystosis carinii pneumonia was also determined. For what disease the given symptoms are characteristic? A. *AIDSB. InfluenzaC. MeaslesD. DiphtheriaE. Anthrax2. A 48-year-old woman died of pulmonary insufficiency. A post-mortem revealed the enlarged lung with the dark red and pinkish-yellow colour areas ("the big motley lung") and necrotic tracheobronchitis. What is the preliminary diagnosis in that case?A. * Influenzal pneumoniaB. Croupous pneumoniaC. Caseous pneumoniaD. Fibrosing alveolitisE. Measles virus pneumonia3. A 44-year-old man died of pulmonary insufficiency and severe intoxication. A post-mortem investigation of a lungs cut surface revealed a motley pattern with plural focal hemorrhages and the zones of emphysema. Histological study showed a hemorrhagic bronchopneumonia with abscesses formation. A cytoplasm of bronchial epithelium cells had eosinophilic and basophilic inclusions. What is the most likely diagnosis?A. * InfluenzaB. ParainfluenzaC. Adenoviral infectionD. Respiratory syncytial infectionE. Staphylococcal bronchopneumonia4. A young man presented with rash in the form of roseolas and small hemorrhages (petechias) on a skin of a stomach, thorax and petechial rash on eyes conjunctiva. Lesion of his brain soon resulted in his death. A microscopical investigation revealed in destructive-proliferative endo-trombovasculitis in medullar oblongata and pons of a brain, also in a skin, kidneys and myocardium. What is the most likely disease?A. *Epidemic typhusB. SepsisC. Periarteritis nodosaD. Systemic Lupus ErythematosusE. Brucellosis5. A 67-year-old man died of pulmonary insufficiency. A post-mortem investigation of a lungs cut surface revealed multiple, bright, red, deaerated 2,5 cm foci with irregular form and inaccurate borders. A microscopic study showed erythrocytes and some lymphocytes within alveoli. The described changes are characteristic for:A. * Influenzal pneumoniaB. Croupous pneumoniaC. Hemorrhagic infarction of the lungD. Measles virus pneumoniaE. Aspiration pneumonia6. A 42-year-old man, with a history of an acute respiratory disease, died of pulmonary insufficiency. A post-mortem investigation revealed fibrinous -hemorrhagic inflammation in a larynx and trachea mucosa, destructive panbronchitis and enlarged lungs. Multiple abscesses, hemorrhages and necroses created a motley appearance of a pulmonal tissue. What of the listed diagnoses the most likely?A. * InfluenzaB. ParainfluenzaC. Respiratory syncytial infectionD. MeaslesE. Adenoviral infection7. A post-mortem of a 40-year-old patient revealed a pneumocystosis carinii pneumonia, Kaposi's sarcoma and B-cell lymphoma. Medical records suggested that he actively practiced unsafe sexual intercourse with multiple partners at his life. What is the most likely diagnosis?A *A human immunodeficiency virus infection, AIDS'StageB. A human immunodeficiency virus infection, pre- AIDS stageC. A secondary immunodeficiency as a result of primary B-cell lymphomaD. A secondary immunodeficiency as a result of Kaposi's sarcoma E.8. A 38-year-old man presented with an acute fever, elevated temperature to 40°C, a headache, a cough and dyspnea. He died on the 5th day of a disease. An autopsy revealed enlarged lungs with a motley pattern of a pulmonary tissue - "the big motley lung". For what disease the described signs are characteristic?

Page 103: Pat an Anew

103

A.* InfluenzaB. Adenoviral infectionC. Croupous pneumoniaD. Respiratory syncytial infectionE. Bronchiectasis9. A 35-year-old narcomaniac (drug abuser) with a history of HTV-infection died at the hospital. A post-mortem revealed that both lungs were dense, dark-red-grey color, with little air. Histological study showed that interalveolar septs were infiltrated by lymphocytes. Some alveolocytes were transformed to the big cells, with a centrally located round nucleus with a light rim of cytoplasm ("an owl's eye"). What opportunistic infection caused pneumonia in that case?A *CytomegalovirusB. Pneumocystis cariniiC. Atypical mycobacteriumD. Herpes-virusE. Toxoplasma10. A 48-year-old male presented with a sustained high fever, severe headache, a dyspnea, palpitation. A physical investigation revealed a pediculosis, a rash in the form of roseolas and small hemorrhages (petechias) on his thorax skin. In addition, decubituses of legs and foot gangrene were also determined. He died of a heart failure. A histological study showed in the medullar oblongata a hyperemia, stasis, perivascular plasmocytes infiltrates and a proliferation of a microglia (Popov's granuloma). What is the most likely diagnosis? A *Epidemic typhusB. Antrax meningocephalitisC. Meningococcal meningitisD. Typhoid feverE. RubellaHA patient died as a result of a heart failure. Macroscopically traces of a preexisted rash in the form of maculae and spots were recognized. A gross investigation revealed decubituses at the areas of breeches and spinous processes of vertebra. Microscopical study showed a destructive-proliferative endo-trombovasculitis and Popov's granulomas in the central nervous system, in a skin, and adrenals. An interstitial myocarditis was diagnosed in his heart. What is the most likely disease? A * Epidemic typhusB. Q feverC. Typhoid feverD. Periarteritis nodosaE. HIV-infection12. A post-mortem of a woman revealed a septicopyemia with metastatic abscesses of lungs and kidneys; pyosalpinx, a purulent peritonitis, a cachexia and lymphadenopathy. Medical records indicated HIV- positive blood test results within Jast 5 years. What clinical period of AIDS corresponds with the given pathomorphologic signs?A. *Acquired immunodeficiency syndrome (AIDS)B. Incubation periodC. Persisten generalised lymphadenopathyD. Pre-AIDS (AIDS -related complex)E. Reactive lymphadenopathy13. A gross investigation of a brain at a post-mortem revealed the edema, a plethora, and fine hemorrhage in medulla oblongata. A microscopical investigation showed a chromatolysis, hydropy and a necrosis of nervous cells. In addition, the eosinophilic intracytoplasmic formations (little bodies the Babesh- Negri bodies) were defined in pyramidal neurons of the hippocampus. What diagnosis answers the described morphological implications?A. * RabiesB. Meningococcal meningitisC. EncephalitisD. EncephalomyelitisE. Brucellosis14. A post-mortem investigation of a 42-year-old man revealed serous -hemorrhagic inflammation in a larynx and trachea mucosa, a focal pneumonia, areas of atelectases and acute lungs emphysema. Histological research showed a vacuolar dystrophy and loss of cilia within the epithelium layer, an exfoliating of cells with oxyphile inclusions. Such changes in lungs are caused:A. * Influenza virusesB. Parainfluenza virusesC. Measles virusD. AdenovirusesE. RS-viruses15. A 66-year-old man presented with acute tracheitis and bronchitis, soon resulted in bronchopneumonia. He died on the 12th day at the hospital from the pulmonal-heart failure. An autopsy revealed fibrinous -hemorrhagic inflammation in a larynx and trachea mucosa, enlarged lungs. On a cut section lungs had a motley pattern due to

Page 104: Pat an Anew

104

areas of pneumonia, hemorrhages in lung parenchyma, acute abscesses and atelectases. In addition, circulative disturbances and dystrophies were determined in internal organs. What is the most likely diagnosis?A. *Influenza, severe formB. Influenza, moderately severe formC. ParainfluenzaD. Respiratory syncytial infectionE. Adenoviral infection16. A post-mortem of a patient, who died of a HIV- infection, revealed alterations of his brain. A histological investigation determined in subcortical white matter, midbrain and a brain stem fine, perivascular necroses, microglial nodules with multinuclear giant cells, focal gliosis and fibrosis. Name the most likely lesion of the centralnervous system?A. *Subacute meningocephalitisB. Metabolic encephalopathyC. Cytomegaloviral encephalitisD. Primary lymphoma of CNSE. Vacuolar myelipathy17. A patient presented to his physician with expressed immunodeficiency, lymphopenia with change of parity T-helpers to T- suppressors, the lesion of a skin of the lower extremities. The cutaneous pathology was characterized by plural tumorous nodules, dome-shaped purple plaques, involved in skin breakdown with resulting fungating lesions. A histological investigation of the skin's biopsy showed the neoplasm of blood vessels, a dilatation of capillaries, slit-like blood-filled vascular spaces. What is the most likely diagnosis?A. *Kaposi's sarcomaB. BasaliomaC. DerraatomycosisD. Phoma of skinE. Inflammatory dermatitisISA man, with a history of frequent drinking non- boiled water from the river, has developed vomiting and explosive 'rice-water' diarrhea. The abdominal pains were absent; the body temperature did not rise. The intestine biopsy revealed a plethora, a marked swallowing, an edema of enterocytes, and some infiltration of villi by lymphocytes, plasmocytes, and few leucocytes. What is the most likely disease?A. *CholeraB. SalmonellosisC. Typhoid feverD. DysenteryE. LambliasisCHILDHOOD INFECTION1. An autopsy of 3-year-old child revealed hyperemic, turbid arachnoids, which had a form of yellowish-greenish 'cap'. At microscopic investigation a characteristic findings of arachnoids thickening, hyperemia, purulent with fibrin exudate infiltration was present. What is the most likely diagnosis?A. *Meningococcal infectionB. TuberculosisC. AnthraxD. IfluenzaE. Measles2. A child presented with catarrhal conjunctivitis, pharyngitis, a laryngotracheobronchitis. A gross investigation of the oral cavity revealed white spot on the buccal mucosa opposite the first molar. In addition, a blotchy and papular rash appeared on the patient's face, trunk and extremities later. What is the most likely diagnosis?A. * MeaslesB. Scarlet feverC. Meningococcal infectionD. Epidemic typhusE. Influenza3. A 4-year-old child presented to the hospital with a fever and a sore throat. An oral cavity examination revealed an edema of a soft palate and grey membranes on tonsils. These membranes left a raw hemorrhagic surface when forcibly removed. What the most likely diagnosis is among listed below?A. *Diphtheria faucesB. Simonovsky-Vensans' anginaC. Lacunar anginaD. Infectious mononucleosisE. Necrotic angina4. An autopsy of 11-year-old child revealed plural hemorrhages, mainly in skin of breeches and lower extremities, serous and mucosas, and also in a brain. Gross investigation determined a focal necrosis and massive hemorrhages

Page 105: Pat an Anew

105

in adrenals, a necrotic nephrosis in kidneys, a purulent arthritis, an iridocyclitis and vasculitis. Choose the correct diagnosis.A. *MeningococcemiaB. Epidemic typhusC. Periarteritis nodosaD. Systemic lupus erythematosusE. Radiation syndrome5. A 7-year-old child presented with raised body temperature up to 38,5°, rhinitis, conjunctivitis, and cough. A physical investigation revealed blotchy and papular skin rash and whitish bran-like rashes in a buccal mucosa. On the fourth day appeared a dyspnea and wet pulmonary rhonchuses. Soon, difficulty of breath resulted in patient death from asphyxia. What is the most likely diagnosis?A. *MeaslesB. MeningococcemiaC. InfluenzaD. The pulmonary form of a mucoviscidosisE. Pneumonia6. A 7-year-old boy presented to the hospital with an acute catarrh in pharynx and tonsils which had extended on a mouth mucosa, tongue and fauces. A tongue was fiery red (crimson red) with prominent papillae ('raspberry' or 'strawberry' tongue). A gross investigation of tonsils showed superficial necroses. A casting-off necrotic tissue resulted in ulcers formation. Cervical lymphatic nodules were enlarged. A punctuate erythematous rash was determined over the trunk, arms and legs. Face was also involved but a small area about the mouth remained unaffected to produce circumoral pallor. What is the most likely diagnosis?A. * Scarlet feverB. DiphtheriaC. AnginaD. MeaslesE. Meningococcal nasopharyngitis7. An autopsy of 4-year-old child revealed hyperemic, yellowish-greenish arachnoids, infiltrated by purulent and fibrinous exudate resembled a 'cap'. What is the most likely diagnosis?A. * Meningococcal meningitisB. Tubercular meningitisC. Influenzal meningitisD. Anthrax meningitisE. Epidemic typhus meningitis8. An autopsy of a child revealed a necrotic tonsillitis, neck phlegmon, purulent otitis and purulent meningitis. These alterations are the most typical for:A. * Septic scarlet feverB. Meningococcal infectionC. Toxic scarlet feverD. Fauces diphtheriasE. Otogenic sepsis9. A child presented with acute a catarrhal nasopharyngitis. In 2 days he died. An autopsy revealed arachnoids were markedly hyperemic, edematous, infiltrated with turbid, thick, yellowish-greenish liquid. A brain was swelled, cerebellum tonsils were enlarged in volume, and sulcus of the cerebrum impaction was well defined. The described changes are the most typical for:A. * Meningococcal infectionB. InfluenzaC. Whooping coughD. DiphtheriaE. Measles10. A 5-year-old child presented with raised body temperature, punctulated rash, and conjunctivitis. He died of the superimposed pneumonia. A histological research of lungs revealed endo - mezo - panbronchitis and giant-cell pneumonia. For what disease the presented changes are characteristic?A. *MeaslesB. Scarlet feverC. Croupous pneumoniaD. DiphtheriaE. Smallpox11. Л 43-year-old male, with a history of kidney transplantation, followed by immunosuppressive therapy, died of intoxication. A histological investigation of his lungs, kidneys and pancreas revealed strikingly enlarged (giant) cells with basophilic cytoplasm and large nucleus. The violet intranuclear inclusions surround by a small clear halo, reminded an eye of an owl. What is the most likely diagnosis?A. *Cytomegaly

Page 106: Pat an Anew

106

B. TuberculosisC. SyphilisD. LepraE. Plague13. 11 -year-old child presented with a body temperature up to 40°C, an acute headache, vomiting, anxiety and a fever. In four days appeared a hemorrhagic skin rash, an oliguria and adrenal insufficiency, resulted in patient's death. A bacteriological study of smear preparation from a pharynx revealed a meningococcus. What form of disease is found out?A. MeningococcemiaB. Meningococcal meningitisC. MeningocephalitisD. Meningococcal nasopharyngitis E.12. A young 19-year-old man, with an acute onset of disease, died of an infectious-toxic shock. An autopsy revealed the enlarged tonsils, covered with grey-white membranes, which extended on the palatine arch. In addition, an edema of the neck soft tissues determined. Histological investigation showed the necrosis of an epithelium of tonsils and the palatine arch. Tissues underneath were infiltrated by a fibrinous exudate, which formed massive stratifications on a surface. What is the most likely diagnosis?A. * DiphtheriaB. Scarlet feverC. Adenoviral infectionD. Infectious mononucleosisE. Staphylococcal infection14. An oral cavity examination of a 4-year-old child, revealed a hyperemia of the fauces and tonsils mucus membranes. Gross investigation showed enlarged tonsils, covered by dense, whitish-yellowish membranes. They left a raw surface, with deep defect, when forcibly removed. Soft tissues of a neck were edematous, regional lymph nodes were enlarged and painful. What the most likely diagnosis is among listed below?A. * DiphtheriaB. Scarlet feverC. MeaslesD. ParotitisE. Adenoviral infection15. An autopsy of 2-year-old child revealed hemorrhagic enanthesis (skin rash), mild hyperemia and edema of nasopharynx mucous, fine hemorrhages in mucosas and internal organs, severe dystrophy in a liver and myocardium, an acute necrotic nephrosis,'massive hemorrhages in adrenals. What disease is the most likely to present such symptoms?A. * Meningococcal infectionB. Scarlet feverC. DiphtheriaD. MeaslesE. Epidemic typhus16. A child presented to a hospital with an acute fever, sore throat and swelling of submaxillary (submandibular) lymph nodes. A gross investigation of fauces revealed the edematous and slightly hyperemic mucous membrane, enlarged tonsils, covered by the grayish-whitish membranes. They left a raw surface, when forcibly removed. What is the most likely diagnosis?A. * DiphtheriaB. Catarrhal anginaC. Scarlet feverD. Meningococcal infectionE. Measles17. A 7-year-old girl developed an acute disease. She presented to the hospital with a sore throat, and a fever, accompanied with a widespread rash. A physical examination revealed severe hyperemia of a pharynx, a 'raspberry' tongue, and enlarged bright red tonsils with some grayish and yellowish dim areas. These zones also extended on peritonsillar tissues. Submandibular lymph nodes were enlarged. For what disease the given changes are characteristic?A. * Scarlet feverB. MeaslesC. Fauces diphtheriaD. Larynx diphtheriaE. Meningococcal nasopharyngitis18. A 9-year-old child developed a sharp pain at swallowing and a neck edema. His body temperature had risen to 39°C. A motley-red, punctulated , widespread rash appeared. A physical examination revealed severe hyperemia of a pharynx ('fiery pharynx') and a 'raspberry' tongue. The enlarged bright red tonsils presented with some grayish necrotic zones. What is the most likely diagnosis?

Page 107: Pat an Anew

107

A. * Scarlet feverB. Meningococcal nasopharyngitisC. DiphtheriaD. InfluenzaE. Measles19. A 6-year-old boy developed pain at swallowing and raised body temperature. On a second day of the disease, a physical investigation revealed the appearance of the fine, densely set, widespread skin rash with a small unaffected circumoral area. An oral cavity examination showed the bright reddening of a pharynx, enlarged tonsils and raspberry-red tongue. What is the most likely diagnose?A. *Scarlet fever.B. Adenoviral infection.C. Fauces diphtheria.D. Measles.E. Streptococcal angina.20. An 8-year-old child presented with raised body temperature up to 39° C, rhinitis, conjunctivitis, and cough. A physical investigation revealed blotchy skin rash and whitish rashes on a buccal mucosa. Suddenly appeared pant resulted in asphyxia, followed by patient death. What is the most likely diagnosis?A. *MeaslesB. Scarlet feverC. DiphtheriaD. Meningococcal nasopharyngitisE. Influenza21. A 3-year-old girl presented with a sharp pain at the swallowing, a marked neck edema, and the body temperature raised to 40° C. A gross investigation of tonsils revealed the grayish-yellowish membranes, which left a raw surface with bleeding ulceration, when forcibly removed. What is the most likely diagnosis?A. *DiphtheriaB. Scarlet feverC. MeaslesD. Meningococcal nasopharyngitisE. Influenza22. A child died of a spontaneous pheumothorax. A post-mortem revealed a collapse and interstitial emphysema of the right lung, focal atelectases of lungs, a punctual hemorrhage in a brain and a conjunctiva of eyes. What is the most likely diagnosis?A.* Whooping coughB. BronchopneumoniaC. MeaslesD. Diphtheria of respiratory tractsE. Scarlet fever23. A child presented to the clinic with paralysis of a phrenic nerve and dysphagia, resulted in his death soon. An autopsy revealed ulcers of tonsils with a cuticularization, a hemorrhage in adrenals, and flaccidity of a myocardium. Specify disease for which the described changes and complications are characteristic.A.* DiphtheriaB. Scarlet feverC. AgranulocytosisD. MeaslesE. Poliomyelitis24. A child complains of the general weakness, a pharyngalgia at swallowing and nausea. Objectively, a bright reddening revealed in fauces; tonsils were enlarged. Later, the fine, densely located maculae, size about papaverous grain, appeared on a neck, in the top of a back and a breast. Microscopical investigation showed in a mucosa and tonsils tissue an acute plethora and grayish, dim areas of necroses. On the periphery of the necrotic zones, within the edema and fibrinous inflammation areas, the chains of streptococcuses were defined. What is the most likely diagnosis?A. * Scarlet feverB. DiphtheriaC. MeaslesD. InfluenzaE. Meningococcal nasopharyngitis25. A 7-year-old child presented to the hospital with a punctuated bright-pink rash on hyperemic skin of a forehead, a neck, in the bottom of abdomen and popliteal fossa. A nasolabial triangle remained pale. Physical investigation of a stomato-pharynx revealed circumscribed bright red hyperemia of mucosa, 'raspberry' tongue. His tonsils were friable and edematous; lacunas filled with pus. Lymph nodes of a neck were enlarged, dense and painful. What is the most likely diagnosis?A. *Scarlet fever

Page 108: Pat an Anew

108

B. RubellaC. Whooping coughD. DiphtheriaE. Infectious mononucleosis26. A child with a history of an acute catarrh of the upper respiratory tract, conjunctivas and blotchy, papular skin rash, died of pulmonary insufficiency. A histological investigation revealed an interstitial pneumonia with focal serous macrophageal inflammation and set of giant cells within alveoli walls. What is the most likely diagnosis?A. *MeaslesB. InfluenzaC. ParainfluenzaD. Adenoviral infectionE. Cytomegaly27. A 4-year-old child died of respiratory insufficiency as a result of a respiratory musculation paralysis. Histological investigation revealed a hyperemia of the moracal part of a spinal cord, the unclear pattern of the grey substance, punctual hemorrhages and fine sinking down sites of an encephalomalacia. In addition, there was an inflammation with a neuroglia proliferation around and by the necrotic neurons. What is the most likely diagnosis?A. *PoliomyelitisB. Meningococcal infectionC. CytomegalyD. ToxoplasmosisE. Adenoviral infectionBACTERIAL INFECTIONS1. An autopsy of a 54-year-old man revealed a fibrinous inflammation at colon. What is your diagnosis? A *DysenteryB. AmebiasisC. Typhoid feverD. CholeraE. Balantidiasis2 A patient, the inhabitant of a seaside city in the south of the country, presented to the hospital with severe diarrheas and vomiting, resulted in his death from dehydration. An autopsy revealed an acute gastroenteritis with serous-desquamative inflammation in a small bowel. Choose the most likely diagnosis. A *CholeraB. Bacterial dysenteryC. TyphoidD. AmebiasisE. Salmonellosis3. A 52-year-old woman died in surgery department of a diffuse purulent peritonitis. A post-mortem revealed in a distal part of a small intestine enlarged Peyer's patches, which projected into the intestine's lumen. There surface was covered by brownish-greenish scab. In the centre of some patches were found deep defects, extended to a serous layer. For what disease described intestinal changes are characteristic? A *TyphoidB. DysenteryC. An intestine tuberculosisD. Crohn's diseaseE. Staphylococcal enteritis4. A 38-year-old man died of diffuse fibrinopurulent peritonitis. A post-mortem revealed an ulcer in a small bowel, which replicated a form of a Peyer's patch and perforated the intestine's wall. Microscopical investigation showed an alteration of lymphoid tissue pattern with replacement of lymphocytes by proliferating monocytes, followed by granulomas formation. The complication of what disease resulted in patient's death? A *Typhoid.B. Dysenteries.C. Cholera.D. BrucellosisE. Nonspecific ulcerative colitis.5. An autopsy of a 48-year-old man, with a history of typhoid fever, revealed groups of enlarged lymphoid follicles in small bowel. The follicles protruded above the mucosal surface; they had a grey-red coloring, juicy appearance and a surface, which reminded gyri and sulci. Microscopical investigation showed granulomas formation. Specify, what of the listed stages of typhoid is the most likely?A *Medullar swelling.B. Necrotic.C. Formation of ulcers.D. Pure ulcers.E. Healing.6. An autopsy of a child, with a history of diarrhea, revealed an exicosis and a widespread fibrinous colitis. A mucosal smear imprint study showed gram-negative rod bacteria. What is your diagnosis?

Page 109: Pat an Anew

109

A *DysenteryB. CholeraC. Staphylococcal intestinal infectionD. TyphoidE. Salmonellosis7. A patient presented with a sustained fever, enlarged and painful spleen. On the 10th day of a disease a rash of flat, rose-colored spots appeared. On the 21st hospital day he died after developing peritonitis. An autopsy revealed in the ileum deep ulcers within the area of necrotized group of lymphoid follicles (Peyer's patches). One of ulcers was perforated; there was a flbrinopurulent diffuse peritonitis. What is the most likely disease? A *TyphoidB. DysenteryC. Intestine amebiasisD. CholeraE. Salmonellosis8A post-mortem of a 57-year-old male, with a history of typhoid, revealed a dense, fragile whitish-yellowish color of the rectus abdominis and a hip muscles. They reminded a stearin candle. The described changes are the result of: A *Zenker's necrosisB. Fibrinoid necrosisC. Caseous necrosisD. Colliqative necrosisE. Apoptosis9. Histological investigation of a rectum revealed large areas of necrosis in mucosa. Necrotic masses were impregnated by fibrin, resulting in membrane formation. Mucous and submucous on periphery of necrotic zones were hyperemic, edematous, with hemorrhages and leukocytic infiltration. What is the most likely diagnosis?A *DysenteryB. CholeraC. AmebiasisD. TyphoidE. Salmonellosis10. A patient died of a diffuse peritonitis. An autopsy revealed in the distal part of a small bowel multiple oval ulcers with rounded edges, which were parallel to the longitudinal axis of the intestine. A bottom of ulcers was clean, smooth and glitter. Base of the ulcers presented either by muscular layer of ileum or its serosa. Two ulcers had perforation apertures 0,5cm in diameter each. What disease resulted on a death of a patient?A * TyphoidB. DysenteryC. CholeraD. TuberculosisE. Epidemic typhus11. A 44-year-old man presented to a surgery department with diffuse flbrinopurulent peritonitis. An operation revealed an ulcer of ileum with a perforation. An ulcer closure, followed by abdomen cavity drainage was performed. Four days later patient died. At post-mortem iliac ulcers of Peyer's patches, parallel to the longitudinal axis of the intestine, were found. A base of the ulcers was presented by serosa. For what disease such changes are characteristic? A *TyphoidB. An intestine tuberculosisC. DysenteryD. Intestinal tumoursE. An intestine amebiasis12 A man, with 4 weeks history of the typhoid, presented with symptoms of 'acute abdomen'. He died soon of intoxication. At post-mortem a pathologist found a perforation of iliac wall and diffuse purulent peritonitis. For what stage of disease the described complication is characteristic? A * Stage of pure ulcersB. Stage of a medullar swellingC. Necrotic stageD. Stage of healing of ulcersE. Bacteriemia13. A 42-year-old patient, with a history of dysentery, presented with symptoms of paraproctitis. What is the most probable stage of local changes presented in that case?A * Formation of ulcersB. Fibrinous colitisC. Follicular colitisD. Catarrhal colitisE. Stage of healing

Page 110: Pat an Anew

110

14. A child presented with diphtheria and died on the 2nd week of the disease from an acute heart failure. A post-mortem revealed numerous fine centres of a myolysis, accompanied with some perifocal lymphoid infiltration. What became a cause of death?A * Myocarditis, caused by bacterial ectotoxinB. Bacterial myocarditisC. Septic myocarditisD. Myocardial infarctionE. Metabolic myocardial necrosis15 A 39-year-old patient died on the 4th day after operation concerning perforation of a colon wall, resulted in diffuse purulent peritonitis. An autopsy revealed the mucosa of a colon wall thickened, covered by fibrinous membrane. Some single ulcers got on different depth. Histological study of a colon wall showed a necrosis of mucous, a fibrinous inflammation, infiltration by leucocytes with focuses of hemorrhages. What complication of disease became a cause of patient's death? A *Dysentery. B. Typhoid.C. Nonspecific ulcerative colitis.D. Crohn's diseaseE. Amebiasis.16 A 45-year-old man presented in the infectious diseases hospital with a profuse diarrhea, exicosis, acute decline of his body temperature. He died soon of an uremia. An autopsy revealed in his small bowel lumen a colorless liquid in the form of rice broth and edematous mucosa. Microscopical investigation of a small bowel showed a hyperemia of vessels, a focal hemorrhage, a desquamation ofenterocytes, a hypersecretion of a beaker (goblet) cells and lympho-leucocytes infiltration of a mucosal stroma. What is the most likely diagnosis? A *CholeraB. SalmonellosisC. DysenteryD. TyphoidE. Crohn's disease17. An autopsy of 47- year-old man revealed in sigmoid and rectum plural red ulcers with irregular form, A mucous between ulcers was covered by dirty grey membrane. What is the most likely etiology of the disease?A *ShigellaB. AmebaC. Mycobacterium tuberculosisD. SalmonellaE. Staphilococcus18. The worker of a cattle-breeding farm died in 4, 5 months of the disease beginning. A post-mortem revealed in his internal organs the granulomas consisted of epithelioid, giant Pirogov- Langhans cells, plasmocytes and eosinophyles. There were many vessels within a granuloma. A histological investigation showed a productive-destructive vasculitis in the vessels of internal organs. What is the most likely disease?A * BrucellosisB. AnthraxC. TuberculosisD. Epidemic typhusE. Lepra19. The worker of a stockbreeding farm presented to the hospital with acute disease and died soon of intoxication. An autopsy revealed the enlarged, flaccid spleen, which had a dark-cherry colouring of its cut surface. The scrape of the spleen's pulp was plentiful. Arachnoids of the brain's fornix and base were edematous, impregnated by blood, had a dark red colouring ("cardinal's hat"). Microscopical investigation found the serous- hemerrhagic inflammation of brain's membranes and tissues with destruction of fine vessels walls. Diagnose a disease.A * Anthrax B. TularemiaC. BrucellosisD. PlagueE. Cholera20. An autopsy of a 53-year-old man revealed few 4-5cm ulcers. The margins were elevated above the mucosa. The walls of ulcers were covered by the yellowish-grayish crumble masses. The Widal test was positive. Diagnose a disease?A * TyphoidB. ParatyphoidC. Recurrent typhusD. DysenteryE. Crohn's disease

Page 111: Pat an Anew

111

21. A histological investigation of a myocardium revealed fatty dystrophy and plural necroses of cardiomyocytes, few focal cellular interstitial infiltrates. A medical record showed that a child had diphtheria, resulted in the heart failure. What kind of myocarditis is the most likely?A * Alterative myocarditisB. Diffusive exudative myocarditisC. Focal exudative myocarditisD. Intersticial myocarditisE. Granulomatous myocarditis22. An autopsy of a 45-year-old man, who died on the 5th day of typhoid fever, revealed groups of enlarged, hyperemic lymphoid follicles in small bowel. The follicles protruded above the mucosal surface; their surface reminded brain's gyri and sulci. Microscopical investigation showed hyperemia, edema and granulomas formation. Specify, what period of local changes of typhoid is the most likely?A * Stage of medullar swellingB. Necrotic stageC. Stage of healingD. Stage of pure ulcersE. Stage of ulcers' formation23. An autopsy 54-year-old man revealed markedly dilated lumen of a small bowel filled with a liquid, which reminded "rice broth". The intestine wall was edematous with multiple petechial hemorrhages. For what infectious disease the described enteritis is characteristic?A *CholeraB. DysenteryC. SalmonellosisD. AmebiasisE. Typhoid24. A histological investigation of a skin biopsy showed serous-hemorrhagic inflammation and a focus of necrosis. His medical records revealed the beginning of the disease from a small red maculae formation with the bubble in the centre, filled by serous-hemorrhagic liquid. Subsequently the central part became black. What disease is the most probable? A *Carbuncle at a malignant anthraxB. Actinomycosis of skinC. Allergic dermatitisD. Streptococcal carbuncleE. Chemical dermatitis25. A 48-year-old patient with severe typhoid developed an acute renal failure, resulted in a lethal outcome of a disease. An autopsy revealed enlarged and swollen kidneys. On a cut section, the cortex was pale grey, while pyramids were dark red. A histological investigation showed that in the majority of tubules the lumen was narrowed, epithelial Cells were enlarged and lost their nuclei; glomeruli were collapsed. A renal stroma was edematous with a small leukocytic infiltration and fine hemorrhage. What pathology of kidneys presented in that case?A *NecronephrosisB. Acute pyelonephritisC. Acute glomerulonephritisD. PyonephrosisE. Hydronephrosis26. An autopsy revealed groups of enlarged, hyperemic lymphoid follicles in small bowel. The follicles protruded above the mucosal surface; their surface reminded brain's gyri and sulci. The cut surface of follicles was succulent; with gray-red coloring. Microscopical investigation showed monocytes and histiocytes proliferation. Macrophages formed aggregates (typhoid granulomas). Name a morphological stage of typhoid.A *Medullar swellingB. NecroticC. Formation of ulcersD. Pure ulcersE. Healing27. A 48-year-old butcher died of a sepsis. A gross investigation of his right cheek revealed a dense, dark red, 6 cm in diameter, corn-shape infiltrate with a black crust in its centre. The right half of his face and neck were edematous and solid. A microscopical investigation of a lesion showed the peracute serous- hemorrhagic inflammation with epidermal and adjacent tissue necrosis within the central zone of an infiltrate. What is the most likely diagnosis?A * AnthraxB. PlagueC. TularemiaD. Phlegmon of a neckE. Furuncle28. An autopsy of a 45-year-old man revealed in his small bowel the enlarged, hyperemic groups of lymphoid follicles. The follicles protruded above the mucosal surface; their surface reminded child's brain gyri and

Page 112: Pat an Anew

112

sulci. Microscopical investigation showed monocytes, histiocytes and reticular cells proliferation. Macrophages formed aggregates (typhoid granulomas), which replaced lymphocytes. For what disease the described changes are characteristic? A * TyphoidB. CholeraC. DysenteryD. SalmonellosisE. Amebiasis29. A 38-year-old patient was on mountain pastures and then presented to the hospital in severe condition, with high body temperature. A physical investigation revealed the lymph nodes were markedly enlarged, soldered to surrounding tissue, motionless, a skin over them was red and very painful. A microscopical study of a lymph node showed the peracute serous-hemorrhagic inflammation. For what disease it is characteristic?A * PlagueB. TularemiaC. SyphilisD. BrucellosisE. Anthrax30. A 39-year-old milkmaid died of cardiovascular insufficiency. A post-mortem reveled in the aortal valve a polypous -ulcerative endocarditis. A microscopical study of the valve showed the polymorph cellular inflammatory infiltration, zones of destruction and thrombuses with the organization. In addition, a sclerosis with granulomas, consisted of randomly located epithelioid, giant, plasmatic, eosinophyie cells were diagnosed within a stroma of a myocardium. A vasculitis was also determined in the heart vessels. For what disease the described changes in heart are characteristic?A * BrucellosisB. Chronic sepsisC. Rheumatic diseaseD. Systemic lupus erythematosusE. Systemic scleroderma31. A 42-year-old woman, who worked at a fur atelier, suddenly died. An autopsy revealed dark red, impregnated by blood arachnoids of the brain's fornix and base, which reminded a red cap. Microscopical investigation found the serous-hemorrhagic inflammation of brain's membranes and tissues with necrosis of fine vessels walls and numerous hemorrhages. What is the most likely diagnosis?A * AnthraxB. Cerebral hemorrhage in case of hypertension diseaseC. Cerebral hemorrhage due to traumaD. Meningococcal infectionE. Tubercular leptomeningitis32. An autopsy of an 8-year-old child revealed in his colon multiple irregular form and various depth defects with uneven edges. In addition, there were grey-white membranes intimately connected with tissues underneath. What is the most likely diagnosis? A * DysenteryB. SalmonellosisC. CholeraD. TyphoidE. Amebiasis33. An autopsy of a 65-year-old man , who died in week from the beginning of a profuse diarrhea, revealed severe exicosis, dry all tissues and thick, concentrated blood. A bacteriological study of contents of a small bowel, which reminded the rice broth, determined vibrioes. What disease resulted in the patient's death?A * CholeraB. DysenteryC. TyphoidD. SalmonellosisE. Alimentary toxic infection34. A male patient, with 5 days history of diarrhea, had a colonoscopy. The investigation revealed a colon mucosa inflammation with grey-green membranes, intimately connected with tissue underneath. What is the most likely diagnosis?A * DysenteryB. TyphoidC. Nonspecific ulcerative colitisD. SalmonellosisE. Crohn's disease35. An autopsy of the patient who died of a plague revealed a hemorrhagic syndrome, accompanied with the hemorrhagic necrosis of a hip skin, a lymphangitis and an inguinal hemorrhagic lymphadenitis. Name the plague form. A * Dermo-bubonic

Page 113: Pat an Anew

113

B. BubonicC. Primary septicD. Primary pulmonicE. Hemorrhagic36. A 38-year-old man died of intoxication. A post-mortem revealed in a small bowel an edema of lymphoid follicles groups. The follicles protruded above the mucosal surface like a soft plaques with irregular surface pattern, which reminded brain's gyri and sulci. What diagnosis is most probable?A * TyphoidB. DysenteryC. SalmonellosisD. Acute enteritisE. Cholera37. A young man died of intoxication on 4th day after eating the crude eggs. An autopsy revealed the stomach and small bowel mucosa inflamed, covered by a mucous exudate. In addition, abscesses were found in lungs, brain and liver. What diagnosis is the most likely? A *Salmonellosis (septic form)B. DysenteryC. Salmonellosis (typhoid form)D. Salmonellosis (intestinal form)E. Typhoid38. An autopsy of a 52- year-old man revealed in sigmoid and rectum plural brown-green membranes, hemorrhages,some blood in the lumen of intestine. Histological study showed a fibrinous colitis. Bacteriological investigation found S. Sonne. What is the most likely diagnosis? A * DysenteryB. CholeraC. SalmonellosisD. YersiniosisE. Crohn's disease39. A histological research of an autopsy material revealed in a brain tissue a wide zone of proliferating microglia cells round small vessels. In addition, the marked proliferation of an endothelium, an adventitia and pericytes were determined in the skin vessels. Perivascular moderate lymphocytes infiltration with single neutrophiles added was also found. The vessels walls underwent various signs of destruction. Name the causative organism of described infection disease?A. * RickettsiaB. SpirocheteC. MeningococcusD. StreptococcusE. Human immunodeficiency virus40. A 7-year-old child, on the 3ld week of disease, presented with hectic fever, skin and sclera icterus. He died soon of the hepatorenal failure. An autopsy reveled in a rectum many irregular form defects with uneven edges. These defects were covered by grey-white membranes, intimately soldered to a tissue underneath. In addition, there was a purulent thrombophlebitis of the intestinal vessels. Plural pylephlebitic abscesses and fatty dystrophy of hepatocytes were diagnosed in a liver. An acute necrotic nephrosis was found in kidneys. What disease caused a lethal outcome?A. *DysenteryB. SalmonellosisC. CholeraD. TyphoidE. Amebiasis41. A worker of the agricultural enterprise presented with an acute disease and died soon of intoxication. An autopsy revealed the enlarged and flaccid spleen with a dark-cherry coloring of a cut surface. There were a superfluous pulp's scrape from the lien cut surface. Arachnoids of the brain's fornix and base were edematous, impregnated by blood ('red cap' or 'cardinal's hat'). Microscopical investigation found the serous- hemorrhagic inflammation of brain's membranes and cerebral tissues. What is the most likely diagnosis?A. * AnthraxB. TularemiaC. PlagueD. CholeraE. Brucellosis42. An autopsy at the Forensic Pathology Department of an unknown man, with a history of alcohol intoxication, revealed in his small bowel the enlarged Peyer's patches, which protruded above the mucosal surface. They were soft, with irregular surface, which reminded brain's gyri and sulci. The cut surface had a gray-red coloring. For what disease the described changes are characteristic?A. * TyphoidB. DysenteryC. Lamb lias is

Page 114: Pat an Anew

114

D. AmehiasisE. Enteritis due to Campylobacter43. A post-mortem of a patient, who died of diffuse fibropurulent peritonitis, revealed in the wall of a small bowel multiple oval ulcers with rounded edges, which were parallel to the longitudinal axis of the intestine. A base of the ulcers presented either by muscular layer of ileum or its serosa. Two ulcers had perforation apertures 0,3cm in diameter each. Define a typhoid stage.A. * Stage of pure ulcersB. Stage of "dirty ulcers" formationC. Necrotic stageD. Stage of a medullar swellingE. Stage of healing44. A 65 -year-old patient addressed to a doctor his complains to frequent urge and painful passage of stool, with mucous and some blood in feces. Proctosigmoidoscopy revealed marked narrowing of a colon's lumen, especially in its distal part. A mucosa was edematous, hyperemic, with areas of necrosis and hemorrhages. A bacteriological study of feces determined the Shigella flexneri. Define the disease?A. * DysenteryB. TyphoidC. AnthraxD. Crohn's diseaseE. Paratyphoid

TUBERCULOSIS1. A histological investigation of the cervical lymph node revealed the aggregation of epithelioid cells, lymphocytes and giant Pirogov - Langhans cells. In the centre of a lesion a caseous necrosis was present. Specify the most likely pathology.A. * TuberculosisB. Rhinoscleroma.C. Sarcoidosis (Boeck's disease)D. GlandersE. Syphilis2. An autopsy of a 42-year-old man revealed in the second segment of the right lung the focus of consolidation, 5 cm in diameter, surrounded with a thin capsule. The centre was presented by a dense dry crumbling tissue with a dim surface. Morphological changes in a lung are characteristic for:A. TuberculomaB. Carcinoma of lungC. ChondromasD. The tumorous formof a silicosisE. Postinflammation pneumosclerosis3. An autopsy of a 63-year-old man revealed in the second segment of the right lung a tubercular panbronchitis, with the focus of a caseous bronchopneumonia. The focus was surrounded by a wall of epithelioid cells with some lymphocytes and few giant Pirogov -Langhans cells. What form of a pulmonary tuberculosis presented in this case?A. *Acute pulmonary tuberculosisB. Primary tuberculous complexC. Growth of primary affectD. Acute miliary tuberculosisE. Infiltration tuberculosis4. A 26-year-old patient arrived in a hospital with complaints to a fever and weakness. One of the enlarged cervical lymph nodes was taken for histological study. Microscopical investigation showed in lymphoid tissue the centers of necrosis, surrounded with epithelioid cells, giant multinuclear Pirogov-Langhans cells and lymphocytes. What is the most likely disease?A. * TuberculosisB. Lymphogranulomatosis (Hodgkin's disease)C. Lymphatic leukemiaD. Sarcoidosis (Boeck's disease)E. Syphilis5. An autopsy of a 44-year-old man revealed tuberculosis. A macroscopical investigation showed in the apex of the right lung the foci of caseous pneumonia. There were caseous lymphadenitis of enlarged lymph nodes of mediastinum and plural milium nodules in many organs. Name the described form of tuberculosis? A. * Primary with mixed form of progressing B. Primary with gematogenic spreadingC. Primary with lymphogenic spreadingD. Primary with growth of a primary affect

Page 115: Pat an Anew

115

E. Gematogenic6. A 7-year-old boy with a history of tonsillitis presented with markedly enlarged paratracheal, bifurcate and cervical lymph nodes. A microscopical investigation of the removed cervical lymph node revealed the centers of a necrosis surrounded with lymphocytes, epithelioid cells and Pirogov-Langhans cells. Specify the most likely pathology.A. *TuberculosisB. Sarcoidosis (Boeck's disease)C. Rhinoscleroma.D. GlandersE. Syphilis7. A 40- year-old man died of a pulmonary bleeding. An autopsy revealed in the upper lobe of the right lung a cavity, 4cm in diameter, with dense walls and a rough internal surface, filled with blood clots. A microscopical study of the wall showed the inner layer, presented with fell to pieces leucocytes; the middle layer formed by epithelioid, lymphoid and giant multinuclear cells with nuclei located like a horseshoe. The changes found out in a lung are characteristic for:A. Tuberculous cavern (cavity)B. Bronchiectatic cavern (cavity)C. Abscess of lungsD. Congenital cystE. Infarct of lung with septic disintegration8. A post-mortem of a 43-year-old man revealed in the 3 segment of the right lung under the pleura an area of consolidation, 1,5 cm in diameter, with the accurate borders, surrounded with a whitish fibrous tissue. A gross investigation of a cut section showed the white-yellow crumbling foci. Presence of the described focus is characteristic for:A. Encapsulation of primary affectB. Peripheral carcinomaC. ChondromaD. FibromaE. Infarct of lung with organization9. A 48- year-old man died of a pulmonary -heart failure. An autopsy revealed pneumosclerosis, emphysema of lungs, and hypertrophy of a right ventricle of heart. In both lungs, mainly under pleura, there were plural foci, 1 cm in diameter. Histological investigation of the foci showed a necrosis zone in the centre and a wall of epithelioid cells, lymphocytes with some macrophages and plasmocytes on periphery. Giant Pirogov -Langhans cells were also defined. The small numbers of vascular capillaries were present on a periphery of the foci. What is the most likely disease?A. Gematogenic tuberculosis.B. Actinomycosis of lungs.C. Sarcoidosis (Boeck's disease)D. Syphilis.E. Silicosis.10. A 56-year-old man with a long history of fibrous - cavernous pulmonary tuberculosis died of a chronic pulmonary and heart failure. Within last months the expressed proteinuria was observed. A post-mortem revealed enlarged, firm kidneys, with a waxy surface. What changes developed in kidneys at this form of tuberculosis?A. Amyloidosis of kidneyB. Tuberculosis of kidneyC. GlomerulonephritisD. NephrolithiasisE. Necrotic nephrosis] 1. A child died of a diffuse peritonitis. An autopsy revealed the primary intestinal tubercular complex with primary affect, a lymphangitis and regional caseous lymphadenitis. In addition, an ulcer of an empty intestine with perforation was found within the primary affect focus. What is the most likely way of tuberculosis infection in presented case?A. Alimentary (nutritional)B. TransplacentalC. Aero genieD. MixedE. Contact12. A 24-year-old man presented with enlarged cervical lymphatic node. A histological investigation revealed the growth of epithelioid, lymphoid cells and macrophages with nuclei in the form of a horseshoe. In some centers the aggregates of unstructured light pink color masses with nuclei fragments were determined. For what disease the described changes are characteristic?A. TuberculosisB. Lymphogranulomatosis (Hodgkin's disease)

Page 116: Pat an Anew

116

C. ActinomycosisD. Metastasis of tumorE. Syphilis13. A patient with a history of fibrous - cavernous tuberculosis died of a renal failure. A post-mortem revealed a urine smell, left ventricle hypertrophy, fibrinous pericarditis, and fibrinous - hemorrhagic enterocolitis. Kidneys were small, reduced in sizes, dense, with plural foci of shrinkage. Histological investigation of the slides with Congo-Red staining showed pink masses within glomuleruli and vessels walls. In addition, destruction and an atrophy of the majority of nephrons, accompanied with a nephrosclerosis were determined. What is the most likely pathology of kidneys?A. Amyloid- contracted kidneysB. Contracted kidneyC. Arteriolosclerotic kidneyD. Atherosclerotic contracted kidneysE. Pielonephritic contracted kidneys14. An autopsy of a 7-year-old child revealed in the apex of the right lung, under the pleura, a focus of caseous necrosis, 15 mm in diameter. There were also enlarged bifurcational lymph nodes with small necrotic foci. A microscopical investigation of the pulmonal focus and the lymph node's lesion showed necrotic masses surrounded by epithelioid cells, lymphocytes and solitary giant multinuclear Pirogov- Langhans' cells. What is the most likely disease?A. *Primary tuberculosisB. Gematogenic tuberculosis with the lesions of lungsC. Gematogenic generalized tuberculosisD. Secondary inflammatory tuberculosisE. Secondary fibre-inflammatory tuberculosis15. A post-mortem of a 58-year-old patient revealed plural pathological cavities in both lungs. Microscopical study showed the inner layer of one cavity presented with necrotic masses and diffuse neutrophil infiltration; the middle one contained an infiltrate of epithelioid cells, lymphocytes and multinuclear giant cells. The external layer consisted of a mature connective tissue. Diagnose the form of secondary tuberculosis.A. *Fibrocavernous tuberculosisB. Acute inflammatoryC. Fibre-inflammatoryD. Acute cavernousE. Cirrhotic tuberculosis16. An autopsy of a 74- year-old man revealed the enlargement and deformation of the right knee joint. A histological investigation determined the massive foci of a caseous necrosis, surrounded by peripheral mantle of epithelioid, lymphoid cells and some multinucleated giant cells.What is the most likely disease?A. *Tuberculous arthritisB. Syphilitic arthritisC. Rheumatic arthritisD. Gonorrheal arthritisE. Deforming osteoarthrosis17. A 52- year-old man with a long history of tuberculosis died of a bleeding from lungs. An autopsy revealed in lungs some oval and round cavities. The walls of cavities were presented with necrotic masses and a pulmonal tissue. What is the most likely form of tuberculosis presented in that case?A. * Acute cavernousB. FibrocavernousC. TuberculomaD. Caseous pneumoniaE. Acute inflammatory18. A post-mortem of a 58-year-old patient, with a history of tuberculosis, revealed in the upper lobe of the right lung a cavity, 3x2cm in sizes, which connected with a bronchus. A cavity had dense walls and three layers. The internal layer was pyogenic; the middle one was presented with tubercular granulation tissue and the external coating of connective tissue. What of the listed diagnoses the most likely?A. * Fibrocavernous tuberculosis.B. Fibrous focal tuberculosisC. Tuberculoma.D. Acute focal tuberculosis.E. Acute cavernous tuberculosis19. A 42-year-old patient complained of a voice timbre change. The larynx biopsy was taken. A histological investigation revealed the aggregates of large, oblong form cells with light nuclei, rich with a cytoplasm. The nuclei were located on periphery in the form of a horseshoe. The described histological changes are characteristic for:A. *Tuberculosis

Page 117: Pat an Anew

117

B. LeprosyC. SyphilisD. RhinoscleromaE. Lymphogranulomatosis (Hodgkin's disease)20. A histological investigation of the enlarged cervical lymph nodes revealed the nodules consisted mainly of flat, slightly extended cells with pale nucleus. There were also giant cells with spherical-oval form, light pink cytoplasm and with nuclei located on a cell periphery. The centre of some nodules was presented with the unstructured, light pink colored masses. The described changes are characteristicfor:A. TuberculosisB. Lymphogranulomatosis (Hodgkin's disease)C. Sarcoidosis (Boeck's disease)D. BrucellosisE. Nonspecific hyperplasic lymphadenitis21. A histological investigation of the enlarged supraclavicular lymph nodes, removed from a 40-year-old woman, revealed an alteration of their structure with aggregates of slightly extended cells with a light nucleus. The multinuclear, large, oval-round form cells were also determined. These cells were rich with a cytoplasm and theirnuclei were located on periphery in the form of a paling. The lymphoid cells created the next layer. The unstructured pinkish masses were seen within the centers of lesions. What the described changes are characteristic for?A. Tuberculous lymphadenitisB. Metastasis of tumorC. Lymphogranulomatosis (Hodgkin's disease)D. Hyperplasic lymphadenitisE. Sarcoidosis (Boeck's disease)22. A lung lobe was removed at Surgery Department due to preliminary diagnosed purulent inflammation of a cyst. A gross investigation of the operative material revealed within the 8 segment a dense focus with irregular roundish form, 1,8 cm in diameter. On a cut, it had white-yellow coloring with a dim surface and fine sites of osteal density. The described macroscopical picture characteristic for:A. * Tuberculous affect with healingB. Peripheral carcinoma of lungC. ChondromasD. CarnificationE. Chronic pneumonia23. A 52-year-old man with a long history of a tubercular prostatitis died of a meningocephalitis. An autopsy revealed in arachnoids of the basis and lateral surfaces of a brain, in a spleen, kidneys and liver a considerable quantity of dense grey color nodules 0, 5-lmm in diameter. Histological investigation of nodules showed epithelioid, lymphoid and some giant cells with nuclei located on a cell periphery in the form of a horseshoe. What is the most likely diagnosis?A. *Acute milliary tuberculosisB. Acute tuberculous sepsisC. Chronic milliary tuberculosisD. Disseminated tuberculosisE. Septicopyemia24. A post-mortem of a 40-year-old patient, with a 10 years history of tuberculosis, revealed a cavity in the 1 and Пsegments of his right lung. It had dense walls. The inner layer was rough, with beams formed by the obliterated bronchi and the thrombosed vessels. The middle and lower lobes were firm, with the yellow foci on a cut surface. What form of tuberculosis is the most likely in that case?A. * FibrocavernousB. Acute cavernousC. Fibrous inflammatoryD. TuberculomaE. Cirrhotic25. A 40-year-old prisoner died in penitentiary of tuberculosis. An autopsy revealed a deformation and reduction of the both lungs apexes, plural cavities with dense, 2-3 mm thick walls in the upper lobes of both lungs. In addition, disseminated foci of a caseous necrosis, 5mm-2 cm in diameter, were found in the lower lobes of lungs. Diagnose the tuberculosis form.A. * Secondary fibrocavernousB. Secondary fibre-inflammatoryC. Gematogenic inflammatory with lesion of lungsD. Primary, growth of primary affectE. Secondary cirrhotic26. A 10-year-old girl arrived in traumatological unit with symptoms of pathological fracture of the right femur. A histological investigation of an operational material revealed in a bone marrow of a femur

Page 118: Pat an Anew

118

fragment, a plural foci of a caseous necrosis with peripheral mantle of epithelioid cells, lymphocytes and solitary Pirogov-Langhans' giant cells. Zones of a caseous necrosis extended on adjoining sites of a bone tissue, resulting in osseous destruction. What is the most likely disease?A. *Tuberculous osteomyelitisB. Tuberculous spondylitisC. Tuberculous koxitisD. General(common) macrofocal tuberculosisE. Secondary acute focal tuberculosis27. An autopsy of a 48-year-old man revealed in the first segment of the right lung the round formation, 5 cm in diameter, surrounded with a thin connective tissue layer. The centre was presented by write, brittle masses. Diagnose the form of secondary tuberculosis.A. * Tuberculoma.B. Caseous pneumoniaC. Acute cavernous tuberculosisD. Acute inflammatory tuberculosisE. Fibrocavernous tuberculosis28. A post-mortem of the 17-year-old girl, who died of a pulmonary insufficiency, revealed confluent zones of a caseous necrosis in the lower lobe of the right lung. The caseous necrosis was also determined in broncho-pulmonal, bronchial and bifurcational lymph nodes. What of the listed diagnoses the most likely?A. *Growth primary affect at primary tuberculosisB. Gematogenic form of progress of primary tuberculosisC. Gematogenic tuberculosis with lesion of lungsD. TuberculomaE. Caseous pneumonia at the secondary tuberculosis29. A 50-year-old man with a history of tuberculosis died of a chronic pulmonary and heart failure. A post-mortem revealed a lobar lesion of the right lung. The upper lobe of the right lung was enlarged, dense, with fibrinous membranes on pleura. A gross investigation of a cut surface showed the yellow brittle tissue. What is the most likely form of secondary tuberculosis described in that case?A. *Caseous pneumoniaB. Fibrous-focal tuberculosisC. Infiltrative tuberculosis.D. TuberculomaE. Acute focal tuberculosis30. An autopsy of a man with a history of tuberculosis revealed in the second segment of the right lung the white-grey focus, 3 cm in diameter, surrounded with a capsule. Microscopic study showed a focus of necrosis with a capsule and lack of a perifocal inflammation. Name the tuberculosis form.A. * TuberculomaB. Acute cavernous tuberculosisC. Fibrocavernous tuberculosis.D. Cirrhotic tuberculosisE. Caseous pneumonia31. A died woman had a chronic pulmonal abscess in her medical records. An autopsy revealed in 2nd segment of the right lung a roundish cavity, 5cm in the size. The internal surface of a cavity was created by caseous masses; the external one - by a dense pulmonal tissue. A pyogenic membrane was absent. What is the most likely pathology?A. * Acute cavernous tuberculosisB. Fibrocavernous tuberculosis.C. Acute abscess.D. Chronic abscess.E. Primary pulmonary cavity (cavern).32. A lungs roentgenograrn investigation revealed a dark patch. During a diagnostic express biopsy of a lymph node of a bronchus was removed. A histological study showed a caseous tissue necrosis, round which lied epithelioid and lymphoid layers with few multinuclear large cells (Langhans' cells). Specify the cause of the lymphadenitis.A. * TuberculosisB. PneumoniaC. SyphilisD. Metastases of cancerE. Adenoviral infection33. A 68-year-old woman, with a 20 years history of fibrous - cavernous pulmonary tuberculosis, presented to the Department of Nephrology with symptoms of uremia, Intravital test on amyloid in kidneys appeared positive. What is the most likely form of an amyloidosis presented in this case?A. * Secondary systemicB. Primary systemicC. Limited (local)

Page 119: Pat an Anew

119

D. Familial congenitalE. Senile (gerontal)34. An 8-year-old child, with no disease in his medical history, presented to the hospital with a dyspnea. An x-ray inspection showed the air and a liquid in a pleural cavity. During operation in the right lung the subpleural cavity, 2,4 cm in diameter was found. It had an irregular form with the residual caseous masses inside. The lesion was connected with the pleural cavity. The similar caseous masses were found at a cut of the enlarged radical lymph nodes. A histological research of a cavity's wall determined lymphocytes, epithelioid cells and multinuclear giant cells. How correctly to name a cavity?A. * Primary pulmonary cavity (cavern) at tuberculosisB. Abscess of lungsC. Empyema of pleuraD. Cavity (cavern) at acute cavernous tuberculosisE. Cavity at fibrocavernous tuberculosis.35. The girl within last 3 months ate the crude cow milk. She arrived in the hospital with the "acute abdomen" symptoms. An operation revealed in a caecum a circular ulcer with perforation. A histological study of the ulcer edges' determined necrotic masses, lymphocytes, epithelioid cells, and few multinucleated giant cellsWhat is your diagnosis?A. * Primary intestinal affect at tuberculosisB. Nonspecific ulcerous colitisC. AmebiasisD. Carcinoma of blind gut (cecum)E. Dysentery36. A histological study of the enlarged cervical lymph nodes revealed nodules with giant, spherical-oval form cells. These cells had a light pink cytoplasm and nuclei located on a periphery. The found out changes are characteristic for:A. *TuberculosisB. Lymphogranulomatosis (Hodgkin's disease)C. Sarcoidosis (Boeck's disease)D. BrucellosisE. Nonspecific hyperplastic lymphadenitis37. A 63-year-old man with a long history of a tubercular prostatitis died of a meningocephalitis. An autopsy revealed in arachnoids of the basis and lateral surfaces of a brain, in a spleen, kidneys and liver a considerable quantity of dense grey color nodules 0, 5-lmm in diameter. Histological investigation of nodules showed epithelioid, lymphoid and some giant cells with nuclei located on a cell periphery in the form of a horseshoe. What is the most likely diagnosis?A. *Acute military tuberculosisB. Chronic milliary tuberculosisC. Acuter tubercular sepsisD. Macrofocal disseminated tuberculosisE. Septicopyemia38. A man died of liver cirrhosis. An autopsy revealed in the 1-2 segment of the right lung three grey-yellow, dense foci, 1-1,5см in diameter with a dim surface. A histological research showed the tubercular endobronchitis, foci of a caseous bronchopneumonia, surrounded with a wall of epithelioid, lymphoid and giant Pirogov-Langhans cells. For what form of a tuberculosis the described changes are characteristic?A. * Acute focal tuberculosisB. Infiltrative tuberculosisC. Fibrous-focal tuberculosisD. Caseous pneumoniaE. Primary pulmonary tuberculous complex39. An autopsy of a 40-year-old man with a history of AIDS revealed both lung lobes enlarged, dense, air free, with fibrinous membranes on pleura. A gross investigation of a cut surface showed the yellow dim pulmonal tissue. A histological investigation determined in alveoli the seroftbrinous and fibrinous exudate with large foci of a pulmonal tissue necrosis. For what disease the described changes in lungs are characteristic?A. * Caseous pneumoniaB. Infiltrative tuberculosisC. Croupous pneumoniaD. Lymphogranulomatosis of lungs (Hodgkin's disease)E. Carcinoma of lung40. A 47- year-old man died of an acute anemia, resulted from the pulmonary bleeding. An autopsy revealed in the 2 segment of the right lung the cavity, 5-6 cm in the size, which had connection with the bronchus lumen. The cavity had irregular roundish form and filled with blood. The internal surface of a cavity was rough and covered by a flabby unstructured yellowish tissue. A wall was thin, presented by the condensed, inflammatorily changed pulmonary tissue A histological investigation showed that the cavity inside layer consisted of the fused caseous

Page 120: Pat an Anew

120

masses with a considerable quantity of segmented leucocytes. What is the most likely pathology presented in that case?A. * Acute cavernous tuberculosisB. Abscess of lungC. Disintegrating carcinoma of lungD. Infarction of lung with septic disintegrationE. Bronchiectatic cavity (cavern)41. A 56-year-old man died of a progressing heart failure. A post-mortem revealed small, dense lungs; the upper lobes were deformed, penetrated by a cicatrical tissue, with the encapsulated foci of a caseous necrosis, 0, 2-0,5cm in diameter. In the upper and middle lobes there were cavities, 3-4 and 4-5cm in the size, with dense walls and fiber internal surface, filled with the opaque yellow-green liquid. The surrounding pulmonary tissue was sclerotic. The heart was enlarged at the expense of the hypertrophied right ventricle. Define the form of secondary pulmonary tuberculosisA. * Cirrhotic tuberculosisB. Fibrous-focal tuberculosis.C. Infiltrative tuberculosisD. Acute cavernous tuberculosisE. Fibrocavernous tuberculosis42. A post-mortem of a 72-year-old vagabond, with marked kypho-scoliosis, revealed the partially destructed thoracal vertebral bodies due to formation of cavities (sequesters), filled with white-yellow brittle masses. A histological investigation determined within altered vertebras big foci of a caseous necrosis, surrounded with solitary giant multinucleated Pirogov-Langhans' cells, epithelioid cells and lymphocytes. Similar changes were diagnosed at mediastinum lymph nodes and disseminated in lungs and kidneys. What is the most likely diagnosis?A. *Tuberculous osteomyelitisB. Purulent osteomyelitisC. Syphilitic osteomyelitisD. OsteoporosisE. Osteoarthrosis43. An ultrasound investigation of the young 17-year-old man's right kidney revealed pyeloectasis, accompanied with disturbance of outflow of urine by urethra. Microscopical investigation of the renal biopsy showed a diffusive infiltration of the interstitium by lymphocytes and hystiocytes. In the cortical and medullar zones there were the foci of a destruction with formation of the cavities, surrounded by peripheral mantle of epithelioid , lymphoid cells and some multinucleated Pirogov-Langhans' giant cells. What disease is the most probable?A. * Tuberculosis of kidneyB. Suppurative (apostematous) nephritisC. Nonspecific tubular-interstitial nephritisD. Polycystic kidneyE. Chronic pyelonephritis44. An autopsy of a child, who died of meningocephalitis revealed under pleura and in pulmonal tissue plural, punctual, yellow-white lesions which reminded millet grains. Histological investigation showed granulomas with a necrosis zone in the centre and a wall of epithelioid cells, lymphocytes, macrophages and some giant Pirogov -Langhans cells. What is the most likely disease?A. *Primary milliary tuberculosisB. Secondary tuberculosisC. Focal pneumoniaD. Croupous pneumonia45. A medical record of a dead child reveled meningeal symptoms in his medical history. His X-ray showed a dark patch in the III segment of the right lung and enlarged radical lymph nodes. At autopsy, in cerebral arachnoids, the millet grains-like nodules were found. Microscopical study of a nodule determined a focus of a caseous necrosis with wall of epithelioid, lymphoid cells with giant cells between them. These giant cells had big nuclei, situated on periphery in the form of a half moon. What is the most likely kind of meningitis?A. *TuberculousB. SyphiliticC. BrucellousD. Influenza!E. MeningococcaiSYPHILIS1. A 10-year-old boy presented with Hutchinson's triad: parenchymatous keratitis, sensorineural deathess and deformed Hutchinson's teeth (the barrel form), along with saddle-nose deformity.For what illness the presented changes are characteristic?A. * Syphilis.B. ToxoplasmosisC. Leprosy.

Page 121: Pat an Anew

121

D. Tuberculosis.E. Opisthorchiasis.2. An autopsy of a 36-year-old man revealed in his liver s roundish formation, 0,5 cm in diameter. Microscopical investigation showed in the centre of lesion necrotic masses, surrounded by the granulation tissue with some plasmatic and lymphoid cells. There were also signs of the vasculitis in the blood vessels. What is the most likely diagnosis in that case?A. *Solitary gumma of liverB. Solitary adenoma of liver.C. Solitary leproma of liver.D. Chronic abscess of liver.E. Carcinoma of liver.3. An autopsy of a 54-year-old man reveled above valvular rupture of an aorta with resulted in a cardiac tamponade. Histological investigation of ascending part of an aorta showed in its external and middle layers inflammatory infiltrates. They consisted of lymphoid, plasmatic and epithelioid cells. There were also recognized the necrosis centers and proliferation of a vessels' wall. Changes in an aorta are characteristic for:A. * Syphilitic aortitisB. Septic aortitisC. Rheumatic aortitisD. AtherosclerosisE. Hypertension diseases4. A 20-year old woman presented with enlarged, not painful, condensed inguinal lymph nodes. In her genital mucosa there was a small ulcer with firm edges and the "varnished" grayish color floor. What is the most likely diagnosis?A. * SyphilisB. Tuberculosis.C. Leprosy.D. Trophic ulcer.E. Gonorrhea.5. A biopsy of cervix uteri was taken from the 34-year-old woman. Histological investigation revealed the inflammatory infiltration with involving of fine arteries and veins walls. The infiltrate composed of plasmocytes, lymphocytes, and epithelioid cells. There were also zones of sclerosis and a tissue hyalinosis. What is the most likely disease?A. * SyphilisB. TuberculosisC. LeukoplakiaD. Cervical erosionE. Condyloma6. The prepuce was removed from a young man. A microscopical investigation revealed the polymorphic infiltrate consisting from randomly located plasmatic, lymphoid and epithelioid cells. There was marked vessels' vasculitis. For what disease the described changes are characteristic?A. *SyphilisB. TuberculosisC. ActinomycosisD. Periarteritis nodosaE. Leprosy7. A man's oral cavity investigation revealed an oval, cartilaginous density, slightly elevated ulcer at his buccal mucosa. A meaty, reddened floor was covered with grayish sebaceous deposits. A microscopical investigation showed lymphocyte infiltration, situated mainly round fine vessels. There was also an endothelium proliferation within the vessel's walls..What disease has developed in that case?A. * SyphilisB. Traumatic ulcerC. Erosive-ulcerated leukoplakiaD. Ulcer-CarcinomaE. Ulcero-necrotic Vensan's stomatitis8. A histological study of a biopsy from an oral cavity ulcer revealed the foci of caseous necrosis surrounded with plasmocytes, epithelioid , lymphoid cells and solitary giant multinucleated Pirogov-Langhans' cells. There were also signs of endo - and a perivasculitis in fine vessels. Specify the disease.A. * SyphilisB. TuberculosisC. LeprosyD. RhinoscleromaE. GlandersSEPSIS

Page 122: Pat an Anew

122

1. A 48-year-old patient with a history of croupous pneumonia died of the pulmonary and heart failure. A post-mortem revealed a hyperplasia of a spleen, a thickening and defects of the aortal valve leaflet with large, up to 2 cm in diameter, thrombuses attached to tissues underneath. What is the most likely diagnosis?A. * Polypous-ulcerative endocarditisB. Acute warty endocarditisC. Recurrent warty endocarditisD. Diffuse endocarditisE. Fibroplastic endocarditis2. An autopsy of a 48-year-old man revealed the infarct of the left hemisphere of a brain. A macroscopic study also showed a big septic spleen, immunocomplex glomerulonephritis, ulcers in aortal valve leaflets, covered with polypous thrombi. The latter ones had colonies of staphilococcuses, determined by microscopic study. What disease resulted in cerebral thromboembolism?A. * Septic bacterial endocarditisB. SepticemiaC. Acute rheumatic valvulitisD. SepticopyemiaE. Rheumatic thromboendocarditis3. A 46-year-old man died of an odontogenic sepsis. An autopsy revealed the marked thickening of semilunar aortic valves. They were whitish, opaque and stiff, with thrombotic masses, 1x1.5 cm in the size on the external surface. What is the most likely form of an endocarditis?A. * Polypous-ulcerative endocarditisB. Diffuse endocarditisC. Acute warty endocarditisD. Fibroplastic endocarditisE. Recurrent warty endocarditis4. A 32-year-old man with a long history of a drug abuse, presented with a fever, the intoxication and a pyuria. An echocardioscopy revealed massive superimpositions (vegetations) on the heart valves. Microbiological blood test showed a presence of a staphylococcus. The patient died of a thromboembolism of a pulmonary artery. Specify the changes of heart, revealed at post-mortem?A. * Polypous-ulcerative endocarditisB. Libman-Sacks endocarditisC. Diffuse valvulitisD. Acute warty endocarditisE. Recurrent warty endocarditis5. A young woman with a history of the out-of-hospital abortion presented with advanced purulent endomyometritis resulted in a lethal outcome. A post-mortem revealed numerous abscesses of lungs, pustules under kidneys' capsule and spleen hyperplasia. What form of a sepsis has developed?A. * SepticopyemiaB. SepticemiaC. ChroniosepsisD. Lung sepsisE. Urosepsis6. A 30-year-old woman with a history of abortion presented with purulent endometritis resulted in a death from sepsis. A post-mortem revealed a purulent lymphadenitis, abscesses in lungs, kidneys, a myocardium, a spleen hyperplasia, and dystrophy of parenchymatous organs. Define the clinico-anatomic form of a sepsis.A. * SepticopyemiaB. SepticemiaC. Septic endocarditisD. Granulomatous sepsisE. Chroniosepsis7. A 28-year-old woman died in the postnatal period. A post-mortem revealed a purulent endometritis and thrombophlebitis of a uterus veins, plural abscesses of lungs, kidneys and a spleen. In addition, there were an apostematous myocarditis and purulent meningitis. What is the most likely kind of a sepsis?A. * SepticopyemiaB. SepticemiaC. ChroniosepsisD. The prolonged septic endocarditis E.8. A man with a history of a lower extremity wound, presented with persistent pyesis of a lesion, resulted in death of intoxication. An autopsy revealed a cachexia, dehydration; a brown atrophy of a liver, a myocardium, a spleen and cross-striated muscles. In addition, there was amyloidosis of kidneys. What of the listed diagnoses is most probable?A. * Chroniosepsis

Page 123: Pat an Anew

123

B. SepticopyemiaC. Septicemia.D. Chernogubov's disease.E. Brucellosis9. A post-mortem revealed on the external surface of the aortal valve the large, 1-2 cm, brownish-red, easily crumbling superimpositions (vegetations). They covered ulcerative defects in the valves. What is the most likely diagnosis?A. * Polypous-ulcerative endocarditisB. Recurrent warty endocarditisC. Acute warty endocarditisD. Fibroplastic endocarditisE. Diffuse endocarditisDISEASE OF THE NERVOUS SYSTEM1. An autopsy of the elderly woman revealed a massive purulent bronchopneumonia. A brain was slightly atrophic, gyri were thinned and sulci were deep. A microscopical study of a brain showed perivascular deposits of amyloid, Hirano bodies and neurofibrillary "tangles".What is the most likely main disease?A. *Alzheimer's diseaseB. Pick's diseaseC. Parkinson's diseaseD. Disseminated sclerosisE. Lateral amyotrophic sclerosis2. An autopsy of the 78-year-old woman revealed a massive purulent bronchopneumonia. There were numerous grey color plaques, up to 15 mm in diameter, found in the brain and spinal cord. A microscopical study of brain and spinal cord tissue showed plural perivenular sites of a demyelization with limphoplazmocytes infiltration and the plural foci of gliosis. Diagnose a basic disease.A. * Disseminated sclerosisB. Pick's diseaseC. Parkinson's diseaseD. Alzheimer's diseaseE. Lateral amyotrophic sclerosis3. A 42- year-old forester, a week later after long stay in spring wood, presented to the hospital with a fever, a headache, consciousness disturbance and epileptoid attacks. He died in 4 days. An autopsy revealed a brain edema, plural petechial hemorrhages. A microscopical study showed perivascular and pericellular edema, plural perivascular, mainly lymphocytic, infiltrates. Diagnose the basic disease.A. Tick-borne encephalitisB. Meningococcal infectionC. PoliomyelitisD. Purulent encephalitisE. Cerebro-vascula disease4. An autopsy of the 68-year-old man revealed the atrophy of brain's tissue. The cerebral atrophy mainly involved the frontal, temporal, and occipital regions. A hydrocephaly was also found. A microscopical study showed atrophic zones in the cerebral cortex, the senile (neuritic) plaques in hippocampus, neurofibrillary "tangles", damage of neurons and Hirano bodies. What is the most likely main disease?A. * Alzheimer's diseaseB. Lateral amyotrophic sclerosisC. Disseminated sclerosisD. EncephalitisE. Ganglioneuroma5. A 75-year-old man, with a history of Alzheimer's disease, died of a bronchopneumonia. A microscopical study of the cerebral cortex revealed the senile (neuritic) plaques, neurofibrillary "tangles" and damaged neurons. What formations characterize cytoskeleton pathology of proximal dendrites at the Alzheimer's disease?A. *Hirano bodiesB. Councilman's bodiesC. Negri bodiesD. Mallory's bodiesE. Russell bodies6. An elderly man presented to the hospital with progressive dementia (aphrenia), with marked emotional lability; thus the focal neurologic symptoms were absent. After his death, a post-mortem revealed the cortex thinning in frontal, occipital and temporal lobes of the brain. A histological study of a brain's tissue showed the considerable quantity of senile plaques (especially in a cortex), reduction of neurones' sizes with cytoplasm vacuolation. In addition, the little Hirano bodies were determined in dendrites. What is the most likely pathology in the central nervous system?

Page 124: Pat an Anew

124

A. * Alzheimer's diseaseB. Lateral amyotrophic sclerosisC. Disseminated sclerosisD. Tick-borne encephalitisE. Viral meningoencephalitisDISEASE OF AN OSTEOMUSCULAR SYSTEM1. A morphological research of a femur reveled the chronic purulent inflammation of compact substance and a bone marrow with sequesters formation. What is the most likely disease?A. * OsteomyelitisB. RetikulosarcomaC. Multiple (plasma cell) myeloma (Kahler's disease)D. OsteoblastoclastomaE. Periostitis2. A 6-year-old girl presented to the hospital with suddenly raised temperature and catarrh of respiratory tracts. For 5th day of illness there was a paralysis of her lower extremities muscles, accompanied with respiratory disorders. In the anterior horns of a spinal cord there was a proliferation of a glia round necrotic neurons. What is the most likely diagnosis?A. *PoliomyelitisB. MeaslesC. DiphtheriaD. MeningococcemiaE. Scarlet fever3. A 48- year-old man presented with advanced staphylococcal purulent periodontitis, resulted in a purulent inflammation of alveolar process' bone marrow and then a mandible's body. Microscopical investigation showed thinned osteal beams, the necrosis centers and the osteal sequesters, surrounded with a connective tissue's capsule. What is the most likely disease?A. *Chronic osteomyelitisB. Acute osteomyelitisC. ParodontomeD. Chronic fibrous periostitisE. Purulent periostitis4. A 52-year-old man with a 10 years history of chronic osteomyelitis and 3 years a nephrotic syndrome, died of uremia. An autopsy revealed dense, white kidneys, with some cicatrices in their cortical layer. They had sebaceous glitter on a cut surface. Specify a pathology of kidneys which has developed.A. * Secondary amyloidosisB. Primary amyloidosisC. Idiopathic amyloidosisD. Chronic glomerulonephritisE. Chronic pyelonephritis5. A 53- year-old man died of intoxication. An autopsy revealed the enlargement of his hip with some fistulas on a skin. There was yellow - green thick liquid discharge from the fistulas. The gross investigation showed the thickened femur with sequesters formation. A microscopical study determined sequestra! cavities, surrounded by granulation and connective tissue with neutrophil infiltration. The bone-marrow channels were obliterated; the compact layer was thickened. What is the most likely diagnosis?A. *Chronic purulent osteomyelitisB. Acute gematogenic osteomyelitisC. Tuberculous osteomyelitisD. Parathyroid osteodystrophyE. Osteopetrosis6. A 20-year-old woman presented with a several years history of muscular weakness, resulted in the motion, talking, chewing and swallowing problems. A physical examination revealed the ptosis of both eyelids. The thorax roentgenography found a tumorous conglomerate in anterior mediastinum. Histological investigation of the tumor biopsy determined a thymoma. There were also some dystrophic alterations in skeletal muscles. Diagnose the disease.A. * MyastheniaB. Progressive muscles dystrophyC. Muscles Erb's dystrophyD. DermatomyositisE. Poliomyelitis7. A 20-year-old woman with a history of pathological cross- striated muscular weakness and ptosis died of asphyxia. An autopsy revealed the follicular hyperplasia of a thymus, atrophy and a necrosis of skeletal muscles with focal lymphocytic infiltrates among the cells. Specify the form of a muscles lesion.A. * Myasthenia

Page 125: Pat an Anew

125

B. Atrophy myositisC. Progressive myopathyD. Mussels Leyden's dystrophyE. Mussels Erb's dystrophy8. An autopsy of a patient, with a lifetime history of numerous pathological fractures, revealed changes of his long tubular bones. The femurs and tibia bones were curved or spiral with uneven surface and the obliterated bone-marrow channels on a cut. There was also a remodeling of a compact cortical layer into the spongy type. Microscopical investigation showed mosaicity. of osteal structures. There were numerous cavities of the resorptions combined with osteoblastic lying down of new bone. These were accompanied with chaotic fine-fibered or lamellar structures of osteal fragments. The arteries were twisted and dilated. What is the most likely diagnosis?A. * Deforming osteodystrophyB. OsteopetrosisC. Parathyroid osteodystrophyD. Fibrous dysplasiaE. Chronic osteomyelitis9. A 20-year-old woman presented with weakness of the eye, chewing, speech and swallowing groups of muscles. Thus, the normal muscles contraction were completely ceased after the vigorous activity and restored again after the rest. Later, the muscles of extremities and intercostals were also involved in pathological process. Inadequate ventilation of the lungs resulted in secondary focal pneumonia, followed by patient's death. An autopsy showed the atrophy of cross-striated muscles, their dystrophy with focal aggregates of lymphocytes in interstitium. In the enlarged thymic gland the follicular hyperplasia was found out. Specify the most probable diagnosis:A. * MyastheniaB. Verdniga-Goffman's spinal amyotrophyC. Dyushen's mussels dystrophyD. Lateral amyotrophic sclerosis.E. Erb's mussels dystrophyMYCOSISES.1. A 48-year-old patient presented to the hospital with deep cerebral coma. It was known, that during lifetime he experienced periodic attacks of a fever. An autopsy revealed grey-flaky colour of a bone marrow, lymph nodes; enlarged a liver and a spleen. Histological investigation showed the haemomelanosis and a hemosiderosis in mentioned organs. What of diagnoses is most probable?A. * MalariaB. Hemolytic anemiaC. Addison diseaseD. SepticemiaE. Black pox2. A 54-year-old man presented to the hospital with reddening and edema of the neck's skin, and then resulted in a small abscess formation. On a cut, a tissue was dense with yellow-green coloring. In purulent masses there were visible white grains. Histological study showed fungus' druses, plasmatic, xanthome cells, and macrophages. Specify the most probable kind of a mycosis.A. *ActinomycosisB. AspergillosisC. CandidosisD. SporotrichosisE. Coccidioidomycosis3. An autopsy of 48-year-old woman, resident of Middle Asia, with long history of exhausting fever, revealed enlarged liver and a spleen. They were grey-flaky color. The same color had hyperplastic bone marrow. A cerebral cortex also had grey-smoky coloring. What is the most likely diagnosis?A. * MalariaB. AIDSC. Epidemic typhusD. SepsisE. Hepatitis4. A 38-year-old man died of cerebral coma. It was known, that after foreign business trip in one of the African countries, the icteruses periodically developed. An autopsy revealed the enlarged, dense spleen, with a black pulp. A liver was also enlarged, hyperemic, and grey-black. A gross investigation of the brain determined brown-grey coloring of a grey matter and plural fine hemorrhages in white matter. What infectious disease should be suspected?A. * MalariaB. MeningococcemiaC. Prion infectionD. Generalized herpes infectionE. Generalized cryptococcosis

Page 126: Pat an Anew

126

5. A 69-year-old man presented to the hospital with edema and a painmlness of a skin and soft tissues of his neck. The phlegmon of a neck and a mediastinitis were diagnosed. The patient died. An autopsy revealed enlarged, dense left tonsil. On a cut, the tonsil was yellow-green, with set of fine cavities similar to porous structures. Microscopical investigation determined fine abscesses with basophilic formations in their centers. These formations consisted of short rhabdoid elements. What is the most likely diagnosis?A. *ActinomycosisB. BrucellosisC. AmebiasisD. LambliasisE. Leishmaniasis